You are on page 1of 232

What layer is absent in the vas deferens wall?

- submucosa
Name the cells of the epithelium of epididymis?
- Principle, basal
Establish the correspondence of the influence of certain factors on the structure of the testicle in
embryogenesis?
- SRY gene - mesonephric duct
- MIF - paramesonephric duct
What is the type of prostrate?
- tubuloalveolar
Establish a correspondence between the cell and the compartment in seminiferous tubule wall?
- Spermatogonia - basal compartment
- Secondary spermatocyte - adluminal compartment
Match the effect of pituitary hormones on testicular function?
- FSH - stimulate spermatogenesis
- LH - stimulate leydig cells
Name the type of developing male reproductive cell containing diploid set of chromosomes?
- Primary spermatocyte
What are the testicular cells that produce mullerian inhibiting factor?
- Sertoli cells
List the testicular tubules and ducts in sequence?
- 1. Convoluted seminiferous tubule
- 2.stratight tubules
- 3. Rete testis
- 4. Efferent tubules
- 5. Epididymis duct
- 6. Vas deference
What are the function of the epididymis duct?
- Promote the motor activity of sperm
- Maturation of sperm
Name the organs of the male reproductive system that develop from the intermediate
mesoderm?
- epididymis
- testis
What are the testicular cells that produce mullerian inhibiting factor?
- Sertoli cells
Name sequentially the components of the blood testis barrier/ start from capillary?
- 1. Endothelium of capillary
- 2. Basal membrane of capillary
- 3. Connective tissue
- 4.basal membrane of spermatogenic epithelium
- 5. Tight junction of sertoli cells
What are th unction of prostrate?
- Secretory product contains enzymes, vitamin etc and produce 70% of ejaculate
Respiratory system (right answers )

1) Stages of lung development – pseudo glamdular – cananicular – saccular – alveolar


2) Elastic cartilage of larynx – epiglottis
3) Terminal bronchiole – simple cuboidal ciliated epithelium , respiratory bronchiole – simple
cuboidal ciliated epithelium + alveoli
4) Principal bronchus – hyaline cartilage ,medium sized bronchus – elastic cartilage
5) Beginning of development of RS – 3-4 weeks
6) Stages – principal bronchus ,lobar bronchus ,segmentary bronchus ,terminal bronchus
7) Correspondence between epithelium and vocal cords of larynx – false cord : pseudostratified
columnar ciliated epithelium , true cord : stratified ,squamous non non keratinised epithelium
8) Epithelial cells of terminal bronchiole – Clara cell,ciliated cells
9) Names of gland in larynx – mixed glands
10) List components of blood air barrier from inside the alveolus to outside : surfactant ,type1
pneumocytes ,basement membrane , endothelial cells
11) Epithelial cells of nasal cavity olfactory portion : receptor and supporting
12) Diameter of alveoli - 100-140 micrometres
13)
Urinary system

1) Name the stage of development of kidney at which it is located in the pelvic region –
metanephros
2) Hormones that stimulate absorption of sodium in distal convulated tubules – aldosterone
3) Hormones that stimulate absorption of calcium in distal convulated tubules – pth
4) Which nephrons have straight vessels – long ( juxtmedullary )
5) Blood pressure and vessels – glomerular capillaries ( 50mm hg ) ,tubular capillaries ( 12 mm hg )
6) Cells of collecting tubules – dark cells ( secrete HCl ) , clear cells ( absorption of water )
7) Correspondence – distal convulated tubule :no microvilli on surface epithelium ,,,, proximal
convulated tubules : micro villi on surface epithelium
8) Location of structure of kidney :medulla : lobes ,,,,, cortex : lobules
9) Endocrine apparatus of kidney : renin- angiotensin ::: vasoconstriction , prostaglandin :::
vasodilation
10) What membrane is missing in bladder trigone : submucosa
11) Name source of kidney development – intermediate mesoderm
12) Pct -loop of Henle – dct – collecting duct
13) Blood vessels of kidney – 1) renal 2) interlobar 3) arcuate 4)interlobular 5(afferent
14) Name substances that donot pass through filtration barrier of kidney – amino acids
15)
Histology CP-3

Match the effect of pituitary hormones on testicular functions

Name the organ or structure shown in photo


Establish a correspondence between the cells of the Corpus luteum and the hormones they produce

Name the cells of epithelium of epididymis

Name the structure shown in photo as number 2

Respiratory bronchiole terminal bronchiole


Name the slide medium sized bronchus

What number indicates collecting duct


.Name the embryonic sources from which uterus develops
.Establish correspondence between the cell and the

compartment in seminiferous tubule wall

Name the structure marked in yellow in the photo


Name the organ and structure indicated by number 2

. Establish the correspondence between epithelium and vocal cords of larynx


List the tubules of kidney starting with the one that leaves the bowmans capsule

Name the slide cortex of kidney


Name the organ or structure shown in photo

.name the follicle in the Wall of which theca appears


. Establish the corresponding between the breast acini cell and the hormone that stimulates it

. Name the sources of kidney development


What is the hormone that stimulates the absorption of sodium in distal convulated tubules

Name the organ or structure shown in photo


.name the epithelial cells of nasal cavity olfactory portion

.. In what structures of the kidney does water reabsorption occur?


Descending limb of henles loop..Collecting ducts

...Establish a correspondence between the the tubule and the type of epithelium

.. establish a correspondence between blood pressure and vessels in the

Cortical nephrons.
Name the types of follicles containing oocytes with a diploid set of chromosomes

list the components of air blood barrier from inside the alveolus to outside
. What is letter indicating the pulmonary surfactant

What component of alveoli is indicated by the letter A: pulmonary surfactant


establish the correspondence between the tubule and the structure of the epithelial cell

Name epithelial cells of the nasal cavity olfactory portion

Establishing correspondence between the location of the structures of the kidney

What is the elastic cartilage of larynx


What is the type of prostate

Name a hormone of hypothalamus that stimulate pituitary cells to produce LH and fSH
Establish a correspondence between the type of gland and the part of uterus

What membrane is not part of the wall of the fallopian tube


Name the organ

What organ is it
Establish a correspondence between the breast cells and the hormone that stimulates it

What are the stages of lung development from early to late


Establish a correspondence between the bronchus and the type of cartilage in its wall

Name the slide


What are the stages of lung development from early to late

Establish a sequence from proximal to distal bronchi


.. name the structure indicated by number 3

Name the epithelial cells of terminal bronchiole


What is the diameter of the alveoli

What is the beginning of development of RS

Name the substances that do not pass through the filtration barrier of the kidney
List the blood vessels of the kidney carrying the blood to the nephron glomerulus

. what membrane is missing in the bladder trigone


.. establish the correspondence between the cells of collecting tubules and their functions

establish a correspondence between the endocrine apparatus of the kidney and its effect on blood
vessels

Name the stage of development of kidney at which it is located in the pelvic region..
list the tubules of the kidney starting with the one that leaves the bowman's capsule
What is the structure of the kidney represented by the number 11.1.

name hormone that stimulates the absorption of calcium in distal convoluted tubule
What are the testicular cells that produce mullerian inhibiting factor

..name the type of developing male reproductive cell containing diploid set of chromosomes
Name the structures indicated in the photo with letter A

Establish a correspondence between the cell and the compartment in seminiferous tubule wall
Name the successive changes in the ovary during the ovarian cycle

SRY gene

MIF
Name the organ
Name the follicle in the wall of which the theca appears

What are the structures of ovary that produce progesterone


Granulosa lutein cell

Theca lutein cell

Name the successive changes in the ovary during the ovarian cycle
Name a hormone that stimulates ovulation
Each of the following statements about leydig cells is correct except

Match the effect of pituitary hormones on testicular functions

Which number indicates the effrent tubules


the structure shown in the photo as number 2

What are the functions of sertoli cells

What are the functions of prostate


Epididymis duct

.. what are the functions of epididymis duct


Name structures indicated in photo with letter A

What layer is absent in the vas defrens wall


Name the follicle in which the oocytes do not have a zona pellucida

..

Establish a correspondence between the breast cell and the hormone that stimulates it
.establish a correspondence between the type of gland and the part of the uterus

name of hormone of hypothalamus that stimulate pituitary cells to produce a LH and FSH

What are the structures of the ovary that produces progesterone

Name the follicle in wall of which theca appears


.

What organ is it mammary gland

Name the type of follicles containing oocytes with a diploid set of chromosomes
.

Name the structures that are related to the indifferent gonad

Name the structures from which the uterus develops paramesonephric ducts and mesenchyme
Name the structure from which the uterus develops

Name the successive changes in the ovary during the ovarian cycle
.

name the organs of male reproductive system that developed from intermediate mesoderm

Testis epididymis
What are the functions of of epididymis duct
Name the cells of epithelium of epididymis

List the testicle tubules and ducts in sequence


Name sequentially the components of the blood testis barrier start from capillary

What are the functions of prostate


.

Name the structures shown in photo as number 2


KP

Результат #72128875

Дата завершения: 22.10.2020 14:49


Потрачено времени: 00:04:10

Ваше имя: Введите ваше имя

Ответы Результат
1

The diagram of the structure of the thymus is shown on the figure. What number corresponds to the epithelial Hassall`s
corpuskles ?

11

12

Неправильный ответ на вопрос


Баллов: 0 из 10

Правильный ответ:
11

Сообщить об ошибке (0)


2

Hematopoietic and stromal cells of the red bone marrow are shown in the figure. What number corresponds to a
megakaryocyte

Неправильный ответ на вопрос


Баллов: 0 из 10

Правильный ответ:

Сообщить об ошибке (0)


3

Which layer of epidermis corresponds to the following cell`s description:


It is layer of terminally differentiated cells, entirely filled with keratin filaments.

Stratum lucidum

Stratum granulesum

Stratum spinosum

Stratum basale

Stratum corneum

Неправильный ответ на вопрос


Баллов: 0 из 10

Правильный ответ:
Stratum corneum

Сообщить об ошибке (0)

6. Which supporting cells of Corti's organ form the tunnel?

border cells

pillar cells

tunnel cells

outer phalangeal cells

internal phalanx cells

Правильный ответ на вопрос


Баллов: 10 из 10

Сообщить об ошибке (0)

At newborn boy congenital involution of a thymus is revealed. Disturbance of what cells differentiation we may wait for?

B-memory cells

B-cells

T-cells

B-cells

Plasma cells

Неправильный ответ на вопрос


Баллов: 0 из 10

Правильный ответ:
T-cells

Сообщить об ошибке (0)


6

In the adenohypophysis, cells of irregular or angular shape are found, they have lobed nuclei and a well-defined
endoplasmic reticulum. The secretory granules are surrounded by a membrane and contain a dense protein core. The
hormone synthesized by them stimulates the secretory activity of the cells of zona fasciculata cells of the adrenal cortex.
Name these cells.

Lactotropocytes

Gonadotropocytes

Somatopropocytes

Corticotropocytes

Thyrotropocytes

Неправильный ответ на вопрос


Баллов: 0 из 10

Правильный ответ:
Corticotropocytes

Сообщить об ошибке (0)

Which middle ear formation regulates the air pressure in the tympanic cavity?

cochlear duct

tympanic membrane

auditory tube

auditory ossicles

oval window

Правильный ответ на вопрос


Баллов: 10 из 10

Сообщить об ошибке (0)


8

What constitutes the inner root sheath?

Cuticle, Henley`s layer, Haxley`s layer

Catagen, anagen, telogen

Cuticle, glassy membrane,

Cuticle, cortex, medulla

Cuticle, connective tissue layers

Неправильный ответ на вопрос


Баллов: 0 из 10

Правильный ответ:
Cuticle, Henley`s layer, Haxley`s layer

Сообщить об ошибке (0)

Which layer of epidermis corresponds to the following cell`s description:


Single layer of small mitotically active cells, that are attached underlying membrane by hemidesmosomes and to each
other by desmosomes.

Stratum granulesum

Stratum corneum

Stratum basale

Stratum spinosum

Stratum lucidum

Неправильный ответ на вопрос


Баллов: 0 из 10

Правильный ответ:
Stratum basale

Сообщить об ошибке (0)

10

A 10-14 % of newborn suffer from developmental anomalies. What embryonic origin do follicular cells of thyroid gland
develop from?

Mesenchyme

Endoderm

Ectoderm

Intermediate mesoderm

Splanchnic mesoderm

Правильный ответ на вопрос


Баллов: 10 из 10

Сообщить об ошибке (0)


11

The spiral ganglion is affected by a chronic inflammatory process. What functional changes will be noted in the animal?

perception of sounds is impaired

perception of linear accelerations is impaired

perception of angular accelerations is impaired

perception of gravity is impaired

perception of vibration is impaired

Неправильный ответ на вопрос


Баллов: 0 из 10

Правильный ответ:
perception of sounds is impaired

Сообщить об ошибке (0)

12

For the treatment of osteoporosis in the elderly, hormone therapy is used. What hormone enhances calcium deposition in
bones?

Parathormone

Mineralocorticoid hormones

Epinephrine

Tetraiodthyronine

Calcitonin

Неправильный ответ на вопрос


Баллов: 0 из 10

Правильный ответ:

Calcitonin

Сообщить об ошибке (0)

13

A 2-year- old child experienced convulsions because of lowering calcium ions concentration in the blood plasma. Function
of what structure is decreased?

Adrenal medulla

Hipophysis

Parathyroid glands

Thyroid gland

Adrenal cortex

Неправильный ответ на вопрос


Баллов: 0 из 10

Правильный ответ:

Parathyroid glands
Сообщить об ошибке (0)

14

Characterizing a stress, the student has supposed an inaccuracy when has told, that synthesis of glucocorticoids of a
suprarenal cortex is stimulated by hormones of a pituitary gland. What clarification is necessary?

Gonadotropic hormones

Тhyrotropic hormone

Prolactin

Аdrenocorticotropic hormon (ACTH)

Somatotropin

Неправильный ответ на вопрос


Баллов: 0 из 10

Правильный ответ:
Аdrenocorticotropic hormon (ACTH)

Сообщить об ошибке (0)


15

Determine the histological specimen

Adrenal gland cortex

Hypophysis

Thyroid gland

Parathyroid gland

Adrenal gland medulla

Неправильный ответ на вопрос


Баллов: 0 из 10

Правильный ответ:
Adrenal gland cortex

Сообщить об ошибке (0)


16

Name mechanism of hormonal control, if the hormones are secreted from some cells and act on the adjacent cells that
express specific receptors

Paracrine

Adjacent

Autocrine

communicative

Endocrine

Неправильный ответ на вопрос


Баллов: 0 из 10

Правильный ответ:

Paracrine

Сообщить об ошибке (0)

17

Determine the histological specimen

Parathyroid gland

Adrenal gland medulla

Thyroid gland

Hypophysis

Adrenal gland cortex

Неправильный ответ на вопрос


Баллов: 0 из 10

Правильный ответ:

Parathyroid gland

Сообщить об ошибке (0)


18

It is determined the blood vessel on the histological micropreparation of the spleen.Its wall consists of tunica intima
(endothelium, basal membrane), lack
of the distinct tunica media, tunica adventitia (contains collagen and elastic fibers, that anchor the vessel in the
surrounding connective tissue trabeculae.
What type of vessel is it?

Muscular vein

Elastic artery

Muscular artery

Fibrous vein

Arteriole

Неправильный ответ на вопрос


Баллов: 0 из 10

Правильный ответ:
Fibrous vein

Сообщить об ошибке (0)

19

A 10-14 % of newborn suffer from developmental anomalies. What embryonic origin do cells of suprarenal gland medulla
develop from?

Mesenchyme

Ectoderm

Endoderm

Neural crest

Coelomic ntermediate mesoderm

Неправильный ответ на вопрос


Баллов: 0 из 10

Правильный ответ:
Neural crest

Сообщить об ошибке (0)


20

At a person genetic disorder of metabolism of amino acid tyrosin broke synthesis of a black pigment. In what cells of loose
connective tissue and a skin epidermis lack of this pigment will be observed?

Adipose cells

Mast cells

Dendritic cells

Melanocytes

Pericytes

Неправильный ответ на вопрос


Баллов: 0 из 10

Правильный ответ:
Melanocytes

Сообщить об ошибке (0)

21

What is the embryonic origin of Langerhans cells?

Ectoderm

Neural tube

Neural crest

Endoderm

Mesenchyme

Неправильный ответ на вопрос


Баллов: 0 из 10

Правильный ответ:
Mesenchyme

Сообщить об ошибке (0)

22

What layer of epidermis performs function of regeneration:

Stratum basale

Stratum corneum

Stratum spinosum

Stratum granulesum

Stratum lucidum

Неправильный ответ на вопрос


Баллов: 0 из 10

Правильный ответ:
Stratum basale

Сообщить об ошибке (0)


23

A 10-14 % of newborn suffer from developmental anomalies. What embryonic origin do cells of suprarenal gland cortex
develop from?

Ectoderm

Mesenchyme

Endoderm

Coelomic ntermediate mesoderm

Neural crest

Неправильный ответ на вопрос


Баллов: 0 из 10

Правильный ответ:
Coelomic ntermediate mesoderm

Сообщить об ошибке (0)

24

During road accident at the man of 30 years the femoral artery has been injured. In hospital the arteriography was
performed. Regeneration of what muscle tissue is necessary for regeneration of a wholeness of the vessel’s tunica media.

Striated skeletal muscle tissue

Smooth muscle tissue

Striated cardiac muscle tissue

Satellite cells

Myoepithelial cells

Неправильный ответ на вопрос


Баллов: 0 из 10

Правильный ответ:
Smooth muscle tissue

Сообщить об ошибке (0)

25

Stromal compartment of some organ is composed of branch-shaped epithelial cells that form tri-dimensional meshwork.
What is this organ?

Red bone marrow

Lymph node

Tonsil

Thymus gland

Spleen

Неправильный ответ на вопрос


Баллов: 0 из 10

Правильный ответ:

Thymus gland
Сообщить об ошибке (0)

26

A 10-14 % of newborn suffer from developmental anomalies. What embryonic origin do cells of parthyroid gland develop
from?

Splanchnic mesoderm

Ectoderm

Mesenchyme

Coelomic ntermediate mesoderm

Endoderm

Неправильный ответ на вопрос


Баллов: 0 из 10

Правильный ответ:
Endoderm

Сообщить об ошибке (0)

27

It is known, that aldosterone controls water and electrolyte balance in the organism. Which of the following cell types of
adrenal gland produce this hormone?

Cells of Zona glomerulosa

Cells of Zona fasciculata

Norepinephrine-secreting cells

Cells of Zona reticularis

Epinephrine- secreting cells

Неправильный ответ на вопрос


Баллов: 0 из 10

Правильный ответ:

Cells of Zona glomerulosa

Сообщить об ошибке (0)

28

What type of hormones does the follicle-stimulating hormone refer to according to its chemical nature?

Mineral salts

Сarbohydrate compounds

Proteins and polypeptides

Amino-acid derivatives

Steroid and fettyacid derivatives

Правильный ответ на вопрос


Баллов: 10 из 10

Сообщить об ошибке (0)


29

Here is a scheme of the structure of the olfactory epithelium. Indicate, what number coresponds the olfactory cilia?

Неправильный ответ на вопрос


Баллов: 0 из 10

Правильный ответ:

Сообщить об ошибке (0)

30

What is the embryonic origin of melanocytes?

Ectoderm

Neural tube

Endoderm

Neural crest

Mesenchyme

Неправильный ответ на вопрос


Баллов: 0 из 10

Правильный ответ:

Neural crest

Сообщить об ошибке (0)


31

Examination of a patient who was subjected to X-ray irradiation revealed injury of the splenic white pulp. What cells of the
white pulp undergo to these pathological changes at first?

Basophils

Splenic cells

Lymphocytes

Neutrophils

Red blood cells

Неправильный ответ на вопрос


Баллов: 0 из 10

Правильный ответ:
Lymphocytes

Сообщить об ошибке (0)

32

In the adenohypophysis, acidophilic endocrinocytes with large oval-shaped granules reaching up to 500-600 nm in length
are found. The hormone secreted by them activates the biosynthesis of milk in the mammary gland. Name these cells.

Gonadotropocytes

Lactotropocytes

Somatopropocytes

Corticotropocytes

Thyrotropocytes

Правильный ответ на вопрос


Баллов: 10 из 10

Сообщить об ошибке (0)


33

What the cytoplasm of melanocytes contains?

Lamellar bodies

Birbeck`s granules

Pacininian bodies

Melanosomes

Keratohyalin granules

Неправильный ответ на вопрос


Баллов: 0 из 10

Правильный ответ:
Melanosomes

Сообщить об ошибке (0)

34

Find on the lymph node diagram the germinative center of the lymph nodulus

Неправильный ответ на вопрос


Баллов: 0 из 10

Правильный ответ:
6

Сообщить об ошибке (0)


35

At a vulgar pemphigus in a skin epidermis forms


vesicles, which breaks out a
wholeness of stratum spinosum and stratum
granulosum. What function of an
epithelium is broken first of all?

Barrier

Absorbtive

Endocrine

Regenerative

Secretory

Неправильный ответ на вопрос


Баллов: 0 из 10

Правильный ответ:
Barrier

Сообщить об ошибке (0)


36

The inner surface of the blood and lymph vessels is lined with epithelium. Name this type of epithelium.

Endothelium

Stratified squamous non-keratinizing epithelium

Mesothelium

Epidermis

Urothelium

Неправильный ответ на вопрос


Баллов: 0 из 10

Правильный ответ:
Endothelium

Сообщить об ошибке (0)


37

The kinocilia and stereocilia of the hair cells of cristae ampullares of the semicircular canals are immersed in …

cupulae gelatinosa

the auditory membrane

tectorial membrane

otolithic membrane

hair membrane

Неправильный ответ на вопрос


Баллов: 0 из 10

Правильный ответ:
cupulae gelatinosa

Сообщить об ошибке (0)

38

Upward invagination of ectoderm lining primitive oral cavity forms the Rathke’s pouch. Which of the following structures
does develop from this embryonic origin?

Median eminence

Neurohypophysis

Adenohypophysis

Neural stark

Pars distalis

Правильный ответ на вопрос


Баллов: 10 из 10

Сообщить об ошибке (0)

39

It is determined the large, oval and round cells with light cytoplasm and a small number of organelles and myofibrils on
histologic micropreparation of the heart wall. What are these cells?

Contractile cardiac muscle

Purkinje fibers

Secretory cardiac muscle

P- cells

Smooth muscle

Неправильный ответ на вопрос


Баллов: 0 из 10

Правильный ответ:
Purkinje fibers

Сообщить об ошибке (0)


40

What cells of the cochlear duct produce endolymph?

cells of stria vascularis

hair cells

pillar cells

phalangeal cells

lymphocytes

Неправильный ответ на вопрос


Баллов: 0 из 10

Правильный ответ:
cells of stria vascularis

Сообщить об ошибке (0)

41

There are acidophilic endocrinocytes with globular granules 350-400 nm in diameter In the adenohypophysis , secreting a
hormone that regulates the growth of the body. Name these cells.

Corticotropocytes

Lactotropocytes

Thyrotropocytes

Gonadotropocytes

Somatopropocytes

Неправильный ответ на вопрос


Баллов: 0 из 10

Правильный ответ:

Somatopropocytes

Сообщить об ошибке (0)


42

Determine the histological specimen

Adrenal gland cortex

Adrenal gland medulla

Thyroid gland

Parathyroid gland

Hypophysis

Неправильный ответ на вопрос


Баллов: 0 из 10

Правильный ответ:
Adrenal gland cortex

Сообщить об ошибке (0)

43
Which number in the picture does the inner elastic membrane correspond to?

Неправильный ответ на вопрос


Баллов: 0 из 20

Правильный ответ:
2

Сообщить об ошибке (0)


44

In experiment the significant amount of stem cells of bone marrow definitely eroded. Regeneration of what population of
cells in composition of loose connective tissue will be inhibited.

Pericytes

Adipose cells

Pigment cells

Macrophages

Fibroblasts

Неправильный ответ на вопрос


Баллов: 0 из 10

Правильный ответ:

Macrophages

Сообщить об ошибке (0)

45

Which layer of the epidermis cells are shown in the electron micrograph

Stratum granulesum

Stratum spinosum

Stratum corneum

Stratum basale

Stratum lucidum

Правильный ответ на вопрос


Баллов: 10 из 10
Сообщить об ошибке (0)

46

At survey of the patient the poor amount of immunoglobulins has been detected. What from the enumerated cells of an
immune system produce immunoglobulins?

B-cells

Plasmablasts

T-helper cells

Т-killers cells

Plasma cells (plasmocytes)

Неправильный ответ на вопрос


Баллов: 0 из 10

Правильный ответ:
Plasma cells (plasmocytes)

Сообщить об ошибке (0)

47

What hormone is needed for the synthesis of milk in the mammary gland

Oxytocin

Prolactin

Production of progesteron by the corpus luteum

Placental lactogen

The influence of vasopressin

Неправильный ответ на вопрос


Баллов: 0 из 10

Правильный ответ:

Prolactin

Сообщить об ошибке (0)

48

They are determined cardiac muscle cells with glycoprotein rich granules. What are these cells?

P-cells

Purkinje cells

Contractile

Secretory

Transitional cells

Правильный ответ на вопрос


Баллов: 10 из 10

Сообщить об ошибке (0)


49

On the histological microslide, the vessels of the microvasculature are determined. What number of the nucleus of smooth
myocytes located in arterioles correspond to

Неправильный ответ на вопрос


Баллов: 0 из 10

Правильный ответ:

Сообщить об ошибке (0)

50

Name gel-like mass,which is mostly the protein thyroglobulin (pink) and bound thyroid hormones (triiodothyronine and
thyroxin) and fills every follicle.

Thyroid fluid

Hormonal mass

Follicular fluid

Gel mass

Colloid

Неправильный ответ на вопрос


Баллов: 0 из 10

Правильный ответ:

Colloid

Сообщить об ошибке (0)

Не знаете как сохранить результат? Отправьте его на email

Комментарии
Пока нет комментариев ... Добавить

Яндекс.Директ

Хотите от‐ Вибросито


крыть свою Sweco –
онлайн Круглые
школу?
marketinator.ru sweco.ru.com
16+
KP

Результат #72124525

Дата завершения: 22.10.2020 14:30


Потрачено времени: 00:07:00

Ваше имя: Введите ваше имя

Ответы Результат

At newborn boy congenital involution of a thymus is revealed. Disturbance of what cells differentiation we may wait for?

B-cells

Plasma cells

B-cells

T-cells

B-memory cells

Неправильный ответ на вопрос


Баллов: 0 из 10

Правильный ответ:

T-cells

Сообщить об ошибке (0)


2

Determine the histological specimen

Thyroid gland

Parathyroid gland

Adrenal gland medulla

Hypophysis

Adrenal gland cortex

Неправильный ответ на вопрос


Баллов: 0 из 10

Правильный ответ:

Parathyroid gland

Сообщить об ошибке (0)

Characterizing a stress, the student has supposed an inaccuracy when has told, that synthesis of glucocorticoids of a
suprarenal cortex is stimulated by hormones of a pituitary gland. What clarification is necessary?

Аdrenocorticotropic hormon (ACTH)

Тhyrotropic hormone

Prolactin

Gonadotropic hormones

Somatotropin

Неправильный ответ на вопрос


Баллов: 0 из 10

Правильный ответ:
Аdrenocorticotropic hormon (ACTH)

Сообщить об ошибке (0)


4

In experiment the significant amount of stem cells of bone marrow definitely eroded. Regeneration of what population of
cells in composition of loose connective tissue will be inhibited.

Adipose cells

Fibroblasts

Macrophages

Pigment cells

Pericytes

Правильный ответ на вопрос


Баллов: 10 из 10

Сообщить об ошибке (0)

What hormone is needed for the synthesis of milk in the mammary gland

Placental lactogen

Production of progesteron by the corpus luteum

Oxytocin

Prolactin

The influence of vasopressin

Неправильный ответ на вопрос


Баллов: 0 из 10

Правильный ответ:

Prolactin

Сообщить об ошибке (0)


6

On the histological microslide, the vessels of the microvasculature are determined. What number of the nucleus of smooth
myocytes located in arterioles correspond to

Неправильный ответ на вопрос


Баллов: 0 из 10

Правильный ответ:
5

Сообщить об ошибке (0)

Find on the lymph node diagram the germinative center of the lymph nodulus

Неправильный ответ на вопрос


Баллов: 0 из 10

Правильный ответ:
6

Сообщить об ошибке (0)


8

It is known that the capillary endothelium releases prostacyclin into the bloodstream. What is the function of the capillaries
provided by this?

Transport.

Oxygen exchange.

Nutrient exchange.

Blood clotting.

Disaggregation

Неправильный ответ на вопрос


Баллов: 0 из 10

Правильный ответ:
Disaggregation

Сообщить об ошибке (0)

A 2-year- old child experienced convulsions because of lowering calcium ions concentration in the blood plasma. Function
of what structure is decreased?

Thyroid gland

Parathyroid glands

Adrenal cortex

Adrenal medulla

Hipophysis

Неправильный ответ на вопрос


Баллов: 0 из 10

Правильный ответ:
Parathyroid glands

Сообщить об ошибке (0)

10

Pinealocytes of the pineal gland undergo atrophy with ages, and the deposition of phosphate and carbonate salts in the
form of layered balls increases in the pineal gland. What are the education data called?

Brain sand

Otolithic membrane

Herring storage bodies

Weibel-Pallade bodies

Pseudofollicles

Неправильный ответ на вопрос


Баллов: 0 из 10

Правильный ответ:
Brain sand
Сообщить об ошибке (0)

11

Stromal compartment of some organ is composed of branch-shaped epithelial cells that form tri-dimensional meshwork.
What is this organ?

Thymus gland

Red bone marrow

Spleen

Lymph node

Tonsil

Неправильный ответ на вопрос


Баллов: 0 из 10

Правильный ответ:
Thymus gland

Сообщить об ошибке (0)

12

A 10-14 % of newborn suffer from developmental anomalies. What embryonic origin do follicular cells of thyroid gland
develop from?

Endoderm

Ectoderm

Mesenchyme

Intermediate mesoderm

Splanchnic mesoderm

Неправильный ответ на вопрос


Баллов: 0 из 10

Правильный ответ:
Endoderm

Сообщить об ошибке (0)


13

Myocardiodistrophy promotes a breach of cardiac muscle cells metabolism.


Which of the following embryonic origin breach causes this pathology?

Myoepicardial plate

Myotome

Endoderm

Ectoderm

Mesenchyme

Неправильный ответ на вопрос


Баллов: 0 из 10

Правильный ответ:

Myoepicardial plate

Сообщить об ошибке (0)

14

The specimens present sections of hemopoietic and immunogenetic organs. Organ consists of lymphoid tissue forming
different structures (lobules, cortex, medulla ). In what organ does antigen-independent, proliferation and differentiation
take place.

Thymus

Lymph nodes

Spleen

Tonsil

Hemolymph nodes

Неправильный ответ на вопрос


Баллов: 0 из 10

Правильный ответ:
Thymus

Сообщить об ошибке (0)


15

In the adenohypophysis, cells of irregular or angular shape are found, they have lobed nuclei and a well-defined
endoplasmic reticulum. The secretory granules are surrounded by a membrane and contain a dense protein core. The
hormone synthesized by them stimulates the secretory activity of the cells of zona fasciculata cells of the adrenal cortex.
Name these cells.

Corticotropocytes

Thyrotropocytes

Gonadotropocytes

Lactotropocytes

Somatopropocytes

Неправильный ответ на вопрос


Баллов: 0 из 10

Правильный ответ:

Corticotropocytes

Сообщить об ошибке (0)


16

Which layer of the epidermis cells are shown in the electron micrograph

Stratum basale

Stratum spinosum

Stratum granulesum

Stratum lucidum

Stratum corneum

Неправильный ответ на вопрос


Баллов: 0 из 10

Правильный ответ:
Stratum spinosum

Сообщить об ошибке (0)


17

The patient complains of hearing loss after antibiotic therapy with aminoglycosides. Which receptor cells are affected?

hair sensory epithelial cells of Corti's organ

hair sensory epithelial cells of the macula of the sacculus and utriculus

phalangeal cells of Corti's organ

pillar cells

cells of the stria vascularis

Неправильный ответ на вопрос


Баллов: 0 из 10

Правильный ответ:

hair sensory epithelial cells of Corti's organ

Сообщить об ошибке (0)

18

On the scanning electron micrograph (magnification 250x) you can see the apical surface of the hair cells. Name these
cells.

internal hair cells of Corti's organ

outer hair cells of Corti's organ

receptor cells of macula of the sacculus

receptor cells of cristae of the ampullae in the semicircular canals

receptor cells of macula of the utricilus

Неправильный ответ на вопрос


Баллов: 0 из 10

Правильный ответ:

outer hair cells of Corti's organ

Сообщить об ошибке (0)


19

What forms the superior medial wall of the cochlear canal?

vestibular membrane

basillary plate

otolith membrane

stria vascularis

tectorial membrane

Неправильный ответ на вопрос


Баллов: 0 из 10

Правильный ответ:

vestibular membrane

Сообщить об ошибке (0)

20

Name gel-like mass,which is mostly the protein thyroglobulin (pink) and bound thyroid hormones (triiodothyronine and
thyroxin) and fills every follicle.

Gel mass

Colloid

Follicular fluid

Hormonal mass

Thyroid fluid

Неправильный ответ на вопрос


Баллов: 0 из 10

Правильный ответ:

Colloid

Сообщить об ошибке (0)

21

What substance fills the scala vestibulae?

perilymph

endolymph

air

tissue fluid

blood

Правильный ответ на вопрос


Баллов: 10 из 10

Сообщить об ошибке (0)


22

At a vulgar pemphigus in a skin epidermis forms


vesicles, which breaks out a
wholeness of stratum spinosum and stratum
granulosum. What function of an
epithelium is broken first of all?

Barrier

Secretory

Regenerative

Absorbtive

Endocrine

Неправильный ответ на вопрос


Баллов: 0 из 10

Правильный ответ:
Barrier

Сообщить об ошибке (0)

23

What hormone is secreted by secretory cardiomyocytes, which are localized in the atrium?

Atrial natriuretic polypeptide

vasoactive intestinal peptide

Insulin

Progesteron

Serotonin

Неправильный ответ на вопрос


Баллов: 0 из 10

Правильный ответ:
Atrial natriuretic polypeptide
Сообщить об ошибке (0)

24

Imagine that you are a research histologist. You need to dye the elastic fibers in the aorta. What kind of dye do you
choose?

Hematoxylin

Eosin

Hematoxylin and eosin

Orcein

Picrofucsin

Неправильный ответ на вопрос


Баллов: 0 из 10

Правильный ответ:
Orcein

Сообщить об ошибке (0)


25

In a patient with clinical signs of immunodeficiency the number and functional activity of T and B-lymphocytes are not
changed. Defect with dysfunction of antigen-presentation to the immunocompetent cells was found during investigation on
the molecule level. Defect of what cells is the most probable?

Macrophages

NK-cells

T-lymphocytes

Fibroblasts, T-lymphocytes, B-lymphocytes

0-lymphocytes

Неправильный ответ на вопрос


Баллов: 0 из 10

Правильный ответ:
Macrophages

Сообщить об ошибке (0)

26

Patient had been receiving the great doses of hydrocortisone for a long time. Which of the following cells of adrenal cortex
must be atrophied after the influence of this exogenous hormone?

Cells of zona fasciculata

Cells of zona reticularis

Cells of adrenal medulla

Cells of zona glomerulosa

Cells of capsule

Неправильный ответ на вопрос


Баллов: 0 из 10

Правильный ответ:

Cells of zona fasciculata

Сообщить об ошибке (0)


27

It is the wall of heart on the histologic micropreparation. There are contractile, secretory and conductive cardiac muscle
cells and endomysium with blood vessels. For which of the following layers are these structures belong to?

Myocardium of atria

Pericardium

Endocardium of ventricles

Tunica adventitia

Epicardium

Неправильный ответ на вопрос


Баллов: 0 из 10

Правильный ответ:
Myocardium of atria

Сообщить об ошибке (0)

28

Studying the tissue of the parathyroid gland, the researcher saw 2 types of cells: major and oxyphilic. Which organelle is
prevalent in oxyphilic cells?

Smooth endoplasmic reticulum

Rough endoplasmic reticulum

Mitochondria

Ribosomes

Lysosomes

Неправильный ответ на вопрос


Баллов: 0 из 10

Правильный ответ:

Mitochondria

Сообщить об ошибке (0)

29

At a patient with AIDS supression of B-cells differentiation is revealed. In what organ antigen independent differentiation
and proliferation of B-cells takes place?

Red bone marrow

Lymph node

Tonsil

Thymus

Spleen

Правильный ответ на вопрос


Баллов: 10 из 10

Сообщить об ошибке (0)


30

It is determined the large, oval and round cells with light cytoplasm and a small number of organelles and myofibrils on
histologic micropreparation of the heart wall. What are these cells?

Purkinje fibers

P- cells

Contractile cardiac muscle

Secretory cardiac muscle

Smooth muscle

Неправильный ответ на вопрос


Баллов: 0 из 10

Правильный ответ:
Purkinje fibers

Сообщить об ошибке (0)

31

Which layer of epidermis corresponds to the following cell`s description:


Several layers of large keratinocytes that are attached each other by desnosomes and contain keratin filaments

Stratum basale

Stratum spinosum

Stratum granulesum

Stratum lucidum

Stratum corneum

Неправильный ответ на вопрос


Баллов: 0 из 10

Правильный ответ:
Stratum spinosum

Сообщить об ошибке (0)

32

What is the embryonic origin of Langerhans cells?

Ectoderm

Neural tube

Neural crest

Mesenchyme

Endoderm

Неправильный ответ на вопрос


Баллов: 0 из 10

Правильный ответ:
Mesenchyme
Сообщить об ошибке (0)

33

At the patient who during 7 years suffers a hypothyroidism, failure of thyroid hormones is detected. What cells of an
Adenohypophysis thus will be modified?

Thyrotropes

Gonadotropes

Mammotropes

Corticotropes

Somatotropes

Неправильный ответ на вопрос


Баллов: 0 из 10

Правильный ответ:

Thyrotropes

Сообщить об ошибке (0)

34

Characterizing a stress, the student has supposed an inaccuracy when has told, that synthesis of glucocorticoids of a
suprarenal cortex is stimulated by hormones of a pituitary gland. What clarification is necessary?

Аdrenocorticotropic hormon (ACTH)

Somatotropin

Prolactin

Gonadotropic hormones

Тhyrotropic hormone

Неправильный ответ на вопрос


Баллов: 0 из 10

Правильный ответ:
Аdrenocorticotropic hormon (ACTH)

Сообщить об ошибке (0)


35

A 10-14 % of newborn suffer from developmental anomalies. What embryonic origin do cells of parthyroid gland develop
from?

Endoderm

Ectoderm

Mesenchyme

Coelomic ntermediate mesoderm

Splanchnic mesoderm

Неправильный ответ на вопрос


Баллов: 0 из 10

Правильный ответ:

Endoderm

Сообщить об ошибке (0)

36

Which layer of epidermis corresponds to the following cell`s description:


Flattened keratinocytes filled with keratogyalin granules and lamellar bodies.

Stratum basale

Stratum spinosum

Stratum granulesum

Stratum lucidum

Stratum corneum

Правильный ответ на вопрос


Баллов: 10 из 10

Сообщить об ошибке (0)

37

Which of the following is an embryonic origin of vessel wall?

Mesenchyme

Ectoderm

Endoderm

Mesoderm

Neural tube

Неправильный ответ на вопрос


Баллов: 0 из 10

Правильный ответ:
Mesenchyme

Сообщить об ошибке (0)


38

In the adenohypophysis, basophilic endocrinocytes of irregular or angular shape with small secretory granules 80-150 nm
in diameter are found. The hormone they produce stimulates the function of the thyroid follicular endocrinocytes. Name
these cells.

Corticotropocytes

Thyrotropocytes

Gonadotropocytes

Lactotropocytes

Somatopropocytes

Неправильный ответ на вопрос


Баллов: 0 из 10

Правильный ответ:
Thyrotropocytes

Сообщить об ошибке (0)

39

At a person genetic disorder of metabolism of amino acid tyrosin broke synthesis of a black pigment. In what cells of loose
connective tissue and a skin epidermis lack of this pigment will be observed?

Melanocytes

Mast cells

Dendritic cells

Adipose cells

Pericytes

Неправильный ответ на вопрос


Баллов: 0 из 10

Правильный ответ:

Melanocytes

Сообщить об ошибке (0)


40

Determine the histological specimen

Thyroid gland

Parathyroid gland

Hypophysis

Adrenal gland cortex

Adrenal gland medulla

Правильный ответ на вопрос


Баллов: 10 из 10

Сообщить об ошибке (0)


41

The red bone marrow is shown in the photograph of the micropreparation. What number corresponds to the bone beam

Неправильный ответ на вопрос


Баллов: 0 из 10

Правильный ответ:
5

Сообщить об ошибке (0)

42

It was determined the anastomosing in cords one of the layers of cardiovascular system organ. These cords consist of
cells with junction specializations making up the intercalated disk. What tissue forms this layer?

Striated cardiac

Dense connective

Striated skeletal

Smooth muscle

Loose connective

Неправильный ответ на вопрос


Баллов: 0 из 10

Правильный ответ:
Striated cardiac

Сообщить об ошибке (0)


43

Describe the sweat gland:

Simple branched alveolar

Complex branched alveolar

Simple unbranched tubular

Simple unbranched alveolar

Complex branched tubular

Правильный ответ на вопрос


Баллов: 10 из 10

Сообщить об ошибке (0)

44

Which layer of epidermis corresponds to the following cell`s description:


It is clear translucent layer, present only in thick skin. Cells are not distinguishable clearly, flattened and closely packed.
Nuclei and organelles are absent, cytoplasm contains granules of eleidin, wich are responsible for acidophilia and
rectractility.

Stratum basale

Stratum spinosum

Stratum granulesum

Stratum lucidum

Stratum corneum

Неправильный ответ на вопрос


Баллов: 0 из 10

Правильный ответ:
Stratum lucidum

Сообщить об ошибке (0)

45

It was determined, that the thyroid follicles are enlarged, owing to increases of follicular epithelial height and increased
thyroglobulin deposits on the micropreparation of the thyroid gland. Which functional state is the following histological
picture characteristic of?

Age changes

Hypo function

Moderate activity

Hyper function

Rise of iodine level the blood plasma

Неправильный ответ на вопрос


Баллов: 0 из 10

Правильный ответ:

Hyper function
Сообщить об ошибке (0)

46

The spleen microslide is shown in the photograph. What is the number of the central artery?

Правильный ответ на вопрос


Баллов: 10 из 10

Сообщить об ошибке (0)

47

Under influence of unfavorable factors on the organism fast reconstruction of a thymus gland took place. It accompanied
by the massive death of T-cells, its migration into peripheral lymphoid organs and by the proliferation of epithelial reticular
stromal cells. What is the name of that reaction?

Accidental involution

Age involution

Thymic atrophy

Thymic hypotrophy

Еhymus necrosis

Неправильный ответ на вопрос


Баллов: 0 из 10

Правильный ответ:

Accidental involution

Сообщить об ошибке (0)


48

There are acidophilic endocrinocytes with globular granules 350-400 nm in diameter In the adenohypophysis , secreting a
hormone that regulates the growth of the body. Name these cells.

Corticotropocytes

Thyrotropocytes

Gonadotropocytes

Lactotropocytes

Somatopropocytes

Неправильный ответ на вопрос


Баллов: 0 из 10

Правильный ответ:

Somatopropocytes

Сообщить об ошибке (0)

49
Which number in the picture does the inner elastic membrane correspond to?

Правильный ответ на вопрос


Баллов: 20 из 20

Сообщить об ошибке (0)


50

Determine the histological specimen

Thyroid gland

Parathyroid gland

Hypophysis

Adrenal gland cortex

Adrenal gland medulla

Неправильный ответ на вопрос


Баллов: 0 из 10

Правильный ответ:

Adrenal gland cortex

Сообщить об ошибке (0)

Не знаете как сохранить результат? Отправьте его на email

Р Е КЛ А М А

Р Е КЛ А М А

Комментарии

Пока нет комментариев ... Добавить


Яндекс.Директ

Хотите от‐ Отель Арчи


крыть свою на Тульской
онлайн
школу?
marketinator.ru hotelarchi.ru
16+
KP

Результат #72120481

Дата завершения: 22.10.2020 14:11


Потрачено времени: 00:04:49

Ваше имя: Введите ваше имя

Ответы Результат

At the woman 30 years old have revealed a shortage of sexual hormones, have found an increase level of follicle
stimulating hormone. What cells of an Adenohypophysis will be modified thus?

Somatotropes

Thyrotropes

Corticotropes

Gonadotropes

Mammotropes

Неправильный ответ на вопрос


Баллов: 0 из 10

Правильный ответ:
Gonadotropes

Сообщить об ошибке (0)

What is the embryonic origin of Merkel`s cells?

Neural crest

Ectoderm

Neural tube

Mesenchyme

Endoderm

Неправильный ответ на вопрос


Баллов: 0 из 10

Правильный ответ:

Neural crest

Сообщить об ошибке (0)


3

Some organ is presented in histological section. It has cortex and medulla. Cortex is composed of outer cortex with
lymphoid nodules and paracortex. Medulla contains medullary cords, sinuses and trabecules. What organ is it?

Thymus

Spleen

Adrenal gland

Kidney

Lymph node

Неправильный ответ на вопрос


Баллов: 0 из 10

Правильный ответ:
Lymph node

Сообщить об ошибке (0)

A 2-year- old child experienced convulsions because of lowering calcium ions concentration in the blood plasma. Function
of what structure is decreased?

Adrenal medulla

Parathyroid glands

Adrenal cortex

Thyroid gland

Hipophysis

Неправильный ответ на вопрос


Баллов: 0 из 10

Правильный ответ:

Parathyroid glands

Сообщить об ошибке (0)

Which of the following hormones increases the deposition of calcium in the bones, reduces the reabsorption of calcium in
the renal tubules, reduces the absorption of calcium in the intestine and thus reduces the level of calcium in the blood?

Glucocorticoidhormones

Calcitonin

Tetraiodthyronine

Parathormone

Mineralocorticoid hormones

Неправильный ответ на вопрос


Баллов: 0 из 10

Правильный ответ:
Calcitonin
Сообщить об ошибке (0)

The removing of an endocrine gland of experimental animals occurs a premature puberty. Which of the following gland is
removing can occur a premature puberty?

Adrenal gland

A pituitary gland

A pineal gland

A thyroid gland

Parathyroid gland

Правильный ответ на вопрос


Баллов: 10 из 10

Сообщить об ошибке (0)

The spiral ganglion is affected by a chronic inflammatory process. What functional changes will be noted in the animal?

perception of vibration is impaired

perception of sounds is impaired

perception of angular accelerations is impaired

perception of gravity is impaired

perception of linear accelerations is impaired

Правильный ответ на вопрос


Баллов: 10 из 10

Сообщить об ошибке (0)

The kinocilia and stereocilia of the hair cells of cristae ampullares of the semicircular canals are immersed in …

hair membrane

otolithic membrane

cupulae gelatinosa

tectorial membrane

the auditory membrane

Неправильный ответ на вопрос


Баллов: 0 из 10

Правильный ответ:
cupulae gelatinosa

Сообщить об ошибке (0)


9

Which layer of the epidermis cells are shown in the electron micrograph

Stratum corneum

Stratum spinosum

Stratum lucidum

Stratum basale

Stratum granulesum

Неправильный ответ на вопрос


Баллов: 0 из 10

Правильный ответ:
Stratum spinosum

Сообщить об ошибке (0)


10

What the cytoplasm of Langergans cells contains?

Pacininian bodies

Keratohyalin granules

Lamellar bodies

Melanosomes

Birbeck`s granules

Неправильный ответ на вопрос


Баллов: 0 из 10

Правильный ответ:
Birbeck`s granules

Сообщить об ошибке (0)

11

Determine the histological specimen

Hypophysis

Thyroid gland

Adrenal gland medulla

Parathyroid gland

Adrenal gland cortex

Неправильный ответ на вопрос


Баллов: 0 из 10

Правильный ответ:
Adrenal gland cortex

Сообщить об ошибке (0)


12

Name gel-like mass,which is mostly the protein thyroglobulin (pink) and bound thyroid hormones (triiodothyronine and
thyroxin) and fills every follicle.

Colloid

Hormonal mass

Gel mass

Follicular fluid

Thyroid fluid

Неправильный ответ на вопрос


Баллов: 0 из 10

Правильный ответ:
Colloid

Сообщить об ошибке (0)

13

It was determined, that the thyroid follicles are enlarged, owing to increases of follicular epithelial height and increased
thyroglobulin deposits on the micropreparation of the thyroid gland. Which functional state is the following histological
picture characteristic of?

Age changes

Hyper function

Moderate activity

Hypo function

Rise of iodine level the blood plasma

Правильный ответ на вопрос


Баллов: 10 из 10

Сообщить об ошибке (0)

14

At the patient who during 7 years suffers a hypothyreoidism, failure of thyroid hormones is detected. Using your
knowledge of the feedback mechanism, indicate how the serum thyroid-stimulating hormone level will change in this
patient

will be reduced

will be increased

without change

Правильный ответ на вопрос


Баллов: 10 из 10

Сообщить об ошибке (0)


15

Stromal compartment of some organ is composed of branch-shaped epithelial cells that form tri-dimensional meshwork.
What is this organ?

Lymph node

Spleen

Tonsil

Thymus gland

Red bone marrow

Неправильный ответ на вопрос


Баллов: 0 из 10

Правильный ответ:
Thymus gland

Сообщить об ошибке (0)

16

In the adenohypophysis, cells of irregular or angular shape are found, they have lobed nuclei and a well-defined
endoplasmic reticulum. The secretory granules are surrounded by a membrane and contain a dense protein core. The
hormone synthesized by them stimulates the secretory activity of the cells of zona fasciculata cells of the adrenal cortex.
Name these cells.

Lactotropocytes

Gonadotropocytes

Thyrotropocytes

Corticotropocytes

Somatopropocytes

Неправильный ответ на вопрос


Баллов: 0 из 10

Правильный ответ:

Corticotropocytes

Сообщить об ошибке (0)


17

It is known, that plasma cells produce specific antibodies on the given antigen. At injection of an antigen the amount of
plasmocytes increases. Due to what blood cells the augmentation of number of plasmocytes is carried out?

Eosinophils

Neutrophils

Basophils

B-cells

T lymphocytes

Неправильный ответ на вопрос


Баллов: 0 из 10

Правильный ответ:

B-cells

Сообщить об ошибке (0)

18

What constitutes the inner root sheath?

Cuticle, Henley`s layer, Haxley`s layer

Cuticle, cortex, medulla

Catagen, anagen, telogen

Cuticle, connective tissue layers

Cuticle, glassy membrane,

Неправильный ответ на вопрос


Баллов: 0 из 10

Правильный ответ:

Cuticle, Henley`s layer, Haxley`s layer

Сообщить об ошибке (0)


19

Hematopoietic and stromal cells of the red bone marrow are shown in the figure. What number corresponds to a
megakaryocyte

Неправильный ответ на вопрос


Баллов: 0 из 10

Правильный ответ:
2

Сообщить об ошибке (0)


20

A 10-14 % of newborn suffer from developmental anomalies. What embryonic origin do follicular cells of thyroid gland
develop from?

Ectoderm

Mesenchyme

Intermediate mesoderm

Endoderm

Splanchnic mesoderm

Неправильный ответ на вопрос


Баллов: 0 из 10

Правильный ответ:

Endoderm

Сообщить об ошибке (0)

21

In a patient after transplantation of the foreign kidney rejection of the transplanted organ developed. What main effecter
cells participate in this reaction at first?

Plasma cells

T-killers (T-cytotoxic)

B-cells

T-lymphocytes –suppressors

Fibroblasts

Неправильный ответ на вопрос


Баллов: 0 из 10

Правильный ответ:
T-killers (T-cytotoxic)

Сообщить об ошибке (0)

22

At a patient with AIDS supression of B-cells differentiation is revealed. In what organ antigen independent differentiation
and proliferation of B-cells takes place?

Tonsil

Thymus

Red bone marrow

Lymph node

Spleen

Правильный ответ на вопрос


Баллов: 10 из 10

Сообщить об ошибке (0)


23

Myocardiodistrophy promotes a breach of cardiac muscle cells metabolism.


Which of the following embryonic origin breach causes this pathology?

Mesenchyme

Myoepicardial plate

Myotome

Endoderm

Ectoderm

Правильный ответ на вопрос


Баллов: 10 из 10

Сообщить об ошибке (0)

24

Patient had been receiving the great doses of hydrocortisone for a long time. Which of the following cells of adrenal cortex
must be atrophied after the influence of this exogenous hormone?

Cells of capsule

Cells of zona fasciculata

Cells of zona reticularis

Cells of adrenal medulla

Cells of zona glomerulosa

Правильный ответ на вопрос


Баллов: 10 из 10

Сообщить об ошибке (0)

25
Which number in the picture does the inner elastic membrane correspond to?

Неправильный ответ на вопрос


Баллов: 0 из 20

Правильный ответ:
2

Сообщить об ошибке (0)


26

It is determined the blood vessel on the histological micropreparation of the spleen.Its wall consists of tunica intima
(endothelium, basal membrane), lack
of the distinct tunica media, tunica adventitia (contains collagen and elastic fibers, that anchor the vessel in the
surrounding connective tissue trabeculae.
What type of vessel is it?

Muscular artery

Fibrous vein

Arteriole

Muscular vein

Elastic artery

Неправильный ответ на вопрос


Баллов: 0 из 10

Правильный ответ:
Fibrous vein

Сообщить об ошибке (0)

27

It was determined the anastomosing in cords one of the layers of cardiovascular system organ. These cords consist of
cells with junction specializations making up the intercalated disk. What tissue forms this layer?

Smooth muscle

Dense connective

Striated skeletal

Striated cardiac

Loose connective

Неправильный ответ на вопрос


Баллов: 0 из 10

Правильный ответ:
Striated cardiac

Сообщить об ошибке (0)


28

It is known, that aldosterone controls water and electrolyte balance in the organism. Which of the following cell types of
adrenal gland produce this hormone?

Cells of Zona fasciculata

Cells of Zona glomerulosa

Norepinephrine-secreting cells

Epinephrine- secreting cells

Cells of Zona reticularis

Неправильный ответ на вопрос


Баллов: 0 из 10

Правильный ответ:
Cells of Zona glomerulosa

Сообщить об ошибке (0)

29

They are determined cardiac muscle cells with glycoprotein rich granules. What are these cells?

P-cells

Purkinje cells

Transitional cells

Secretory

Contractile

Неправильный ответ на вопрос


Баллов: 0 из 10

Правильный ответ:
Secretory

Сообщить об ошибке (0)

30

In experiment the significant amount of stem cells of bone marrow definitely eroded. Regeneration of what population of
cells in composition of loose connective tissue will be inhibited.

Adipose cells

Pigment cells

Fibroblasts

Pericytes

Macrophages

Неправильный ответ на вопрос


Баллов: 0 из 10

Правильный ответ:
Macrophages

Сообщить об ошибке (0)


31

In the adenohypophysis, basophilic endocrinocytes of irregular or angular shape with small secretory granules 80-150 nm
in diameter are found. The hormone they produce stimulates the function of the thyroid follicular endocrinocytes. Name
these cells.

Lactotropocytes

Gonadotropocytes

Corticotropocytes

Somatopropocytes

Thyrotropocytes

Неправильный ответ на вопрос


Баллов: 0 из 10

Правильный ответ:

Thyrotropocytes

Сообщить об ошибке (0)

32

The operation of thyroid gland sectoral ectomy caused spastic muscular contraction of a 30- year old woman. It was
determined the low blood calcium. Lack of what hormone secretion disrupts neuromuscular function?

Antidiuretic hormone

Melatonine

Somatotropin

Thyroxin

Parathyroid hormone

Неправильный ответ на вопрос


Баллов: 0 из 10

Правильный ответ:
Parathyroid hormone

Сообщить об ошибке (0)


33

What is the embryonic origin of melanocytes?

Neural tube

Ectoderm

Neural crest

Endoderm

Mesenchyme

Неправильный ответ на вопрос


Баллов: 0 из 10

Правильный ответ:
Neural crest

Сообщить об ошибке (0)

34

At survey of the patient the poor amount of immunoglobulins has been detected. What from the enumerated cells of an
immune system produce immunoglobulins?

Plasmablasts

T-helper cells

Т-killers cells

Plasma cells (plasmocytes)

B-cells

Правильный ответ на вопрос


Баллов: 10 из 10

Сообщить об ошибке (0)

35

Characterizing a stress, the student has supposed an inaccuracy when has told, that synthesis of glucocorticoids of a
suprarenal cortex is stimulated by hormones of a pituitary gland. What clarification is necessary?

Аdrenocorticotropic hormon (ACTH)

Тhyrotropic hormone

Gonadotropic hormones

Somatotropin

Prolactin

Неправильный ответ на вопрос


Баллов: 0 из 10

Правильный ответ:
Аdrenocorticotropic hormon (ACTH)

Сообщить об ошибке (0)


36

Characterizing a stress, the student has supposed an inaccuracy when has told, that synthesis of glucocorticoids of a
suprarenal cortex is stimulated by hormones of a pituitary gland. What clarification is necessary?

Somatotropin

Prolactin

Gonadotropic hormones

Аdrenocorticotropic hormon (ACTH)

Тhyrotropic hormone

Неправильный ответ на вопрос


Баллов: 0 из 10

Правильный ответ:
Аdrenocorticotropic hormon (ACTH)

Сообщить об ошибке (0)

37

In the adenohypophysis, rounded basophilic endocrinocytes with an eccentric arrangement of nuclei and secretory
granules of 200-300 nm, containing follitropin and lutropin, are found. Name these cells.

Thyrotropocytes

Lactotropocytes

Corticotropocytes

Somatopropocytes

Gonadotropocytes

Неправильный ответ на вопрос


Баллов: 0 из 10

Правильный ответ:
Gonadotropocytes

Сообщить об ошибке (0)


38

At the patient who during 7 years suffers a hypothyroidism, failure of thyroid hormones is detected. What cells of an
Adenohypophysis thus will be modified?

Somatotropes

Thyrotropes

Mammotropes

Gonadotropes

Corticotropes

Неправильный ответ на вопрос


Баллов: 0 из 10

Правильный ответ:

Thyrotropes

Сообщить об ошибке (0)

39

There are acidophilic endocrinocytes with globular granules 350-400 nm in diameter In the adenohypophysis , secreting a
hormone that regulates the growth of the body. Name these cells.

Lactotropocytes

Gonadotropocytes

Corticotropocytes

Thyrotropocytes

Somatopropocytes

Неправильный ответ на вопрос


Баллов: 0 из 10

Правильный ответ:
Somatopropocytes

Сообщить об ошибке (0)


40

What forms the superior medial wall of the cochlear canal?

tectorial membrane

vestibular membrane

basillary plate

otolith membrane

stria vascularis

Правильный ответ на вопрос


Баллов: 10 из 10

Сообщить об ошибке (0)

41

At a person genetic disorder of metabolism of amino acid tyrosin broke synthesis of a black pigment. In what cells of loose
connective tissue and a skin epidermis lack of this pigment will be observed?

Dendritic cells

Melanocytes

Pericytes

Adipose cells

Mast cells

Правильный ответ на вопрос


Баллов: 10 из 10

Сообщить об ошибке (0)

42

What hormone is secreted by secretory cardiomyocytes, which are localized in the atrium?

Progesteron

Serotonin

Atrial natriuretic polypeptide

vasoactive intestinal peptide

Insulin

Правильный ответ на вопрос


Баллов: 10 из 10
Сообщить об ошибке (0)

43

Usage of oral contraceptives with sex hormones inhibits secretion of the hypophyseal hormones. Secretion of which of the
indicated hormones is inhibited while using oral contraceptives with sex hormones.

Follicle-stimulating

Thyrotropic

Somatotropic

Vasopressin

Oxytocin

Правильный ответ на вопрос


Баллов: 10 из 10

Сообщить об ошибке (0)

44

What substance fills the scala vestibulae?

perilymph

blood

air

endolymph

tissue fluid

Неправильный ответ на вопрос


Баллов: 0 из 10

Правильный ответ:
perilymph

Сообщить об ошибке (0)

45

Describe the sebaceous gland:

Complex branched tubular

Simple unbranched alveolar

Simple unbranched tubular

Simple branched alveolar

Complex branched alveolar

Неправильный ответ на вопрос


Баллов: 0 из 10

Правильный ответ:
Simple branched alveolar

Сообщить об ошибке (0)


46

Patient with Cushing syndrome (high level of glucocorticoids in the blood) has the high blood sugar. Which of the
following organ pathology is the cause of this disease?

Pancreas

Adrenal cortex

Adrenal medulla

Thyroid gland

Pineal gland

Неправильный ответ на вопрос


Баллов: 0 из 10

Правильный ответ:
Adrenal cortex

Сообщить об ошибке (0)

47

At a vulgar pemphigus in a skin epidermis forms


vesicles, which breaks out a
wholeness of stratum spinosum and stratum
granulosum. What function of an
epithelium is broken first of all?

Endocrine

Regenerative

Secretory

Barrier

Absorbtive

Неправильный ответ на вопрос


Баллов: 0 из 10

Правильный ответ:

Barrier

Сообщить об ошибке (0)


48

For the treatment of osteoporosis in the elderly, hormone therapy is used. What hormone enhances calcium deposition in
bones?

Parathormone

Calcitonin

Tetraiodthyronine

Mineralocorticoid hormones

Epinephrine

Правильный ответ на вопрос


Баллов: 10 из 10

Сообщить об ошибке (0)

49

The specimens present sections of hemopoietic and immunogenetic organs. Organ consists of lymphoid tissue forming
different structures (lobules, cortex, medulla ). In what organ does antigen-independent, proliferation and differentiation
take place.

Lymph nodes

Hemolymph nodes

Spleen

Thymus

Tonsil

Неправильный ответ на вопрос


Баллов: 0 из 10

Правильный ответ:
Thymus

Сообщить об ошибке (0)


50

The spleen microslide is shown in the photograph. What is the number of the central artery?

Правильный ответ на вопрос


Баллов: 10 из 10

Сообщить об ошибке (0)

Не знаете как сохранить результат? Отправьте его на email

Have A Bright Future


with MAAC
MAAC

Join MAAC for best courses in animation,


VFX, gaming, graphic designing & multimeida

OPEN

Комментарии

Пока нет комментариев ... Добавить


Яндекс.Директ

Такси Внуко‐ Станьте тре‐


во Сочи нером по Гим‐
настике лица!

logitaxi.ru beauty-academy-of-js.ru
Эмбриология 3. Inactivation of homologous
[1] chromosomes
The child inherits an antigen of 4. Inactivation of one Y chromosome
histocompatibility from the father and mother. It 5. Inactivation of one chromatid
is known, that the expression of parent antigens
in an embryogenesis begins very early. But the [6]
immune system of mother does not tear away a When does a secondary oocyte complete its
fetus. What fetal membrane for the first time second meiotic division to become a mature
hinders fetus from mother’s organism? ovum?
1.Chorion 1. at ovulation
2.Cytotrophoblast 2. before ovulation
3.Аllantois 3. at fertilization
4.Yolk sack 4. puberty
5.Placenta 5. before birth

[2] [7]
Producing series of hormones, the placenta How soon after fertilization occurs within the
plays a role of a temporal endocrine gland. What uterine tube does the blastocyst begin
hormone can be specific in a blood of the implantation?
woman for 3-4 day after the beginning of an 1. by day
implantation, which is used, in medical practice 2. by 12 hours
for early diagnosing pregnancy? 3. by day 1
1.Chorionic gonadotropin 4. by day 2
2.Somatostatin 5. 7within minutes
3.Progesterone
4.Vasopressin [8]
5.Oxytocin Where does the blastocyst normally implant?
1. functional layer of the endometrium
[3] 2. functional layer of the cervix
The germ of an organ, which has endocrine 3. basal layer of the endometrium
function, is formed from a trophoblast during 4. myometrium
embryogenesis. Name this organ. 5. perimetrium
1.Villous chorion
2.Yolk sack [9]
3.Amnion All of the following structures are necessary for
4.Аllantois blastocyts implantation EXCEPT?
5.Umbilical cord 1. zona pellucida
2. endometrium in progestational phase
[4] 3. syncytiotrophoblast
It is known, that some microorganisms which 4. cytotrophoblast
cause infectious diseases, can transit through a 5. functional layer of the endometrium
placental barrier. What the structure designated
below, enter its composition from? [10]
1.All structural ingredients of the tertiary villi Which of the following components plays the
2.All structural ingredients of the secondary villi most active role in invading the endometrium
3.All structural ingredients of the primary villi during blastocyst implantation?
4.Yolk sack and allantois 1. syncytiotrophoblast
5.Chorion and an amnion 2. epiblast
3. hypoblast
[5] 4. extraembryonic somatic mesoderm
Fetal sex can be diagnosed by noting the 5. extraembryonic visceral mesoderm
presence or absence of the Barr body in cells
obtained from the amniotic fluid. What is the [11]
etiology of the Barr body? Between which two layers is the
1. Inactivation of one X chromosome extraembryonic mesoderm located?
2. Inactivation of both X chromosomes 1. exocoelomic membrane and
cytotrophoblast
2. syncytiotrophoblast and cytotrophoblast 5. notochord
3. syncytiotrophoblast and endometrium
4. exocoelomic membrane and [18]
syncytiotrophoblast The developing embryo has a distinct human
5. epiblast and hypoblast appearance by the end of
1. week 8
[12] 2. week 5
During the second week of development, the 3. week 6
embryoblast receives its nutrients via 4. week 7
1. diffusion 5. week 4
2. osmosis
3. reverse osmosis [19]
4. fetal capillaries The lateral mesoderm is divided into two
5. yolk sac nourishment distinct layers by the formation of the
1. intraembrionic coelom
[13] 2. extraembrionic coelom
Which germ layers are present at the end of 3. cardiogenic region
week 3 of development (day 21)? 4. notochord
1. ectoderm, mesoderm, and endoderm 5. yolk sac
2. epiblast and hypoblast
3. ectoderm and endoderm [20]
4. epiblast only In pregnant woman on the second day after a
5. epiblast, mesoderm, and hypoblast fertilization as a result of cleavage had apostates
of light and dark blastomeres. What possible
[14] result of such variant of cleavage will be?
Which process establishes the three definitive 1. A birth of monozygotic twins
germ layers? 2. The arrest of an embryogenesis
1. gastrulation 3. A birth of the boy
2. neurulation 4. A birth of Siamese twins
3. craniocaudal folding 5. A birth of the girl
4. lateral folding
5. angiogenesis [21]
In embryo coated with a fertilization membrane,
[15] synthesis of lytic enzymes of a trophoblast
The first indication of gastrulation in the genetical inhibited. What process of an
embryo is embryogenesis can be late failed?
1. formation of the primitive streak 1. Implantation
2. formation of the notochord 2. Gastrulation
3. formation of the neural tube 3. Delamination
4. formation of extraembryonic mesoderm 4. Epiboly
5. formation of tertiary chorionic villi 5. Immigration

[16] [22]
Somites may differentiate into all of the On microslide of human embryo taken at a
following EXCEPT consensual abortion, it is detected primitive
1. epidermis of skin groove in which two layers of cells range:
2. cartilage ectoderm and endoderm. At what stage of
3. muscle embryonic development there was an embryo?
4. bone 1. Gastrulations
5. dermis of skin 2. Neurulations
3. Organofaction
[17] 4. A progenesis
Intermediate mesoderm will give rise to the 5. A histogenesis
1. kidneys and gonads
2. heart [23]
3. neural tube As a result of blocking separate ingredients of a
4. somites genome, a cell gain characteristic for them
morphological, biochemical and function 1.The fourth through eighth week of fetal life
singularities. What title has this process? 2.The third month of fetal life
1. Differentiation 3.Fertilization to one week of fetal life
2. Capacitation 4.The second and third week of fetal life
3. Reception 5.The third trimester of fetal life
4. Adhesion
5. Determination [29]
A 10-14 % of newborn suffer from
[24] developmental anomalies. Which embryonic
Breaking architectonics in a somite region near origin urinary system develops?
to an endoderm and notochord is found during 1.Intermediate mesoderm
biopsy examination of embryonic material. 2.Dorsal mesoderm
Disorders of what tissues or organs will be? 3.Visceral layer of ventral mesoderm
1. Skeletal tissues 4.Parietal layer of ventral mesoderm
2. Striated skeletal muscle 5.Mesenchyme
3. Urogenital system
4. Striated cardiac muscle [30]
5. Fibrillar connective tissue of a skin Slow healing of an umbilicus is caused by
patent primitive urinary duct - urachus. Which
[25] embryonic origin are urinary bladder and
Breaking of differentiation of an endoderm is urachus develop from?
detected in embryonic material. Modification of 1. Allantois
development of what organ can arise at the 2. Cloaca
given process? 3.Urogenital sinus
1. Stomach 4.Paramesonephric duct
2. Hearts 5.Mesonephric duct
3. Kidney
4. Aorta [31]
5. Salivary gland At early stages of embryos development appear
a finger-like diverticulum of a yolk sac which
[26] grows in connecting stalk. What name has this
The gastrulation or formation of germinal layers fetal membrane?
of an embryo descends different ways. What is 1.Allantois
the way of human ectoderm and endoderm 2.Amnion
formation? 3.Placenta
1. Delamination 4.Umbilical cord
2. An invagination 5.Yolk sac
3. Immigration
4. Invagination and epiboly [32]
5. Epiboly On a microslide of a cross section of an organ
that is formed by the mucous connective tissue,
[27] two arteries and a vein is introduced. What is the
During a gastrulation the embryos passes from organ on microslide?
histotrophic to hematrophic type of a feeding. 1. Umbilical cord
What fetal membrane for the first time ensures 2.Yolk sac
it? 3.Amnion
1. A chorion 4.Allantois
2.Yolk sack 5.Placenta
3.Amnion
4. A trophoblast [33]
5.Allantois On microslide of the bulb of the fetus eye a
damage of a cornea is observed. Part of what
[28] germ layer has been struck during embryonic
During which period is the embryo most development
susceptible to environmental influences that 1. Ectoderm
could induce the formation of nonlethal 2.Mesoderm
congenital malformations? 3. Endoderm
4.Dermatome organism happened normally. What
5.Nephrotome morphologic components of the umbilical cord
promoted to this?
[34] 1. Mucuous tissue
During an embryogenesis there was a breaking 2. Allantois residue
shaping of a pharyngeal gut. Specify possible 3.Tunic of umbilical arteries
localization of anomalies of development? 4.Tunic of umbilical vein
1.Organs of an oral cavity. 5.Yolk sac residue
2.A stomach
3.An ileum [40]
4.A pancreas There is connecting with the gut vesicle is
5.A jejunum visible on the histologic micropreparation . It is
an extraembryonic organ. Primordial germ cells
[35] and erythroblasts (megaloblasts) are located in
Women was ill by flu on the 7-14 days of its wall. What organ is this?
pregnancy. In which of the following structures 1. Yolk sac
may be development of pathologic changes? 2. Allantois
1. Breach of ecto- and endordermal formation 3.Placenta
2.Breach of mesenchyme formation 4.Umbilical cord
3. Breach of mesordermal formation 5.Amnion
4.Breach of the process of epibole
5.Breach of the process of invagination [41]
Destruction of neural crest took place in embryo
[36] after bacterial toxins influence. What element
As the result of acrosomal reaction lack the development will be absent in this case?
destruction of junctions between oocyte 1. C-cells, pigment cells, suprarenal medulla
granulosa cells did not occur under fertilization. 2.Myocardium, epicardium
What process was broken? 3.Kidney, ureter
1. Denudation 4. Skin dermis
2. Capacitation 5.Gastric and intestinal epithelium
3.Cortical reaction
4.Zona reaction [42]
5.Determination The prechordal plate is origin of development
of the future epithelium of
[37] 1. esophagus
It was determined compression of the fetus 2.heart
umbilical cord, but it was not destroyed blood 3. umbilical cord
circulation between fetus and mother. Presence 4.anus
of what structures cause this process? 5.nose
1. Wharton's jelly
2.Layer of artery [43]
3.Layer of vein For what purpose can be used the definition
4. Allantois of Barr body?
5.Yolk sac stalk 1.to identify the sex of a fetus
2.to definition a certain genetic diseases
[38] 3.to definition the age of a fetus
A metaphase-blocking dose of colchicines
works by which of the following mechanisms
1.Inhibition of tubulin polymerization
2.Depolymerization of myosin
3.Enhancement of tubulin polymerization
4.Depolymerization of actin
5.Binding to and stabilizing microtubules

[39]
Compression of umbilical cord took place. But
blood circulation between fetus and maternal
TESTING BASIC TISSUES.
1. Tissue. Definition. Classification. Value of works of A.A.Zavarsin and
N.G.Hlopin. in development of study of tissues.
2. Tissue as one of levels of living organisms organization. Definition. Classification
of tissues types. Concept of determination and differentiation of tissues.
3. Concept of differon and sterm cell.
4. Tissue as one of levels of living organisms organization. Definition. Derivates of
cells ( sincytium, symplastos, extracellular matrix).
5. Tissues. Definition. Physiologic and reparative regeneration of different types of
tissues.
EPITHELIAL TISSUES.
6. Epithelial tissues . General characteristic . Morphofunctional and genetic
classification of their types.
7. Epithelial tissues .Morphofunctional and genetic characteristic of different
types of surface epithelium.
8. Glandular epithelium . Classification and structure of glands. Morphology of
secretory cycle. Various types of secretion.
BLOOD AND HEMOPOIESIS.
9.Prenatal hemopoiesis(development of blood as tissue). Peculiarities of
mesoblastic and hepatic hemopoiesis.
10. Postnatal hemopoiesis. Modern scheme of hemopoiesis.
11. Postnatal hemopoiesis. Interaction of stromal and hemopoietic elements.
12. Hemogram and leykocytic formula, significance for clinic. Erythrocytes,
structure and functions.
13. Hemogram and leykocytic formula. Plateletes, their number, functions, life span.
14. Thrombopoiesis. Structure and functions of platelets.
15. Leykocytic formula. Prenatal and postnatal leykocytopoiesis .
16. Leykocytes. Classification,morphofunctional characteristic. Leukocytes formula,
its peculiarities on different stages of ontogenesis.
17.Leykocytes of blood. Basophils and eosinophils.
18. Leukocytes formula. Morphofunctional characteristic of monocytes.
Concept of mononuclear phagocyte system of organism.
19.Macrophages and lymphocytes, their structure, histochemical characteristic and
participation in immune reactions.
20.Characteristic of immunocompetent cells T- and B-lymphocytes, their
development, proliferation and differentiation.
CONNECTIVE TISSUES.
21.Connective tissue proper. Its structure, types and functional significance.
Formation of extracellular matrix (on example of collagen synyhesis).
22.Extracellular matrix of connective tissue( fibers, ground substance), structure,
significance.
23.Extracellular matrix of connective tissue. Collagen and elastic fibers, their
structure and functions.
24.Cells of connective tissue. Their structure and functions.
25.Loose connective tissue. Morphofunctional characteristic. Macrophages: structure
and origins of development. Concept of mononuclear phagocyte system of
organism.
26.Dense connective tissue. Morphofunctional characteristic. Structure of dense
regular connective tissue ( on example of tendon).
27.Macrophages: morphofunctional characteristic. Their participation in immune
reactions. Concept of mononuclear phagocyte system of organism.
28.Cells of connective tissue. Macrophages, plasma cells and their participation in
defence reactions of organism.
29.Special connective tissue ( reticular ,mucous, pigment, adipose). Structure and
functions.
CARTILAGE AND BONE.
30.Cartilage. Its classification, structure and functions. Development of cartilages,
their regeneration and age changes.
31.Bone. Its classification. Morphofunctional characteristic.
32. Woven bone. Its histogenesis, structure, regeneration and age changes.
33. Lamellar bone. Tubular bone. Structure, development, regeneration.
34. Lamellar bone. Their morphofunctional characteristic. Regeneration
of tubular bone and factors, which regulate bone structure .

TESTING SPECIAL TISSUES.


MUSCLE TISSUE.
1.Muscle tissue. Origins of development. General morphofunctional characteristic.
Smooth muscle. Histogenesis, structure, regeneration.
2.Muscle tissue. Origins of development, general morphofunctional characteristic.
Skeletal muscle. Structure, nerve supply, basic structures of contructile apparatus.
Regeneration.
3. Skeletal muscle. Conception of red and white muscle fibers. Structure of muscle as
organ.
4. Cardiac muscle. Development, microscopic and ultramicroscopic structure.
NERVOUS TISSUE.
5. Nervous tissue. Morphofunctional characteristic. Origins of development.
Neurons. Morphologic and functional classifications.
6. Neuroglia. Classification, structure and significance of different types of
neuroglia.
7. Nerve fibers. Morphofunctional characteristic of myelinated and unmyelinated
nerve fibers.
8. Nerve endings. Classification of types. Morphofunctional characteristic of
effector endings.
9. Nerve endings. Morphofunctional characteristic of receptors.
10. Nervous tissue. General characteristic. Synapses, their structure and functions.
11.Origins of development. Morphofunctional characteristic. Conception of
somatic and autonomic reflex arcs.
S/M 5-6
1 At survey of the patient the poor amount of immunoglobulins has been detected. What from the enumerated cells of an
immune system produce immunoglobulins?
A. Plasma cells (plasmocytes)
B. Plasmablasts
C. Т-killers cells
D. T-helper cells
E. T-suppressor cells

2. It is known, that plasma cells produce specific antibodies on the given antigen. At injection of an antigen the amount of
plasmocytes increases. Due to what blood cells the augmentation of number of plasmocytes is carried out?
A.B-cells
B. Eosinophils
C. Neutrophils
D. Basophils
E. T lymphocytes

3. At the patient after transplantation of heterologous renal transplant the response of a casting-off has developed. What
basic effecter cells participate in the given immunological response?
A. Cytotoxic T lymphocytes
B. T-helper cells
C. T-suppressor cells
D. Plasmocytes
E. В-plasmocytes

4. At survey frequently sick child the poor amount of immunoglobulins has been detected. Which of the enumerated cells of
an immune system produce immunoglobulins?
A. Plasma cells
B. T-helper cells
C. Macrophages
D. Plasmablasts
E. T-suppressor cells

5. In a patient with clinical signs of immunodeficiency the number and functional activity of T and B-lymphocytes are not
changed. Defect with dysfunction of antigen-presentation to the immunocompetent cells was found during investigation on
the molecule level. Defect of what cells is the most probable?
A. Macrophages, B-cells
B. NK-cells
C. T-lymphocytes
D. Fibroblasts, T-lymphocytes, B-lymphocytes
E .0-lymphocytes

6. The specimens present sections of hemopoietic and immunogenetic organs. Organ consists of lymphoid tissue forming
different structures (lobules, cortex, medulla ). In what organ does antigen-independent, proliferation and differentiation
take place.
A. Thymus
B. Lymph nodes
C. Spleen
D. Tonsil
E. Hemolymph nodes

7. Live vaccine is injected into the human body. Increasing activity of what cells of connective tissue can be expected?
A. Plasma cells and lymphocytes cells
B. Fibroblasts and mast cells
C. Adipose cells and adventitial
D. Macrophages and fibroblasts
E. Pigment cells and pericytes

1
8. The patient with complaints of pain in eyes, which have arisen after duration stay of the patient in a field during dust burs,
converted to the ophthalmologist. The doctor has established the surface damages of a cornea epithelium. What cells will
provide regeneration of the defective epithelium?
A. Cells stratum basale
B. Cells of a stratum corneum
C. Cells of a stratum granulosum
D. Cells of a stratum lucidum
E. Superficial cells

32. The inner envelope of the blood and lymph vessels lines an epithelium. Name it.
A. An endothelium
B. Stratified squamous epithelium
C. Simple squamous epithelium
D. Epidermis
E. Transitional epithelium

33. The patient with complains to a pain in a thoracic cavity at respiration converted to the doctor. After survey, there were
detected modifications of an epithelium of pleura. What epithelium has tested modifications?
A. Simple squamous
B. Simple columnar ciliated
C. Simple cuboidal
D. Simple columnar
E. Stratified columnar

34. Under action of radiation cells of a basal layer of an epidermis have suffered. What function of the last will weaken or will
be break first?
A. Regenerative
B. Absorptive
C. Protective
D. Barrier
E. Secretory

35. In experiment the significant amount of stem cells of bone marrow definitely eroded. Regeneration of what population of
cells in composition of loose connective tissue will be inhibited.
A. Macrophages
B. Fibroblasts
C. Pigment cells
D. Adipose cells
E. Pericytes

36. The initial state of an inflammation - alterative is characterized by dilating of blood capillaries on a field of damage,
decrease of a circulation, a raise of a permeability of a wall of vessels. What cells which given below, the leading role in
inflammatory response is necessary?
A. Mast cells
B. Fibroblasts
C. Plasma cells
D. Eosinophils
E. Macrophages

39. After the transferred chemical injury of an esophagus there has occurred its local narrowing owing to formation of
collagenic cicatrix. What cells of a loose connective tissue share in formation of cicatrices?
A. The mature active fibroblasts
B. Inactive fibroblasts
C. Fibrocytes
D. Myofibroblasts
E. Adipose cells

40. At injection to the patient of a proteinaceous drug the amount of plasma cells which produce specific antibodies on the
given antigen increases. Due to what blood cells there is an augmentation of an amount of plasma cells?
2
A. B cells
B. T-suppressor cells
C. Т-killers
D. T-helper cells
E. В-memory cells

56. At the patient the tremor and convulsions that is caused by breaking of nerve fibers myelination of a central nervous
system is observed. Function of what cells suffers at this disease?
A. Olygodendrocytes
B. Fibrillar astrocytes
C. Ependymal cells
D. Protoplasmatic astrocytes
E. Microglial cells

58. At patient with a chronic alcoholism disturbance of coordination of movements and equilibrium owing to morphologic
disturbances in cerebellum. Damage of what Cerebella cells we can suppose at a first head.
A. Purkinje cells
B. Basket cells
C. Golgi cells
D. Pyramidal cells
E. Stellate cells

59 Some nervous system organ is presented in histological section. It contains capsule, connective tissue layers that extend
from the capsule. Bipolar (pseudounipolar) neurons are located bellow capsule. Peryikaryons of these neurons are surrounded
by glial cells. Center of organ contains neurons processes. What organ is it?
A. Dorsal root ganglion (spinal ganglion)
B. Vegetative ganglion
C. Spinal cord
D. Cerebral cortex
E. Cerebellar cortex

60. Cerebral cortex contains large and giant pyramidal shaped neurons. It is a feature of the cerebral cortex. Discovery of
these cells was connected with the name of following scientist:
A. Bets
B. Purkinje
C. Golgi
D. Nissle`s
E. Paneth`s

61. In human embryo neural tube neuroepithelial ectodermal cells differentiate into neuroblasts and gliablasts. Owing to
migration of these cells some zones (layers) are formed in neural tube. Which of the following zone is where bodies of
neuroblasts locate in?
A. Mantle (intermediate) zone
B. Ventricular zone
C. Marginal zone
D. Outer limiting membrane
E. Inner limiting membrane

62 The increased intraocular tension is observed in the patient with glaucoma. Secretion of aqueous humor by the ciliary
body is normal. Injury of what structure of the eyeball wall caused the disorder of flow-out from the anterior chamber?
A. Venous sinus
B. Ciliary body
C. Choroid
D. Ciliary muscle
E. Posterior epithelium of cornea

63. Stromal compartment of some organ is composed of branch-shaped epithelial cells that form tri-dimensional meshwork.
What is this organ?
A. Thymus gland
B. Red bone marrow
3
C. Spleen
D. Lymph node
E. Tonsil

64. In a patient after transplantation of the foreign kidney rejection of the transplanted organ developed. What main effecter
cells participate in this reaction at first?
A.T-killers (T-cytotoxic)
B. plasma cells
C. T-lymphocytes –suppressors
D. B-cells
E Fibroblasts

65. Examination of a patient who was subjected to X-ray irradiation revealed injury of the splenic white pulp. What cells of
the white pulp undergo to these pathological changes at first?
A. Lymphocytes
B. Basophils
C. Neutrophils
D. Mast cells
E. Red blood cells

66. Morphologic investigation of a spleen revealed activation of immune reactions in organism. Within what splenic
structure antigen dependent proliferation of T-cells, begins
A. Periarterial lymphatic sheath
B. Corona of a lymphoid nodule
C. Marginal zone
D. Germinal center of a lymphoid nodule
E. Red pulp

67. Under influence of unfavorable factors on the organism fast reconstruction of a thymus gland took place. It accompanied
by the massive death of T-cells, its migration into peripheral lymphoid organs and by the proliferation of epithelial reticular
stromal cells. What is the name of that reaction?
A Accidental involution
B Age involution
C. Thymic atrophy
D.Thymic hypotrophy

68 Some organ is presented in histological section. It has cortex and medulla. Cortex is composed of outer cortex with
lymphoid nodules and paracortex. Medulla contains medullary cords, sinuses and trabecules. What organ is it?
A.Lymph node
B.Thymus
D. Kidney
C. Adrenal gland
E. Spleen

69 .At newborn boy congenital involution of a thymus is revealed. Disturbance of what cells differentiation we may wait for?
A. T-cells
B. B-cells
C. Macrophages
D. Plasma cells
E. B-memory cells

70. At a patient with AIDS supression of B-cells differentiation is revealed. In what organ antigen independent differentiation
and proliferation of B-cells takes place?
A. Red bone marrow
B.Lymph node
C.Tonsil
D.Thymus
E. Spleen

4
71. There is congenital atrophy of a thymus at a newborn. Absence of what zone in peripheral lymphatic organs takes place at
this child?
A. Paracortex in lymph node
B. Outer cortex in lymph node
C. Medulla in lymph node
D. Red pulp in spleen
E. Lymphatic nodules in spleen

72. There is displayed an organ of layer’s type structure at the histological section, which is covered by stratified squamous
keratinized epithelium. Under basement membrane of epithelium loose connective tissue, which protrudes in the form of
papillae, is found. The dense irregular connective tissue is located below and forms reticular layer. What organ does have this
morphological feature?
A. Skin
B. Tongue
C. Lingual tonsil
D. Esophagus
E. Uterine cervix

73 The child’s scratch, which had been caused by light skin’s trauma, disappeared after 10 days. What skin layer provided
for reparative regeneration?
A. Stratum basale;
B. Stratum spinosum;
C. Stratum granulosum;
D. Stratum corneum;
E. Stratum brilliantum.

74. It is determined the blood vessel on the histological micropreparation of the spleen. Its wall consists of tunica intima
(endothelium, basal membrane), lack of the distinct tunica media, tunica adventitia (contains collagen and elastic fibers, that
anchor the vessel in the surrounding connective tissue trabeculae. What type of vessel is it?
A. Fibrous vein
B. Muscular vein
C. Arteriole
D. Muscular artery
E. Elastic artery

75. Which of the following is an embryonic origin of vessel wall?


A. Mesenchyme
B. Endoderm
C. Mesoderm
D. Ectoderm
E. Neural tube

76. It is determined the large, oval and round cells with light cytoplasm and a small number of organelles and myofibrils on
histological micropreparation of the heart wall. What are these cells?
A. Purkinje fibers
B. Contractile cardiac muscle
C. Secretory cardiac muscle
D. Smooth muscle
E. P- cells

77. It is the wall of heart on the histological micropreparation. There are contractile, secretory and conductive cardiac
muscle cells and endomysium with blood vessels. For which of the following layers are, these structures belong to?
A. Myocardium of atria
B. Endocardium of ventricles
C. Pericardium
D. Tunica adventitia
E. Epicardium

78. They are determined cardiac muscle cells with glycoprotein rich granules. What are these cells?
A. Secretory
5
B. Contractile
C. P-cells
D. Purkinje cells
E. Transitional cells

79. It was determined the anastomosing in cords one of the layers of cardiovascular system organ. These cords consist of cells
with junctional specializations making up the intercalated disk. What tissue forms this layer?
A. Striated cardiac
B. Striated skeletal
C. Smooth muscle
D. Loose connective
E. Dense connective

80. Myocardiodistrophy promotes a breach of cardiac muscle cells metabolism. Which of the following embryonic origin
breach causes this pathology?
A. Myoepicardial plate
B. Myotome
C. Endoderm
D. Ectoderm
E. Mesenchyme

81. In the histological specimen slender conical projections of the stratified keratinized epithelium and underlining connective
tissue are found, 2-3 mm in length, with their tips pointing toward the one side. Which of the following are in the slide?
A. Filiform papillae of the tongue
B. Circumvallate papillae of the tongue
C. Papillary layer of the dermis
D. Dental papillae
E. Fungiform papillae of the tongue

82. The teen-ager was admitted to an infectious disease hospital with epidemic parotids. The virus of epidemic parotitis
destroys glandular cells of parotid glands. Which of the following cell types undergo destruction?
A. Serous cells
B. Mucous cells
C. Plasma cells
D. Myoepithelial cell
E. Pigment cells

83. The serous cells form the secretory potions of a parotid gland. Which of the following organelles provide synthesis and
secretion of saliva components?
A. Roughed endoplasmic reticulum, Golgi apparatus
B Mitochondria, Golgi apparatus
C. Smooth endoplasmic reticulum
D. Lysosomes
E. Endosomes

84. The patient is 46-years-old man. He complains of increased appetite, heartburn, acid belching, epigastric pain and unstable
stool (constipation mainly). Laboratory examination of gastric juice pH discovers total hyperacidity. Which of the following
cells’ activity is increased?
A. Parietal (oxyntic) cells
B. Chief cells
C. Mucous neck cells
D. APUD cells
E. Paneth`s cells

85. The 5-year-old boy was admitted to the gastroenterologic department of children’s hospital. He has felt acute pain in the
epigastria area just after he drank unknown fluid. Patient complains of nausea, vomiting, cutting stomach pain. Endoscopic
examination of the stomach reveals destruction of the gastric mucous layer. Regeneration of epithelial cell types in the
stomach does come from differentiation of:
A. Mucous neck cells
B. Parietal (oxyntic) cells
6
C. APUD cells
D. Chief cells
E. Surface-lining cells

86. The patient 32-years-old complains of lost appetite, belching, nausea and dull pain in the epigastria region. Laboratory
examination of the gastric juice reveals absence of pepsinogen. Pepsinogen production is associated with which of the
following cell types?
A. Chief cells
B. M-cells
C. Parietal (oxyntic) cells
D. APUD cells
E. Mucous neck cells

87. In the surgical specimen (biopsy) colored by the hematoxylin-eosin you can see large cells with oxyphilic cytoplasm.
Electron microscopy shows system of intracellular canaliculi. What do they produce?
A. Hydrochloric acid (chlorides)
B. Gastrin
C. Pepsinogen
D. Secretin
E. Mucinogen

88. Patient 42-years-old is 2 years after total resection of the stomach. He complains of general weakness, lost appetite,
paleness. Doctor put him diagnosis “Pernicious anemia”. Which of the following cell type’s absence does initiate this
pathology?
A. Parietal (oxyntic) cell
B. Chief cell
C. Mucous neck cell
D. Surface – lining cell
E. APUD cell

89. In a histological specimen of a hollow organ the lining stratified squamous nonkeratinized epithelium is seen in the
mucous. What organ is it?
A. Esophagus
B. Uterus
C. Duodenum
D. Colon
E. Appendix

90. One of the heart layers similar to blood vessel wall by histogenesis and cellular structure. What is the embryonic origin
of this layer?
A. Mesenchyme
B. Splanchnic mesoderm
C. Endoderm
D. Ectoderm
E. Somites

91. Submucous layer of intestine contains serous gland. What part of intestine is in histological slide?
A. Duodenum
B. Coecum
C. Large intestine
D. Ileum
E. Appendix

92. Section of basal portion of intestinal crypts of Lieberkuhn showing the basophilic cells with large acidophylic granules.
What type of cell is it?
A. Paneth`s cells
B. Surface absorptive cells
C. Goblet cells
D. Enteroendocrine cells
E. M-cells
7
93. After examination of a patient with small intestine disease disturbance of terminal digestion was found. Terminal digestion
is associated with which of the following cell type?
A. Surface absorptive cells
B. Goblet cells
C. Enteroendocrine cells
D. M-cells
E. Paneth`s cells

94. In light microscope section you may see epithelial cells are grouped in interconnected plates that form polygonal lobule.
The space between these plates contains sinusoidal capillaries. What organ is presented?
A. Liver
B. Spleen
C. Pancreas
D. Thymus
E. Lymph node

95. Secretion of which gastrointestinal hormones is primarily decreased in a patient with removed duodenum?
A. Cholecystokinin and secretin
B. Gastrin
C. Neurotensin
D. Gastrin and histamine
E. Histamine

96. An attack of bronchial asthma began at a patient at night. Cyanosis and dyspnea are observed at a patient. What parts of
the bronchial tree participate in this pathologic process?
A. Small (intralobular) bronchi
B. Primary ( large, extrapulmonary) bronchi
C. Medium (lobar) bronchi
D. Terminal bronchiole
E. Alveoli.

97. Autopsy of a man who suffered from pulmonary disease revealed pathologic process in bronchi. Histological
investigation reveled presence of glands, cartilage plates and simple columnar pseudostratified epithelium. Into what bronchi
type changes were observed?
A. Medium (lobar) bronchi
B. Principal (large, extrapulmonary) bronchi
C. Small (intralobular) bronchi
D. Terminal bronchioles
E. Alveoli

98. After histological investigation of lungs at a patient who suffered from pneumonia. Injury of alveolar cells that are
responsible for respiratory function was observed. What cells are these?
A. Pneumocyte type 1
B. Pneumocyte type 2
C. Alveolar macrophages
D. Clara cells
E. Fibroblasts

99. After histological examination of the respiratory system some tubular organ is revealed. In the wall of this organ
epithelium is not tall, smooth muscle layer is very thick, absence of glands and cartilage. Name this organ.
A. Small (intralobular) bronchi
B. Larynx
C. Principal (large, extrapulmonary) bronchi
D. Trachea
E. Alveoli

100. Lung of premature infant is presented on electron micrograph of biopsy material. Collapse of the alveolar wall caused
by the deficiency of surfactant was revealed. Which cell type dysfunction of the alveolar wall has been caused it?
A. Pneumocyte type 2
8
B. Fibroblasts
C. Alveolar macrophages
D. Secretory cells
E. Pneumocyte type 1

101. Characterizing a stress, the student has supposed an inaccuracy when has told, that synthesis of glucocorticoids of a
suprarenal cortex is stimulated by hormones of a pituitary gland. What improvement is necessary?
A. Аdrenocorticotropic hormon (ACTH)
B. Somatotropin
C. Prolactin
D. Gonadotropic hormones
E. Тhyrotropic hormone

102. At the woman 30 years old have revealed a shortage of sexual hormones, have found an increase level of follicle
stimulating hormone. What cells of an Adenohypophysis will be modified thus?
A. Gonadotropes
B. Thyrotropes
C. Mammotropes
D. Corticotropes
E. Somatotropes

103. At the patient who during 7 years suffers a hypothyroidism, failure of thyroid hormones is detected. What cells of an
Adenohypophysis thus will be modified?
A.Thyrotropes
B. Gonadotropes
C. Mammotropes
D. Corticotropes
E. Somatotropes

104. The removing of an endocrine gland of experimental animals occurs a premature puberty. Which of the following gland
is removing can occur a premature puberty?
A. A pineal gland
B. A pituitary gland
C. Adrenal gland
D. A thyroid gland
E. Parathyroid gland

105. During a puberty man's sexual glands produce to the blood man's sexual hormone testosterone which is responsible for
development of the secondary sexual features.. What cells of testes synthesize this hormone?
A. Leydig cells
B. Sertoli cells
C. Spermatozoons
D. Fibroblasts
E. Spermatids

106. It was determined, that the thyroid follicles are enlarged, owing to increases of follicular epithelial height and increased
thyroglobulin deposits on the micropreparation of the thyroid gland. Which functional state is the following histological
picture characteristic of?
A. Hyper function
B. Hypo function
C. Moderate activity
D. Age changes
E. Rise of iodine level the blood plasma

107. A 2-year- old child experienced convulsions because of lowering calcium ions concentration in the blood plasma.
Function of what structure is decreased?
A. Parathyroid glands
B. Hypophysis
C. Pineal gland
D. Thymus
9
E. Adrenal cortex

108. Usage of oral contraceptives with sex hormones inhibits secretion of the hypophyseal hormones. Secretion of which of
the indicated hormones is inhibited while using oral contraceptives with sex hormones.
A. Follicle-stimulating
B. Oxytocin
C. Vasopressin
D. Somatotropic
E. Thyrotropic

109. Upward invagination of ectoderm lining primitive oral cavity forms the Rathke’s pouch. Which of the following
structures does develop from this embryonic origin?
A. Adenohypophysis
B. Median eminence
C. Neurohypophysis
D. Neural stark
E. Pars distalis

110. Patient with Cushing syndrome (high level of glucocorticoids in the blood) has the high blood sugar. Which of the
following organ pathology is the cause of this disease?
A. Adrenal cortex
B. Adrenal medulla
C. Pancreas
D. Pineal gland
E. Thyroid gland

111. It is known, that aldosterone controls water and electrolyte balance in the organism. Which of the following cell types
of adrenal gland produce this hormone?
A. Cells of Zona glomerulosa
B. Cells of Zona reticularis
C. Epinephrine- secreting cells
D Cells of Zona fasciculata
E. Norepinephrine-secreting cells

113. The operation of thyroid gland sectoral ectomy caused spastic muscular contraction of a 30- year old woman. It was
determined the low blood calcium. Lack of what hormone secretion disrupts neuromuscular function?
A. Parathyroid hormone
B. Thyroxin
C. Melatonine
D. Somatotropin
E. Antidiuretic hormone

114. Patient had been receiving the great doses of hydrocortisone for a long time. Which of the following cells of adrenal
cortex must be atrophied after the influence of this exogenous hormone?
A. Cells of zona fasciculata
B. Cells of zona reticularis
C. Cells of adrenal medulla
D. Cells of zona glomerulosa
E. Cells of capsule

115. Arterial hypertension is caused by the stenosis of the renal arteries in the patient. Activation of what system is the main
link in the pathogenesis of this form of hypertension.
A. Renin-angiotensin
B. Hypothalamic-pituitary
C. Parasympathetic
D. Sympathoadreal
E. Kallikrein-kinin

116. In a patient with chronic glomerulonephritis acidity of urine does not occur and bacteriostatic effect is absent. Which
structures of collecting tubules are associated with hydrochloric acid production?
10
A. Dark cells
B. Glomerulus
C. Podocytes of capsule
D. Proximal tubule
E. Distal tubule

117. The electron microscopic photograph of a kidney’s fragment shows afferent arteriole. Under the endothelium of arteriole,
large cells are seen with secretory granules in the cytoplasm. Name this type of cells:
A. Juxtaglomerullar cells
B. Intraglomerullar mesangial cells
C. Extraglomrullar cells
D. Smooth muscle cells
E. Interstitial cells

118. The electron microscopic photograph of renal corpuscle demonstrates branched cells between capillaries of glomeruli.
Large number of filaments is seen in the cytoplasm of these cells. Name this type of cells.
A. Mesangial cells
B. Jucstaglomerullar cells
C. Podocytes
D. Smooth muscular cells
E. Interstitial cells

119. The patient with glomerulonephritis complains of high blood pressure caused pathological process of nephrons. Which
of the following vessels of the kidney was distressed?
A. Fenestrated capillaries
B. Continuous capillaries
C. Discontinuous capillaries
D. Fibrous veins
E. Elastic artery

120. A 10-14 % of newborn suffer from developmental anomalies. Which embryonic origin urinary system develops from?
A. Intermediate mesoderm
B. Dorsal mesoderm
C. Visceral layer of ventral mesoderm
D. Parietal layer of ventral mesoderm
E. Mesenchyme

121. Slow healing of an umbilicus is caused by patent primitive urinary duct - urachus. Which embryonic origin is urinary
bladder and urachus develop from?
A. Allantois
B. Cloacae
C. Urogenital sinus
D. Paramesonephric duct
E. Mesonephric duct

122. Clinical examination of the 35-year-old women with kidney’s disease reveals presence of blood cells and fibrinogen in
urine that says about distressed filtration barrier. Renal filter consists of:
A. Endothelium, podocyte, basal membrane
B. Podocyte, basal membrane
C. Only basal membrane
D. Endothelium, basal membrane
E. Endothelium, podocyte

123. Electron microscopic photograph of a renal corpuscle shows the large cell with large and numerous small processes.
Small processes attach the basal membrane of fenestrated capillaries. Name this cell.
A. Podocyte
B. Juxtavascular cell
C. Smooth muscular cell
D. Endothelial cell
E. Mesangial cell
11
124. In microslide the muscle - gland organ, which has a lobules, is introduced. In lobules glands are posed, ducts are
unclosed in the large canal located at the center of a organ, the wall of this canal is covered by a pseudostratified colomnar
epithelium. Stroma of the organ compound of loose connective and muscle tissues. What organ is it?
A. A prostate
B. A uterus
C. A seminal vesicles
D. An epididymis
E. A mammary gland

125.During a puberty man's sexual glands produce in a blood man's sexual hormone testosterone which is responsible for
development of the secondary sexual features.. What cells of spermaries synthesize this hormone?
A. Leydig cells
B. Sertoli cells
C. Spermatozoons
D. Fibroblasts
E. Spermatids

126. In histological micropreparation there is an organ, which is covered by tunica vaginalis and tunica albuginea. Stroma of
the organ consists of connective tissue with the Leydig’s cells. Parenchyma contains tubules, which are lined by the
spermatogeneous epithelium. What organ is there?
A. Testis
B. Prostate gland
C. Epididymis
D. Mammary gland
E. Ovary

127. Patient complains of frequent and difficult urination. Imperfection of what formation can cause it.
A. Prostate
B. Epedidimis
C. Bulb urethral glands
D. Sperm bubbles
E. Testicles

128. In the ovary specimen colored with hematoxylin-eosin, follicle is determined where cubic-shaped follicle epithelium
cells are placed in 1-2 layers, and scarlet covering is seen around ovocyte. Name this follicle:
A. Primary
B. Secondary
C. Mature
D. Atretic
E. Primordial

129. At the histological section of ovary stained by hematoxylin and eosin one can determine follicle consists of 1-2 layers
cuboidal follicular cells. A blazing–red coat covers oocyte. Name this follicle.
A. Primary follicle;
B. Primordial follicle;
C. Secondary follicle;
D. Graafian (mature) follicle;
E. Atretic follicle.

130. There are lot of large branched-shaped cells found in the myometrium, which contain smooth myofillaments in the
cytoplasm, and well-developed rER. What are these cells?
A. Smooth muscle cells;
B. Myoepithelial cells;
C. Skeletal muscle fibers;
D. Fibrocytes;
E. Myofibroblasts.

131. In the microslide of ovary a spherical structure 5 sm in diameter is revealed. Its large cells contain yellow pigment and
lipid droplets. From what type of cells this ovarian structure is composed of?
12
A. Lutein cells;
B. Myoid cells;
C. Follicular cells;
D. Interstitial cells;
E. Fibroblasts.

132. The pregnant woman was a subject of caesarean section operation with the aim of fetus extraction. The uterus wall was
sected at the long distance. By what means will a healing in the myometrium stitch take place?
A. Formation of connective tissue cicatrix
B. Hypertrophy of smooth muscle cells
C. New formation of smooth muscle tissue
D. Formation of skeletal muscle fibers
E. Proliferation of satellite cells

133. A 34-years-old patient was admitted to a hospital after significant hemorrhage. Decreased blood volume promotes
decreased blood pressure and activates the renin-angiotensin-II- aldosterone mechanism that contributes to the maintenance of
blood pressure in a patient’s organism. Which of the following cells are producing renin?
A. Juxtamedullary cells
B. Juxtaglomerular cells
C. Light cells of collecting tubule
D. Interstitial cells
E. Dark cells of collecting tubule

134. A girl of 5-years-old has admitted to nephrologic department of a municipal hospital. She complains of pain in low
back, urine contains constantly raised amount of red blood cells (erythrocytes), proteins (albumins and globulins). Which
from the following structures is destroyed?
A. Glomerulus of nephrons
B. Proximal convoluted tubule of nephron
C. Distal convoluted tubule of nephron
D. Mesangium of glomerulus
E. Collecting tubule of nephron

135. A woman complains of excessive menstruation, menstrual colic usually. Ultrasound investigation of the uterus shows
16 mm thickness of endometrium (N 8-9mm) 1day before menstruation. Which from the following cells are producing an
increased level of estrogen and are stimulating overgrowth of endometrium?
A. Cells of granulosa in secondary and Graafian follicles
B. Endocrine cells of Adenohypophysis
C. Endocrine cells of pars nervosa
D. Cells of adrenal cortex
E. Cells of adrenal medulla

136. A patient has observed by a doctor at a polyclinic. Patient complains of acute pain in epigastria region after he’s taken
a tablet of aspirin. Aspirin can disrupt the epithelial layer of stomach and initiate the ulceration. Which type of cells can be
destroyed at first?
A. Surface- lining cells
B. Mucous neck cells
C. DNES cells
D. Parietal cells
E. Chief cells

137. A 2-years-old boy was admitted to the hospital with malabsorption syndrome. Deficiencies of disaccharides and
dipeptidases have been found in this patient. What from the following types of epithelium is providing deficiency of
enzymes?
A. Simple columnar epithelium
B. Simple cuboidal epithelium
C. Simple squamous epithelium
D. Pseudostratified epithelium
E. Simple columnar ciliated epithelium

13
138. In the histological specimen an organ is seen. Wall of this organ consists of 4 layers: first is mucous layer forming pits
and glands; second is submucous layer of connective tissue; third is muscularis externa containing tree layers of smooth
muscles; fourth is serous layer. Which organ can be recognized in the specimen?
A. A stomach
B. A jejunum
C. A duodenum
D. An esophagus
E. A colon

139. At a vulgar pemphigus in a skin epidermis forms vesicles, which breaks out a wholeness of stratum spinosum and
stratum granulosum. What function of an epithelium is broken first?
A. Barrier
B. Regenerative
C. Absorptive
D. Secretory
E. Endocrine

140. Under the influence of long vibration and a motion in motor regions of cortex brain occurs a gliolis – phagocytosis of
damaged neurons by a neuroglial cells. What neuroglia participates in this process?
A. Microglia cells
B. Protoplasmatic astrocytes
C. Oligodendrocytes
D. Ependymal cells
E. Fibrous astrocytes

141. At a person genetic disorder of metabolism of amino acid tyrosine broke synthesis of a black pigment melanin. In what
cells of loose connective tissue and a skin epidermis lack of this pigment will be observed?
A. Pigment cells
B. Dendritic cells
C. Adipose cells
D. Mast cells
E. Pericytes

142. At a combustion of a skin the patient feels a strong pain. Processes of what neurons first conduct pain nervous impulse in
central nervous system.
A. Pseudounipolar neurons
B. Bipolar neurons
C. Unipolar neurons
D. Multipolar neurons
E. Purkinje neurons

143. After two months of appendectomy the patient again has arrived in surgical department with the diagnosis: «The
intestinal obstruction, an adhesive disease». During operation damage of what epithelium covering a peritoneum has reduced
in development of adhesive disease?
A. Mesothelium
B. Endothelium
C. Transitional epithelium
D. Simple columnar epithelium
E. Simple cuboidal epithelium

144. During road accident at the man of 30 years the femoral artery has been injured. In hospital the arteriorraphy was
performed. Regeneration of what muscle tissue is necessary for regeneration of a wholeness of the vessel’s tunica media.
A. Smooth muscle tissue
B. Striated skeletal muscle tissue
C. Striated cardiac muscle tissue
D. Myoepithelial cells
E. Satellite cells
14
147. On microslide of the bulb of the fetus eye a damage of a cornea is observed. Part of what germ layer has been struck
during embryonic development
A. Ectoderm
B. Endoderm
C. Mesoderm
D. Dermatome
E. Nephrotome

148. At examination, replying about development of hard and soft tissues of dens, the student has made a mistake, when has
told, that enamel is formed of a mesenchyme cells. What should be a right answer?
A. From inner enamel epithelium
B. From stellate reticulum
C. From outer enamel epithelium
D. From dental sac
E. From dental papilla

149. At a brain injury are defected glial cells, which meet in gray matter to central nervous system more often. What is the
name of these cells?
A. Protoplasmic astrocytes
B. Oligodendrocytes
C. Microglia
D. Fibrous astrocytes
E. Ependymal cells

150. As a result of the transferred orchitis (an inflammation of a testis) at the man of 43 years old the spermatozoons
producion was broken. In what testis structures pathological changes were formed?
A. Seminiferous tubules
B. Rete testis
C. Tubuli seminiferi recti
D. Ductuli efferentes testis
E. Ductus deferens

151. During an embryogenesis there was a breaking shaping of a pharyngeal gut. Specify possible localization of anomalies of
development?
A. Organs of an oral cavity.
B. A stomach
C. An ileum
D. A pancreas
E. A jejunum

152. The neonatal child has a disorder of a thymus gland. What type of a hemopoiesis will be disturbed?
A. A lymphopoiesis
B. A monocytopoiesis
C. An erythropoiesis
D. Granulocytopoiesis
E. Platelet formation

157. In chronically diseased liver some cells proliferate and acquire the feature of myofibroblasts with or without the lipid
droplets. These cells are found close to the damaged hepatocytes and play a major role in development of fibrosis. What are
these cells?
A. Ito’s cells
B. Hepatocyte
C. Kupffer cells
D. Pit cells
E. Adipose cells

15
158. One of the principal types of diabetes (type 1) is an autoimmune disease in which antibodies against endocrine cells of
pancreas (islets of Langhergans) and depresses activity of cells. Which cells activity is depressed?
A. B-cells
B. A-cells
C. D-cells
D. PP-cells
E. D1-cells

159. Examination of the patient with pernicious anemia revealed antibodies against cell’s proteins of gastric glands. What
kind of cells in gastric glands was destroyed?
A. Parietal cells
B. Mucous cells
C. Chief cells
D. DNES-cells
E. Surface- lining cells

160. The epithelial enamel organ is observed in maxilla’s histological section of 3.5-month human fetus. Mesenchyme cells
surround this enamel organ. What is name of these cells?
A. Dental sac
B. Odontoblasts
C. External enamel cells
D. Internal enamel cells
E. Stellate reticulum

161. The histological slide shows an organ of oral cavity consisting of tree principal parts: cutaneus, intermediate, mucous.
What organ is described here?
A. Lip
B. Gum
C. Hard palate
D. Soft palate
E. Cheek

163. In the histological specimen of an organ numerous lymphocytes in periphery of each lobule and stromal reticular
epithelial cells can be observed. Which of the following hematopoietic tissue or organ develops from endoderm?
A. Thymus
B. Tonsils
C. Bone marrow
D. Spleen
E. Blood islands

165. The cystoscopy of a patient with cystic tumor revealed the area of mucous layer that is without folds.
Which of the following parts of urinary bladder was observed?
A. Fundus
B. Body
C. Cervix
D. Apex
E. All mucous layer

166. Hematuria (damaged erythrocytes) is detected in the urine of a patient with glomerulonephritis. Which part of nephron is
destroyed in this patient?
A. Bowmen’s capsule
B. Proximal convoluted tubule
C. Distal convoluted tubule
D. Collecting tubule
E. Henle’s loop

171. Scientists marked developing B lymphocytes of the red bone marrow by radioactive label. In what regions of
peripheral hematopoietic organs this label may be determined at first?
A. Outer cortex and medulla of lymph node
16
B. Paracortex of lymph node
C. PALS of spleen
D. Splenic red pulp cords
E. Splenic sinusoids

172.Scientiests marked developing T lymphocytes of thymus gland by the radioactive label. In what regions of peripheral
hematopoietic organs this label may be determined at first?
A. Paracortex of lymph node, PALS of spleen
B. Medulla of lymph node
C. Outer cortex of lymph node
D. Splenic red pulp cords
E. Splenic sinusoids

173. After overdoses of -rays destruction of B-lymphocytes in human hematopoietic organs happens. Reduction of what
blood plasma substances will be observes in this case.
A. Immunoglobulins (antibodies)
B. Albumins
C. Cholesterol
D. Glucose
E. Fibrinogen

174. A patient with leucosis has disturbance of some type of hematopoiesis. This type has next characteristic: antigen
independent and antigen dependent phases, blastic transformation, clonal expansion, recirculation. What type of
hematopoiesis has enumerated features?
A. Lymphocytopoiesis
B. Granulopoiesis
C. Erythropoiesis
D. Monopoiesis
E.Thrombopoiesis

175.A patient suffers from epithelial tumor of the right extrapulmonary bronchus. What morphologic type of epithelium gave
rise to tumor?
A. Simple columnar pseudostratified
B. Simple squamous
C. Simple cuboidal
D. Stratified squamous nonkeratinized
E. Transitional

176. Wife and husband complained of children birth impossibility. Injury of spermatogenic testicular epithelium was revealed
after examination. Due to this absence of spermatozoa in seminal fluid and male infertility took place. Injury of what testicular
regions took place in this case?
A. Seminiferous tubules
B. Straight tubules
C. Rete testis
D. Ductus epididymis
E. Efferent ductules

177. After gynecologic operation woman complained of pain in lower abdominal region and vaginal discharge. Gynecologist
made a diagnosis - inflammation of inner uterine layer. Name this layer.
A. Endometrium
B. Myometrium
C. Perimetrium
D. Parametrium
E. Peritoneum

179. A dentist revealed abnormal development of the enamel at a patient. What components of the enamel organ were injured
at an embryo?
A. Outer epithelium of enamel organ
B. Inner epithelium of enamel organ
C. Stellate reticulum of enamel organ
17
D. Dental papilla
E. Dental sac

180. Injury of superficial mesenchyme cells of enamel organ’s dental papilla took place. Disturbance in formation of what
dental tissue can wait for?
A. Dental dentin
B. Dental enamel
C. Dental pulp
D. Dental cementum
E. Cuticle of enamel

181. Toxin of virus caused injury of cells of enamel organs dental sac. Disturbance in formation of what dental tissue can
wait for?
A. Dental cementum
B. Dentin
C. Dental pulp
D. Enamel
E. Cuticle of enamel

182. A patient is ill with cerebellum tumor. Due to this shakiness of the step is developed. What Cerebella cortex cells
provide equilibrium in normal states at first?
A. Purkinje cells
B. Small pyramidal cells
C. Basket cells
D. Large pyramidal cells
E. Golgi cells

183. Some sensory organ is presented on the histological section. Main morphologic component of it is a chain that
composed of 3 neurons. Name this structure.
A. Retina of the eye
B. Cornea of the eye
C. Organ of Corti
D. Organ of equilibrium
E. Organ of taste

184. Toxin of virus caused death of pseudounipolar neurons. Due to this loss of sensitivity took place at a patient.
Disturbance of what organ function is observed at this patient.
A. Dorsal root ganglion
B. Spinal cord
C. Cerebral cortex
D. Cerebellar cortex
E. Sympathetic ganglion

185. Death of ciliated epithelial cells in bronchi took place at a worker of chemical concern after inspiration of hydrochloric
steams. After 30 days, normal structure of conducting portion epithelium was restored. What epithelial cells in respiratory
epithelium took part in regeneration?
A. Basal cells
B. Goblet cells
C. Clara cells
D. Endocrine cells
E. Brush cells

186. Enteroendocrine sells differ from goblet cells in:


A. The direction of release of secretion
B. The use of exocytosis for release of secretary product from the cell
C. The presence in the small and large intestine
D. Secretion by the regulated pathway
E. They origin from a crypt stem cell

18
187. Parents brought a one-month-old male infant to the emergency room by his. The examining emergency room physician
notes that his skin and sclera are icteric. A blood test indicates elevated unconjugated bilirubin in the serum. The elevated
bilirubin levels in this patient are most likely the result of
A. Deficiency of hepatocyte enzymes regulating bilirubin solubility
B. Decreased destruction of red blood cells
C. Increased function of Ito-cells
D. Decreased function of Pit-cells
E. Deficiency of Kupffer- cells

188. The retinal pigment epithelium is characterized by


A. The presence of the photoreceptor (rod and cone) perikaria
B. Phagocytosis of worn-out components of photoreceptor cells
C. Origin from the inner layer of optic cup during embryonic development
D. Presence of amacrine cells
E. Synthesis of vitreous humor

192. Deficiency of vitamin A causes a breach of vision in twilight. Which of the following cells carry out this function?
A. Rods
B. Cones
C. Bipolar cells
D. Amacrine cells
E. Ganglion cells

193. There is connecting with the gut vesicle is visible on the histological Micropreparations. It is an extraembryonic organ.
Primordial germ cells and erythroblasts (megaloblasts) are located in its wall. What organ is this?
A. Yolk sac
B. Allantois
C. Placenta
D. Umbilical cord
E. Amnion

194. It was determined the fall of activity of amylase in saliva owing to investigation of 47- years- old man with extracted
salivary gland. What gland was extracted?
A. Parotid
B. Buccal
C. Submandibular
D. Palatinar
E. Submandibular

197. Which of the following is found exclusively in the renal medulla?


A. Thin loops of Henle
B. Distal convoluted tubules
C. Collecting ducts
D. Afferent arterioles
E. Proximal convoluted tubules

198. A 33-year-old patient with an average menstrual cycle of 28 days comes in for a routine Pap smear. It has been 35 days
since the start of her last menstrual period, and a vaginal smear reveals clumps of basophilic cells. As her physician, you
suspect
A. There are detectable levels of hCG in her serum and urine
B. She will ovulate within a few days
C. Her serum progesterone levels are very low
D. She will begin menstruating in a few days
E. She is undergoing menopause

199. Synthesis of milk by the mammary gland specifically requires


A. Prolactin
B. Production of progesteron by the corpus luteum
C. The influence of vasopressin
D. Placental lactogen
19
E. Neurohumoral reflexes

201.A histological spacemen presents parenchymal organ, which has cortex and medulla. Cortex consists of epitheliocytes bars,
between them there are blood capillaries; the bars form three zones. Medulla consists of chromafinocytes and venous sinusoids.
What organ has these morphological features?
A. Adrenal gland
B. Kidney
C. Lymph node
D. Thyroid
E. Thymus

202.A 39-year-old patient after radiotherapy because of hepatoma developed ulcer of small intestine. It was caused by the
inhibition of mitotic activity of the cells, which are responsible for regeneration of small intestine surface epithelium. Inhibition
of what cells mitotic activity does this patient have?
A. Regenerative cells
B. Paneth`s cells
C. Surface absorptive cells
D. DNES-cells
E. Goblet cells

204. The electronic microphotograph of kidney fragment has demonstrated the afferent glomerular arteriole, which under its
endothelium has giant cells, containing secretory granules. Name the type of these cells:
A. Juxtaglomerular
B. Interstitial
C. Mesangial
D. Juxtavascular
E. Smooth muscular

207.A patient complaining of weight loss (10 kg during 2 months), palpitation and exophthalmia came to the endocrinologist.
For the hyper function of what endocrine gland (glands) are these complaints the most typical?
A. Thyroid
B. Pancreas
C. Adrenal glands
D. Parathyroid glands
E. Ovaries

210.In the microspecimen of red bone marrow there were revealed multiple capillaries through the walls of which mature blood
cells penetrated. What type of capillaries is it?
A. Sinusoidal
B. Somatic
C. Fenestrated
D. Lymphatic
E. Visceral

212. Decreased blood supply to the organs causes hypoxia that activates fibroblast function. Volume of what elements is
increased in this case?
A. Exracellular matrix
B. Nerve elements
C. Parenchymatous elements of the organ
D. Lymphatic vessels
E. Vessels of microcirculatory bed

217. Careless student occasionally met his dean, The concentration of what hormone will most likely increase in the blood of
the student?
A. Epinephrine
B. Corticotropic
C. Thyrotropin-releasing hormone
D. Cortisol
20
E. Somatotropin

211.The increased intraocular tension is observed in the patient with glaucoma. Secretion of aqueous humor by the ciliar body is
normal. Injury of what structure of the eyeball wall caused the disorder of fluid flow-out from the anterior chamber.
A. Venous sinus
B. Back epithelium of cornea
C. Choroid
D. Ciliar body
E. Ciliary muscle

21
1
Basic tissues Моdul 2 Primordial germ cells and erythroblasts (megaloblasts) are
located in its wall. What organ is this?
1.After two months of appendectomy the patient again has A. Yolk sac
arrived in surgical department with the diagnosis: «The B. Allantois
intestinal obstruction, an adhesive disease». During operation C. Placenta
damage of what epithelium covering a peritoneum has D. Umbilical cord
reduced in development of adhesive disease? E. Amnion
A. Mesothelium
B. Endothelium 8.The primary function of brown adipose tissue is
C. Transitional epithelium A. To produce heat
D. Simple columnar epithelium B. To store unilocular energy
E. Simple cuboidal epithelium C. To produce hormons
D. To mobilize lipid for export as fatty acide
2.Differentiation of cells within some type of hematopoiesis E. To initiate the shivering-induced mobilization of lipids
is accompanied by a nucleus extrusion. What cell type is
characterized by such morphologic changes? 9. After birth, growth in the length of long bones occurs
A. Orthochromatophilic erythroblast (normoblast) primarily through
B. Polychromatophilic erythroblast A. Interstitial growth of cartilage cells in the epiphysial
C. Basophilic erythroblast plate
D. Monoblast B. Increased bone deposition under the periosteum
E. Megakaryoblast C. The action of osteoblasts in the primary ossification center
D. The action of osteoblasts in the secondary ossification
3.Blood analysis of a healthy adult man revealed 3% of some center
blood formed elements. What formed elements have this E. Appositional grows from the periphery
characteristic?
A. Eosinophils, band neutrophils 10. The microscopic examination of wound lavage of a
B. Band neutrophils, lymphocytes patient with acute woundy process of his shin revealed big
C. Eosinophils, lymphocytes contents of irregular extended-formed cells, with tough
D. Eosinophils, mature neutrophils nucleus, the basophilic cytoplasm of which includes many
E. Lymphocytes, monocytes Lysosomes, phagosomes and pinocytotic bubbles. What cells
are found out in the wound?
4. A patient with leukosis has disturbance of some type of A. Connective tissue macrophages
hematopoiesis. This type has next characteristic: antigen B. Fibroblasts
independent and antigen dependent phases, blastic C. Plasmocytes
transformation, clonal expansion, recirculation. What type of D. Tissue basophils
hematopoiesis has enumerated features? E. Fibrocytes
A. Lymphocytopoiesis
B. Granulopoiesis
C. Erythropoiesis 11. After the radioactive exposure a patient has
D. Monopoiesis haematopoietic stem cells disorder. The regeneration of what
E.Thrombopoiesis cells of loose connective tissue will be damaged?
A. Macrophages
5. A patient suffers from epithelial tumor of the right B. Fibroblasts
extrapulmonary bronchus. What morphologic type of C. Adipocytes
epithelium gave rise to tumor? D. Pericytes
A. Simple columnar pseudostratified E. Pigment cells
B. Simple squamous
C. Simple cuboidal 12. What parameter of a blood analysis most probably
D. Stratified squamous nonkeratinized testifies the presence in an organism of an acute
E. Transitional inflammation?
A. Ascending an amount of neutrophilic leucocytes
6. Compression of umbilical cord took place. But blood B. Ascending an amount of reticulocytes
circulation between fetus and maternal organism happened C. Ascending an amount of thrombocytes
normally. What morphologic components of the umbilical D. Decrease of an amount of basophilous leucocytes
cord promoted to this? E. Decrease of an amount of monocytes
A. Mucuous tissue
B. Allantois residue 13. At study of a blood smear of the person with presence of
C. Tunic of umbilical arteries inflammatory process it is possible to see a great number of
D. Tunic of umbilical vein spherical cells with a segmented nucleus, acidophilic
E. Yolk sac residue cytoplasm and shallow pink-violet granules in the cytoplasm.
What blood cells are these?
7.There is connecting with the gut vesicle is visible on the A. Neutrophilic granulocytes
histologic micropreparation. It is an extraembryonic organ. B. Erythrocytes
C. Eosinophilic granulocytes
2
D. Basophilous granulocytes 20. At survey frequently sick child the poor amount of
E. Lymphocytes immunoglobulins has been detected. Which of the
enumerated cells of an immune system produce
14. In a blood smear of the person who suffers an allergy, it immunoglobulins?
is possible to see a great number of spherical cells with a A. Plasma cells
segmented nucleus, weakly basophilic cytoplasm and major B. T-helper cells
brightly pink granules in cytoplasm. What blood cells type is C. Macrophages
in the smear? D. Plasmablasts
A. Eosinophilic granulocytes E. T-suppressor cells
B. Erythrocytes
C. Neutrophilic granulocytes 21. In a patient with clinical signs of immunodeficiency the
D. Basophilous granulocytes number and functional activity of T and B-lymphocytes are
E. Lymphocytes not changed. Defect with dysfunction of antigen-presentation
to the immunocompetent cells was found during investigation
15. After a bite of a bee in 12 hours allergic appearances on a on the molecule level. Defect of what cells is the most
skin (the edema, an itch) have disappeared. It is known, that probable.
one of mediators of an allergy is histamine. What blood cells A. Macrophages, B-cells
have taken part in elimination of excess of histamine in a B. NK-cells
place of a bite? C. T-lymphocytes
A. Eosinophils D. Fibroblasts, T-lymphocytes, B-lymphocytes
B. Neutrophils E .0-lymphocytes
C. Basophils
D. Lymphocytes 22. Live vaccine is injected into the human body. Increasing
E. Monocytes activity of what cells of connective tissue can be expected?
A.Plasma cells and lymphocytes cells
16. Punctuate hemorrhage was found our in the patient after B.Fibroblasts and mast cells
application of a tourniquet. With dysfunction of what blood C.Adipose cells and adventitial
cells is it connected? D.Macrophages and fibroblasts
A. Platelets E. Pigment cells and pericytes
B. Monocytes
C. Eosinophils 23. The patient with complaints of рain in eyes, which have
D. Neutrophils arisen after duration stay of the patient in a field during dust
E. Lymphocytes burs, converted to the ophthalmologist. The doctor has
established the surface damages of a cornea epithelium. What
17. At survey of the patient the poor amount of cells will provide regeneration of the defective epithelium?
immunoglobulins has been detected. What from the A. Cells stratum basale
enumerated cells of an immune system produce B. Cells of a stratum corneum
immunoglobulins? C. Cells of a stratum granulosum
A. Plasma cells (plasmocytes) D. Cells of a stratum lucidum
B. Plasmablasts E. Superficial cells
C. Т-killers cells
D. T-helper cells 24. The inner envelope of the blood and lymph vessels lines
E. T-suppressor cells an epithelium. Name it.
A. An endothelium
18. It is known, that plasma cells produce specific antibodies B. Stratified squamous epithelium
on the given antigen. At injection of an antigen the amount of C. Simple squamous epithelium
plasmocytes increases. Due to what blood cells the D. Epidermis
augmentation of number of plasmocytes is carried out. E. Transitional epithelium
A.B-cells
B. Eosinophils 25. The patient with complaints to a pain in a thoracic cavity
C. Neutrophils at respiration converted to the doctor. After survey there were
D. Basophils detected modifications of an epithelium of a pleura. What
E. T lymphocytes epithelium has tested modifications?
A. Simple squamous
19. At the patient after transplantation of heterologous renal B. Simple columnar ciliated
transplant the response of a casting-off has developed. What C. Simple cuboidal
basic effector cells participate in the given immunological D. Simple columnar
response? E. Stratified columnar
A. Cytotoxic T lymphocytes
B. T-helper cells 26. Under action of radiation cells of a basal layer of an
C. T-suppressor cells epidermis have suffered. What function of the last will
D. Plasmocytes weaken or will be breaked first?
E. В-plasmocytes A. Regenerative
B. Absorptive
3
C. Protective
D. Barrier 33. At a wound repair on the defective field of penetrating
E. Secretory layers of a skin occurs granular tissue. What cells are
discovered after wound and produce granular tissue?
27. In experiment the significant amount of stem cells of A. Myofibroblasts
bone marrow definitely eroded. Regeneration of what B. Fibrocytes
population of cells in composition of loose connective tissue C. Plasma cells
will be inhibited? D. Macrophages
A. Macrophages E. Adipose cells
B. Fibroblasts
C. Pigment cells 34. After wound of an extremity the woman had bleeding
D. Adipocytes which is accompanied by time-lagged coagulation of a blood
E. Pericytes (at a normal amount of platelets in analysis of a blood). What
substance of loose connective tissue hinders with process of
28. The initial state of an inflammation - alterative is blood coagulation? What cell is it produced by?
characterized by dilating of blood capillaries on a field of A. Heparin, mast cells
damage, decrease of a circulation, a raise of a permeability of B. Histamin, mast cells
a wall of vessels. To what from the cells given below, the C. Heparin, macrophage
leading role in inflammatory response is necessary? D. Heparin, plasma cells
A. To mast cells E. Heparin, fibroblast
B. To fibroblasts
C. To plasma cels 35. After a wound repair on its place the cicatrix has formed.
D. To eosinophils What substance is the basic ingredient of this type of a
E. To macrophages connective tissue?
A. Collagen
29. On a histology microslide in a loose connective tissue B. Keratan sulfate
big cells filled by basophilic metachromatic granules are C. Elastin
found. Histochemically it fixed, that granules contain heparin D. Hyaluronic acid
and histamin. What cells are most probably found in a E. Chondroitin sulfate
microslide?
A. Mast cells 36. At an endochondral bone formation of tubular bones
B. Fibroblasts between epiphysis and diaphysis centers of an ossification
C. Plasma cells form a plate, which further ensures body height of bones in
D. Macrophages length. How this plate is named?
E. Аdipose cells A. Epiphyseal plate
B. Bone lamellae
30. After the transferred chemical injury of an esophagus C. Subperiosteal bone collar
there has occurred its local narrowing owing to formation of D. An osteon
collagen cicatrix. What cells of a loose connective tissue E. Inner circumferential lamellar
share in formation of cicatrices?
A. The mature active fibroblasts 37. Owing to chondrodystrophia (anomaly of development of
B. Inactive fibroblasts a cartilage) has been damaged a fibrocartilage. In what organ
C. Fibrocytes is possible to observe pathological modifications?
D. Мyofibroblasts A. In intervertebral disks
E. Adipose cells B. In a trachea
C. In a larynx
31. At injection to the patient of a proteinaceous drug the D. On articulate surfaces
amount of plasma cells, which produce specific antibodies on E. In an auricle
the given antigen, increases. Due to what blood cells there is
an augmentation of an amount of plasma cells? 38. At the patient with a permanent injury of the upper
A. B cells extremity breaking processes of regeneration of a cartilage
B. T-suppressor cells owing to damage of chondrogenic cells is observed. What
C. Т-killers cells have tested damage?
D. T-helper cells A. Cells of inner cellular layer of perichondrium
E. В-memory cells B. Cells of an upper layer of perichondrium
C. Cells in isogenous droups
32. The leading part in shaping a vascular phase of an D. Cells of region of a young cartilage
inflammation is played with histamine. What cell of loose E. Cells which enter from blood vessels
connective tissue produce histamine?
A. Mast cell 39. It is offered to the student two microslides. On first - an
B. A fibroblast elastic cartilage (stained by orsein), on second - hyaline
C. A plasma cells (stained by hematoxylin - eosine). To what features of them it
D. A fibrocyte is possible to distinguish.
E. A macrophage A. On presence of elastic fibers
4
B. On presence of isogenic groups of cells 46. An experimental animal was undergone by overdoses of
C. On presence of region a young cartilage X-rays. Due to this total destruction of CFU-GM cells in
D. On presence of perichondrium bone marrow took place. Further development of what blood
E. On presence of an extracellular matrix formed elements will be happen in this case normally?
A. Erythrocytes
40. At people of advanced age it is observed exuberant B. Monocytes
losses of mass of an osteal tissue that reflects development of C. Eosinophils
an osteoporosis. Activation of what cells of bone tissue D. Basophils
causes development of the given disease ? E. Neutrophils
A. Osteoclasts
B. Osteoblasts 47. Differentiation of cells within some type of
C. Macrophages hematopoiesis is accompanied by a nucleus extrusion. What
D. Mast cells cell type is characterized by such morphologic changes?
E. Osteocytes A. Orthochromatophilic erythroblast (normoblast)
B. Polychromatophilic erythroblast
41. At the laboratory experiment the leukocyte culture was C. Basophilic erythroblast
mixed with staphylococci. Neutrophil leukocytes engulfed D. Monoblast
and digested bacterial cells. This process are termed: E. Megakaryoblast
A. Phagocytosis
B. Facilitated diffusion 48. Blood analysis of a healthy adult man revealed 3% of
C. Pinocytosis some blood formed elements. What formed elements have
D. Osmosis this characteristic?
E. Diffusion A. Eosinophils, band neutrophils
B. Band neutrophils, lymphocytes
42. In a patient after transplantation of the foreign kidney C. Eosinophils, lymphocytes
rejection of the transplated organ developed. What main D. Eosinophils, mature neutrophils
effecter cells participate in this reaction at first? E. Lymphocytes, monocytes
A.T-killers (T-cytotoxic)
B. plasma cells 49.After invasion of bacteria into wounded surface of skin
C. T-lymphocytes –supressors they were destroyed by receptor mediated phagocytosis.
D. B-cells What formed blood elements participate in this protective
E Fibroblasts reaction?
A. Neutrophils
43.Examination of a patient who was subjected to X-ray B. Basophils
irradiation revealed injury of the splenic white pulp. What C. Eosinophils
cells of the white pulp undergo to this pathological change at D. Lymphocytes
first? E. Platelets
A. Lymphocytes
B. Basophils 50. After overdoses of -rays destruction of B-lymphocytes
C. Neutrophils in human hematopoietic organs happens. Reduction of what
D. Mast cells blood plasma substances will be observes in this case.
E. Red blood cells A. Immunoglobulins (antibodies)
B. Albumins
44. The child’s scratch, which had been caused by light C. Cholesterol
skin’s trauma, disappeared after 10 days. What skin layer D. Glucose
provided for reparative regeneration? E. Fibrinogen
A. Stratum basale;
B. Stratum spinosum; 51. A patient with leucosis has disturbance of some type of
C. Stratum granulosum; hematopoiesis. This type has next characteristic: antigen
D. Stratum corneum; independent and antigen dependent phases, blastic
E. Stratum brilliantum. transformation, clonal expansion, recirculation. What type of
hematopoiesis has enumerated features?
45. After two months of appendectomy the patient again has A. Lymphocytopoiesis
arrived in surgical department with the diagnosis: «The B. Granulopoiesis
intestinal obstruction, an adhesive disease». During C. Erythropoiesis
operation, damage of what epithelium covering a peritoneum D. Monopoiesis
has reduced in development of adhesive disease. E. Thrombopoiesis
A. Mesothelium
B. Endothelium 52.A patient suffers from epithelial tumor of the right
C. Transitional epithelium extrapulmonary bronchus. What morphologic type of
D. Simple columnar epithelium epithelium gave rise to tumor?
E. Simple cuboidal epithelium A. Simple columnar pseudostratified
B. Simple squamous
C. Simple cuboidal
5
D. Stratified squamous nonkeratinized G. Fibroblasts
E. Transitional H. Adipose cells
I. Pericytes
53.Compression of umbilical cord took place. However, J. Pigment cells
blood circulation between fetus and maternal organism
happened normally. What morphologic components of the
umbilical cord promoted to this? 60. The neonatal child has a disorder of a thymus gland.
A. Mucous tissue What type of a hemopoiesis will be disturbed?
B. Allantois residue 1. Lymphopoiesis
C. Tunic of umbilical arteries 2. Monocytopoiesis
D. Tunic of umbilical vein 3. An erythrogenesis
Е Yolk sac residue 4. Granulocytopoiesis
5. Platelet formation
54. The primary function of brown adipose tissue is
A. To produce heat 61. An experimental animal was undergone by overdoses of
B. To store unilocular energy X-rays. Due to this total destruction of CFU-GM cells in
C. To produce hormones bone marrow took place. Further development of what blood
D. To mobilize lipid for export as fatty acids formed elements will be happening in this case normally.
E. To initiate the shivering-induced mobilization of lipids 1. Erythrocytes
2. Monocytes
55. The extra cellular matrix and the cytoskeleton 3. Eosinophils
communicate across the cell membrane through 4. Basophils
A. Integrins 5. Neutrophils
B. Proteoglycans
C. Microtubules 62. Differentiation of cells within some type of
D. Hyaluronic acid hematopoiesis is accompanied by a nucleus extrusion. What
E. Microfilaments cell type is characterized by such morphologic changes?
1 .Orthochromatophilic erythroblast (normoblast)
56. After birth, growth in the length of long bones occurs 2. Polychromatophilic erythroblast
primarily through 3 .Basophilic erythroblast
A. Interstitial growth of cartilage cells in the epiphysial 4. Monoblast
plate 5. Megakaryoblast
B. Increased bone deposition under the periosteum
C. The action of osteoblasts in the primary ossification center 63.Blood analysis of a healthy adult man revealed 3% of
D. The action of osteoblasts in the secondary ossification some blood formed elements. What formed elements have
center this characteristic?
E. Appositional grows from the periphery 1. Eosinophils, band neutrophils
2. Band neutrophils, lymphocytes
57. The microscopic examination of wound lavage of a 3. Eosinophils, lymphocytes
patient with acute wound process of his shin revealed big 4. Eosinophils, mature neutrophils
contents of irregular extended-formed cells, with tough 5. Lymphocytes, monocytes
nucleus, the basophilic cytoplasm of which includes many
Lysosomes, phagosomes and pinocytotic bubbles. What cells 64. After invasion of bacteria into wounded surface of skin
are found out in the wound? they were destroyed by receptor mediated phagocytosis.
F. Connective tissue macrophages What formed blood elements participate in this protective
G. Fibroblasts reaction?
H. Plasma cells 1. Neutrophils
I. Mast cells 2. Basophils
J. Fibrocytes 3. Eosinophils
4. Lymphocytes
58.Blood sampling for bulk analysis is recommended to be 5. Platelets
performed on an empty stomach and in the morning. What
changes in blood count can occur if to perform blood
sampling after food intake? 65. A patient complains of pain in the abdomen. It is
A. Increased contents of leukocytes determined that the pain was a result of large substances
B. Reduced contents of erythrocytes passing between cells that line the intestinal lumen directly
C. Increased contents of erythrocytes into the underlying connective tissue. Which of the following
D. Reduced contents of thrombocytes types of junctions were absent or not completely functional?
E. Increased plasma proteins A. Zonulae occludens
B. Maculae adherens
59.After the radioactive exposure, a patient has C. Zonulae adherens
haematopoietic stem cells disorder. The regeneration of what D. Gap junctions
cells of loose connective tissue will be damaged? E. Hemidesmosome
F. Macrophages
6
66. Junctions are essential in maintaining the close 73. Macrophages would be the most abundant in which
association of the cells within the epidermis. Which of the of the following tissues?
following junctions in maintaining cell adherence in all layers a. Loose connective tissue
of the epidermis? b. Dense regular connective tissue
a. Maculae adherens c.Brown adipose tissue
b. Gap junctions d. Dense irregular connective tissue
c. Hemidesmosome e.Embryonic tissue
d. Zonulae adherens
e. Zonulae occludens 74. What type of epithelium normally lines the lumen of
the trachea?
67. Which of the following proteins aids in maintaining a. Pseudostratified ciliated columnar
the close association of the cells of the epidermis of the skin epithelium
a. Intermediate filaments b. Stratified columnar epithelium
b. Occludins c.Transitional epithelium
c. Cadherins d. Simple squamous epithelium
d. Vinculin e.Simple columnar epithelium
e. Collagen
75. A pathologist is examining a tissue section with an
68. Simple squamous epithelium is prevalent throughout electron microscope. He notices immune cells in connective
the body and organ systems, which of the following is a tissue that have a prominent Golgi apparatus and a nucleus
primary function of this type of epithelium. with heterochromatin arranged in a spoke wheel fashion.
a. Lubrication Which of the following cell types is being observed?
b. Secretion a.Plasma cell
c. Adsorption b.Macrophage
d. Protection c.Fibroblast
e. Excretion d.Mast cell
e.Brown adipose cell
69. A biopsy of the urinary bladder is done in a patient who
has complained of lower abdominal pain. In the pathologist’s 76. Which of the following provides a primary structural
report, the luminal epithelium is described as normal. What network in hematopoietic organs?
type of epithelium did the pathologist observe? a. Reticular connective tissue
a. Transitional epithelium Stratified squamous b. White adipose tissue
epithelium c.Brown adipose tissue
b. Simple squamous nonkeratinized epithelium d. Pigment connective tissue
c. Pseudostratified ciliated columnar epithelium e.Elastic connective tissue
d. Simple columnar epithelium
77. Your patient suffered a complete tear of biceps brachial
70. Microvilli are essential components of epithelial cells of muscle at its insertion site at the radial tuberosity. For proper
the small intestine. Which of the following functions would treatment, you must identify the damaged tissue. Which of
be defective resulting from a lack of microvilli on epithelia? the following tissues was damaged at the tear site?
a. Absorption A Dense regular connective tissue
b. Stretching b.Reticular tissue
c. Movement c.Elastic tissue
d. Protection d.Loose areolar connective tissue
e. Secretion e.Dense irregular connective tissue

71. The layer of cells that lines the outer surface of the lungs 78. Your patient suffers from an immediate
can be irritated by inhaled asbestos particles. These cells can hypersensitivity (allergy) reaction. Which of the following
become cancerous and lethal. Which of the following is the cell types is responsible for this condition?
cell layer being described? a. Mast cell
a. Simple squamous mesothelium b. Plasma cell
b. Simple squamous endothelium c.Fibroblast
c. Transitional epithelium d. Macrophage
d. Simple cuboidal epithelium e.Adipose cell
e. Simple columnar epithelium
79. Which of the following types of collagen is the most
72. Which of the following is a characteristic typical of widely distributed and abundant within the body?
simple layers of epithelia? a. Type I collagen
a. All the cells rest on a basal lamina b. Type II collagen
b. All the cells cover surface of the skin c.Type III collagen
c.All the cells function to prevent abrasion d. Type IV collagen
d. All the cells are joined by synapses e.Type V collagen
e.All the cells have flagella
7
80. Immunocytochemistry is performed on a tissue c. Tracheal rings
section using an antibody specific for type IV collagen. The d. Cartilage at pubic symphysis
investigator notes staining within an organ that contains e. Costal cartilage
abundant and varied connective tissue. Which of the
following would be identified by the antibody to type IV 88. Which of the following organelles plays an essential
collagen? role in the sulfation of glycosaminoglycans within
a. Basal lamina chondrocytes?
b. Reticular fibers a. Golgi apparatus
c. Elastic fibers b. Polyribosomes
d. Loose connective tissue c. Smooth endoplasmic reticulum
e. Adipose cells d. Rough endoplasmic reticulum
e. Ribosomes associated with the nuclear
81. Which of the following microscopically best envelope
characterizes brown fat cells?
a. Multiple droplets of lipid 89. Within compact bone, which of the following form
b. Lack of mitochondria connections between Haversian systems?
c. Peripheral flattened nucleus a. Volkmann’s canals
d. Abundant RER b. Howship’s lacuna
e. Poor blood supply. c. Canaliculi
d. Lacunae
82. Which of the following provide the nutrients to e. Cement lines
mature chondrocytes?
a. Diffusion through the matrix 90. Cessation of growth at which of the following sites
b. Vasculature of the cartilage would result in no further longitudinal growth of long bones.
c. Canals within the cartilage a. Epiphyseal plate
d. Capillary network within the matrix b. Endosteum
e. Cellular interconnections between adjacent c. Primary center of ossification
chondrocytes d. Secondary center of ossification
e. Diaphysis
83. Which of the following would best characterize
fibrocartillage? 91. A blood analysis of your patient shows an increased
a. Rows on chondrocytes level of parathyroid hormone. Which of the following would
b. Presence of a perichodrium result from increased levels of this hormone?
c. Found at the epiphyseal plate a. Increased activity of osteoclasts and bone
d. Large, densely stained territorial matrix resorbtion
e. Lines bones at articular sites b. Increased mitotic activity of osteoprogenitor
84. Which of the following is the major component of cells
the matrix of the cartilage? c. Decreased serum calcium levels
a. Collagen d. Increased bone formation
b. Isogenous groups e. Increased hydroxyapatite crystal formation
c. Lacunas
d. Chondrocytic processes 92. Which of the following could be administered to
e. Perichondrium override the effects of increased parathyroid hormone?
a. Calcitonin
85. Chondroblasts are derived from which of the b. Vitamin D
following cell types? c. Bone morphogenic protein
a. Mesenchymal cells d. Osterprotegerin
b. ibroblasts e. Somatrophin
c. Proliferating chondrocytes
d. Cells within isogenous groups 93. Osteoclasts would be found at which of the
e. Cells of the perichondrium following sites in bone?
a. Howship’s lacuna
86. Which of the following best characterizes hyaline b. Haversian canal
cartilage? c. Canaliculi
a. Abundant matrix d. Endosteum
b. No perichondrium e. Periosteum
c. Single chondrocytes
d. Found between vertebral bodies 94. Bone of a 75-year-old man shows a decreased
e. Least resilient of all forms of cartilage number of osteoblasts. However, the number of osteoclasts is
greatly increased. This individual’s bones are easily
87. Elastic cartilage is found in which of the following fractured. Which of the following disease conditions would
body structures? you expect?
a. Epiglottis a. Osteoporosis
b. Intervertebral discs b. Osteoartrosis
8
c. Osteopetrosis C. In half of A-band
d. Osteochondrosis D. In A band and in I band
e. Rickets E. In half of I band

95. The hormone calcitonin acts directly inhibited on 102. At examination of a striated muscle fiber after treatment
which of the following cell types? of hydrolyses enzymes destruction of thin myofilaments is
a. Osteoclasts observed. What supermolecular structures have tested
b. Osteocytes damage?
c. Osteoblasts A. Actin myofilaments
d. Chondroblasts B. Myosin myofilaments
e. Chondrocytes C. Tonofibrils
D. Tropocollagin complexes
96. At which of the following sites in bone would you E. Nucleoprotein complexes
find osteoblasts during active deposition of new bone matrix?
a. Surface of bone 103. During an operative measure at the patient the part of a
b. Haversian canal small intestine is removed. Due to what elements
c. Lacuna regeneration of a muscular layer is possible?
d. Periosteum A. Smooth muscle cells
e. Canaliculi B. Мyosyncithium
C. Adipose cells
97. In which of the following bone sites are canaliculi D. Satellite cells
found? E. Fibrocytes
a. Compact bone
b. Perichondrium 104. Patient with injured muscles of the lower extremities
c. Bone marrow was admitted to the traumatological department. Due to what
d. Newly mineralized bone matrix cells is reparative regeneration of the muscle fibers and
e. Howship’s lacuna restoration of the muscle function possible?
A. Satellite-cells
SPECIAL TISSUE B. Myoepithelial cells
98.. Breaking architectonics in a somite region near to an C. Myoblasts
endoderm and notochord is found during biopsy examination D. Myofibroblasts
of embryonic material. Disorders of what tissues or organs E. Fibroblasts
will be?
A. Skeletal tissues 105. At the patient the tremor and convulsions that is caused
B. Urogenital system by breaking of nerve fibers myelination of a central nervous
C. Striated skeletal muscle system is observed. Function of what cells suffers at this
D. Striated cardiac muscle disease?
E. Fibrillar connective tissue of a skin A. Olygodendrocytes
B. Fibrillar astrocytes
99..In traumatological hospital, the patient with damage of C. Ependymal cells
muscles of the inferior extremity is supplied. Due to what D. Protoplasmatic astrocytes
cells reparation of muscle filaments and regeneration of E. Microglial cells
function of muscles is possible?
A. Satellite cells 106. In what cells during life the mitosis is not observed, and
B. Myoepithelial cells the quantitative content DNA remains to stationary values?
C. Myofibroblasts A. In neurones
D. Fibroblasts B. In hemopoietic
E. Myoblasts C. In striated muscle fiber
D. In smooth muscle
100.. At formation of a muscle tissue in an embryogenesis E. In an epidermis
process of fusion of myoblasts in myosyncithium is
quenched. What tissue development will be disturbed? 107 At patient with a chronic alcoholism disturbance of
A. A skeletal muscle tissue coordination of movements and equilibrium owing to
B. A muscle tissue derivate of neural tube morphologic disturbances in cerebellum. Damage of what
C. A muscle tissue derivate of ectoderm cerebellar cells we can suppose at a first head?
D. A smooth muscle tissue derivate of mesenchyme A. Purkinje cells
E. A cardiac muscle tissue B. Basket cells
C. Golgi cells
101. At examination of a striated muscle fiber after a D. Pyramidal cells
mechanical trauma destruction of thick myofilaments is E. Stellate cells
observed. Where pathological modifications if filaments will
be studied in polarized light will be localized? 108. Some nervous system organ is presented in histological
A. In A band section. It contains capsule, connective tissue layers that
B. In I band extend from the capsule. Bipolar (pseudounipolar) neurons
9
are located bellow capsule. Peryikaryons of this neurons are A. A skeletal muscle tissue
surrounded by glial cells. Center of organ contains neurons B. A muscle tissue derivate of neural tube
processes. What organ is it? C. A muscle tissue derivate of ectoderm
A. Dorsal root ganglion (spinal ganglion) D. A smooth muscle tissue derivate of mesenchyme
B. Vegetative ganglion E. A cardiac muscle tissue
C. Spinal cord
D. Cerebral cortex 115. At examination of a striated muscle fiber after a
E. Cerebellar cortex mechanical trauma destruction of thick myofilaments is
observed. Where pathological modifications if filaments will
109. Cerebral cortex contains large and giant pyramidal be studied in polarized light will be localized?
shaped neurons. It is a feature of the cerebral cortex. A. In A band
Discovery of this cells was connected with the name of B. In I band
following scientist: C. In half of A-band
A. Bets D. In A band and in I band
B. Purkinje E. In half of I band
C. Golgi
D. Nissle`s 116. At examination of a striated muscle fiber after treatment
E. Paneth`s of hydrolyses enzymes destruction of thin myofilaments is
observed. What super molecular structures have tested
110. In human embryo neural tube neuroepithelial ectodermic damage?
cells differentiate into neuroblasts and gliablasts. Owing to A. Actin myofilaments
migration of these cells some zones (layers) are formed in B. Myosin myofilaments
neural tube. Which of the following zone is where bodies of C. Tonofibrils
neuroblasts locate in? D. Tropocollagin complexes
A. Mantle (intermediate) zone E. Nucleoprotein complexes
B. Ventricular zone
C. Marginal zone 117. During an operative measure at the patient the part of a
D. Outer limiting membrane small intestine is removed. Due to what elements
E. Inner limiting membrane regeneration of a muscular layer is possible?
A. Smooth muscle cells
111. During road accident at the man of 30 years the femoral B. Мyosyncithium
artery has been injured. In hospital the arteriorraphy was C. Adipose cells
performed. Regeneration of what muscle tissue is necessary D. Satellite cells
for regeneration of a wholeness of the vessel`s tunica media? E. Fibrocytes
A. Smooth muscle tissue
B. Striated skeletal muscle tissue 118. Patient with injured muscles of the lower extremities
C. Striated cardiac muscle tissue was admitted to the traumatological department. Due to what
D. Myoepithelial cells cells is reparative regeneration of the muscle fibers and
E. Satellite cells restoration of the muscle function possible?
A. Satellite-cells
112. The patient with complaints of pain in eyes, which have B. Myoepithelial cells
arisen after duration stay of the patient in a field during dust C. Myoblasts
burs, converted to the ophthalmologist. The doctor has D. Myofibroblasts
established the surface damages of a cornea epithelium. What E. Fibroblasts
cells will provide regeneration of the defective epithelium?
A. Cells stratum basale 119. At the patient the tremor and convulsions that is caused
B. Cells of a stratum corneum by breaking of nerve fibers myelination of a central nervous
C. Cells of a stratum granulosum system is observed. Function of what cells suffers at this
D. Cells of a stratum lucidum disease?
E. Superficial cells A. Olygodendrocytes
B. Fibrillar astrocytes
113.In traumatological hospital, the patient with damage of C. Ependymal cells
muscles of the inferior extremity is supplied. Due to what D. Protoplasmatic astrocytes
cells reparation of muscle filaments and regeneration of E. Microglial cells
function of muscles is possible?
A. Satellite cells 120. In what cells during life the mitosis is not observed, and
B. Myoepithelial cells the quantitative content DNA remains to stationary values?
C. Myofibroblasts A. In neurones
D. Fibroblasts B. In hemopoietic
E. Myoblasts C. In striated muscle fiber
D. In smooth muscle
114. At formation of a muscle tissue in an embryogenesis E. In an epidermis
process of fusion of myoblasts in myosyncithium is
quenched. What tissue development will be disturbed?
10
121. At patient with a chronic alcoholism disturbance of 127. At a combustion of a skin the patient feels a strong pain.
coordination of movements and equilibrium owing to Processes of what neurons first conduct pain nervous impulse
morphologic disturbances in cerebellum. Damage of what in central nervous system?
Cerebellar cells we can suppose at a first head. A. Pseudounipolar neurons
A. Purkinje cells B. Bipolar neurons
B. Basket cells C. Unipolar neurons
C. Golgi cells D. Multipolar neurons
D. Pyramidal cells E. Purkinje neurons
E. Stellate cells
128. At a brain injury are defected glial cells, which meet in
122. Some nervous system organ is presented in histological gray matter to central nervous system more often. What is the
section. It contains capsule, connective tissue layers that name of these cells?
extend from the capsule. Bipolar (pseudounipolar) neurons A. Protoplasmic astrocytes
are located bellow capsule. Peryikaryons of these neurons are B. Oligodendrocytes
surrounded by glial cells. Center of organ contains neurons C. Microglia
processes. What organ is it? D. Fibrous astrocytes
A. Dorsal root ganglion (spinal ganglion) E. Ependymal cells
B. Vegetative ganglion
C. Spinal cord 129. In skeletal muscle fibers, which of the following
D. Cerebral cortex would exhibit ATPase activity?
E. Cerebellar cortex a. Myosin filaments
b. Actin filaments
123. Cerebral cortex contains large and giant pyramidal c. Troponin
shaped neurons. It is a feature of the cerebral cortex. d. T-tubule system
Discovery of this cells was connected with the name of e. Tropomyosin
following scientist:
A. Bets 130. Dense bodies within smooth muscle fibers most closely
B. Purkinje associate with which of the following components of skeletal
C. Golgi muscle?
D. Nissle`s a. Z line
E. Paneth`s b. A band
c. I band
124. In human embryo neural tube neuroepithelial ectodermal d. H band
cells differentiate into neuroblasts and gliablasts. Owing to e. M line
migration of these cells some zones (layers) are formed in
neural tube. Which of the following zone is where bodies of 131.Which of the following best characterizes a cardiac
neuroblasts locate in? muscle fiber?
A. Mantle (intermediate) zone a. Intercalated disks
B. Ventricular zone b. Multiple nuclei
C. Marginal zone c. Lack of striations
D. Outer limiting membrane d .Spindle shaped fiber
E. Inner limiting membrane e. Myoneural junctions

125. Myocardiodistrophy promotes a breach of cardiac 132. Which of the following events occur during the
muscle cells metabolism. Which of the following embryonic process of contraction of skeletal muscle?
origin breach causes this pathology? a. Release of calcium ions by sarcoplasmic
A. Myoepicardial plate reticulum
B. Myotome b. Calcium ions bind to tropomyosin
C. Endoderm c. Actin filaments shorten
D. Ectoderm d. Z line disappears
E. Mesenchyme e. Sarcomeres lengthen

126. Under the influence of long vibration and a motion in 133. Which of the following best characterizes smooth
motor regions of cortex brain occurs a gliolis – phagocytosis muscle fibers?
of damaged neurons by neuroglial cells. What neuroglia a. Caveolae as the calcium ion binding vesicals
participates in this process? b. Regularly arranged actin and myosin myofilaments
A. Microglia cells c. Multiple, peripheral nuclei
B. Protoplasmatic astrocytes d. Extensive myoneural junctions
C. Oligodendrocytes e. Extensive sarcoplasmic reticulum
D. Ependymal cells
E. Fibrous astrocytes 134. Which of the following regions contains the Z line
in skeletal muscle?
a. I band
b. H band
11
c. A band
d. M band
e. At the junction of the A band and I band

135. Which of the following molecules is found within


the groove created by actin filaments?
a. Tropomyosin
b. Troponin I
c. Myosin
d. Dystrophin
e. Alpha Actinin

136. What is the function of the T tubule system in


skeletal muscle?
a. Regulates Ca2+ release from sarcoplasmic reticulum
b. Stores Ca2+
c. Provides channel for Ca2+ movement with muscle
d. Wraps individual muscle fibers to form a fascicle
e. Site of attachment of actin filaments
Embriology2

[1]
Fetal sex can be diagnosed by noting the presence or absence of the Barr body in cells obtained from the amniotic
fluid. What is the etiology of the Barr body?
1. Inactivation of one X chromosome
2. Inactivation of both X chromosomes
3. Inactivation of homologous chromosomes
4. Inactivation of one Y chromosome
5. Inactivation of one chromatid

2]
When does a secondary oocyte complete its second meiotic division to become a mature ovum?
1. at fertilization
2. at ovulation
3. before ovulation
4. puberty
5. before birth

3]
How soon after fertilization occurs within the uterine tube does the blastocyst begin implantation?
1. by 6-7 day
2. by 12 hours
3. by day 1
4. by day 2
5. 7within minutes

[4]
Where does the blastocyst normally implant?
1. functional layer of the endometrium
2. functional layer of the cervix
3. basal layer of the endometrium
4. myometrium
5. perimetrium

[5]
All of the following structures are necessary for blastocyts implantation EXCEPT?
1. zona pellucida
2. endometrium in progestational phase
3. syncytiotrophoblast
4. cytotrophoblast
5. functional layer of the endometrium

[6]
Which of the following components plays the most active role in invading the endometrium during blastocyst
implantation?
1. syncytiotrophoblast
2. epiblast
3. hypoblast
4. extraembryonic somatic mesoderm
5. extraembryonic visceral mesoderm

[7]
Between which two layers is the extraembryonic mesoderm located?
1. exocoelomic membrane and cytotrophoblast
2. syncytiotrophoblast and cytotrophoblast
3. syncytiotrophoblast and endometrium
4. exocoelomic membrane and syncytiotrophoblast
5. epiblast and hypoblast

1.

[8]
Which germ layers are present at the end of week 3 of development (day 21)?
1. ectoderm, mesoderm, and endoderm
2. epiblast and hypoblast
3. ectoderm and endoderm
4. epiblast only
5. epiblast, mesoderm, and hypoblast

[9]
Which process establishes the three definitive germ layers?
1. gastrulation
2. neurulation
3. craniocaudal folding
4. lateral folding
5. angiogenesis

[10]
The first indication of gastrulation in the embryo is
1. formation of the epiblast end hypoblast
2. formation of the notochord
3. formation of the neural tube
4. formation of extraembryonic mesoderm
5. formation of tertiary chorionic villi

[11]
Somites may differentiate into all of the following EXCEPT
1. epidermis of skin
2. cartilage
3. muscle
4. bone
5. dermis of skin

[12]
Intermediate mesoderm will gives rise to the
1. kidneys and gonads
2. heart
3. neural tube
4. somites
5. notochord

[13]
The developing embryo has a distinct human appearance end named fetus by the end of:
1. week 8
2. week 5
3. week 6
4. week 7
5. week 4

[14]
The lateral mesoderm is divided into two distinct layers by the formation of the
1. intraembrionic coelom
2. extraembrionic coelom
3. cardiogenic region
4. notochord
5. yolk sac

15]
On microslide of the bulb of the fetus eye a damage of a corneal epithelium is observed. Part of what germ layer
has been struck during embryonic development
1. Ectoderm
2.Mesoderm
3. Endoderm
4.Dermatome
5.Nephrotome

[16]
During an embryogenesis there was a breaking shaping of a pharyngeal gut. Specify possible localization of
anomalies of development?
1.Organs of an oral cavity.
2.A stomach
3.An ileum
4.A pancreas
5.A jejunum

[17]
A metaphase-blocking dose of colchicines works by which of the following mechanisms
1.Inhibition of tubulin polymerization
2.Depolymerization of myosin
3.Enhancement of tubulin polymerization
4.Depolymerization of actin
5.Binding to and stabilizing microtubules

[18]
Destruction of neural crest took place in embryo after bacterial toxins influence. What element development will be
absent in this case?
1. C-cells, pigment cells, suprarenal medulla
2.Myocardium, epicardium
3.Kidney, ureter
4. Skin dermis
5.Gastric and intestinal epithelium

[19]
The prechordal plate is origin of development of the future epithelium of
1. esophagus
2.heart
3. umbilical cord
4.anus
5.nose

[20]
For what purpose can be used the definition of Barr body?
1.to identify the sex of a fetus
2.to definition a certain genetic diseases
3.to definition the age of a fetus

[21].
As a result of blocking separate ingredients of a genome, a cell gain characteristic for them morphological,
biochemical and function singularities. What title has this process?
1. Differentiation
2. Capacitation
3. Reception
4. Adhesion
5. Determination
Цитология 4.Golgi complex
[1]
What system of lenses of a light microscope [8]
collects and focuses the Illumination to produce a All the following organelles incorporate unit
cone of light that illuminates the object to be membrane EXEPT:
observed? *1. Ribosome
*1. Condenser 2. Lysosomes
2. Objective 3.Golgi complex
3. Ocular 4. Rough endoplasmic reticulum

[2] [9]
What maximal resolving power of the light Differentiated adult human cells incorporate
microscope can be? following structures EXEPT:
*1.0,2 micrometers *1. sarcomeres
2.0,1 nm 2. Endoplasmic reticulum
3.1 mm 3. Plasma membranes
4.Golgi complex
[3]
What type of microscopy include the using of [10]
microspectrophotometer? What is the total thickness of the plasma
*1. fluorescence membrane?
2.polarizing *1.8-10 nm
3. electron 2.2-5 mm
4.ultraviolet 3.1-2 nm

[4] [11]
What substance uses for clearing? What organelles contain oxidative enzymes,
*1.xylene particularly - catalase?
2.ethanol *1. Peroxisomes
3.eosin 2. Ribosomes
4.hematoxylin 3. Lysosomes
4.Golgi complex
[5]
Enumerate acidophilic components of tissues: [12]
*1. mitochondria, secretory granules, collagen What inclusions the organic solvents used to
2.microtubules, centrioles, basal body prepare tissues for both light and electron
3. nucleoproteins, glycosaminoglycans, microscopy can extract?
glycoproteins *1.lipid
2.glycogen
[6] 3.pigment
By using of Feulgen reaction can be identified:
*1. DNA [13]
2. histones In what phase of cell cycle synthesis and
3.lipids replication of DNA take place?
4.RNA *1. S
2.M
[7] 3. Go
Which one of the following subcellular 4.G1
components has the largest single dimension? 5.G2
*1.microtubule
2.lysosome [14]
3.centriole In what phase of mitosis reappears the nucleolus?
1.telophase [22]
2.prophase Oxidative phosphorylation to adenosine
3.anaphase triphosphate in mitochondria is associated with
*1. Cristae
[15] 2. Outer mitochondrial membrane
What is the diameter of nuclear pore? 3. Matrix
*1.70 nm 4. non of the above
2.70 mm
3.14 mm [23]
The mitotic spindle is composed mostly of
[16] *1. Microtubules
For what purpose can be used the definition of 2. Smooth-surfaced endoplasmic reticulum
Barr body? 3. Golgi flattened sac
*1.to identify the sex of a human 4.none of the above
2.to definition a certain genetic diseases
3.to definition the age of a human [24]
The fluid-mosaic model of the membrane
[17] structure
What is the function of a smooth endoplasmic *1. has hydrophilic elements directed away from
reticulum? the center
*1.lipid absorption and metabolism 2.has hydrophobic elements directed away from
2.protein secretion the center
3.dehydrating function 3. contains actin filaments
4.none of the above
[18]
What cells in such a stage of development is [25]
referred to as being in a Go-phase? During anaphase
*1. Neurons *1. the chromatids separate, moving away from
2. Epithelial cells each other
3. Connective tissue cells 2.the chromosomes become visible and condense
3. the chromosomes uncoil becoming indistinct
[19] 4.none of the above
What is the principal function of the rough
endoplasmic reticulum? [26]
*1.Protein synthesis All of the following are considered membranous
2.Steroid synthesis organelles except
3.Side of intracellular digestion *
1. centrioles
[20] 2. lysosomes
What organelle contains the DNA? 3.mitochondria
*1. Mitochondria 4.Golgi apparatus
2. Ribosomes 5.peroxysomes
3.Golgi complex
[27]
[21] The pars granulosa and pars fibrosa refer to
DNA is replicated during specific portions of the
*1. S- period of interphase *1. nucleolus
2.G1 -period of interphase 2. nucleus
3. Prophase 3.nuclear envelop
4.G2 -period of interphase 4.rough endoplasmatic reticulum
5.metaphase 5.smooth endoplasmatic reticulum
[28] To hospital converted the man with genetical set
The serous cells form the secretory potions of a of Kartenger’s symptoms, in which locomotion of
parotid gland. Which of the following organelles cilia and flagella absent. What are molecular
provide synthesis and secretion of salivary mechanisms of this pathology?
enzymes? *1.Lack of dynein
*1. Rough endoplasmic reticulum, Golgi apparatus 2.An excess of dynein
2.Smooth endoplasmic reticulum 3.Lack of tubulin
3.Lysosomes 4.A deficit of spectrins
4.Endosomes 5.A deficit of hyaluronidase
5. Mitochondria, Golgi apparatus
[33]
[29] Moving of the daughter chromatids to the poles
A solution of amino acid leucine with a radioactive of the cell is observed in the mitotically dividing
label has been added to a culture medium where cell. On what stage of the mitotic cycle is this
animal cells are growing. A high concentration of cell?
marked amino acid near some organelles has been *1.Anaphase
found by a method of radioautography. These 2.Interphase
organelles can be: 3.Metaphase
*1.Ribosomes 4.Prophase
2.Lysosomes 5.Telophase
3.Smooth endoplasmic reticulum
4.Cell center [34]
5.Golgi apparatus The cell of the laboratory animal was overdosed
with Roentgen rays. As a result, albuminous
[30] fragments formed in the cytoplasm. What cell
At reversion to the doctor the patient complained organelles will take part at their utilization?
of the strongest girdle pains at a level of an *1.Lysosomes
epigastrium, temperature, nausea and vomiting. He 2.Cells center
is hospitalized with the diagnosis - a pancreatitis. 3.Ribosome
What cell structures at destruction of the 4.Endoplasmic reticulum
membrane are capable to cause an autolysis and 5.Golgi complex
fusion of cells and tissues?
*1.Lysosomes [35]
2.Ribosomes As a result of blocking separate ingredients of a
3.Smooth endoplasmic reticulum genome, a cell gain characteristic for them
4.Cell center morphological, biochemical and function
5.Golgi apparatus singularities. What title has this process?
*1.Differentiation
[31] 2.Capacitation
A cell laboratory animal have subjected to an 3.Reception
exuberant X-ray irradiating. In outcome 4.Adhesion
roteinaceous fragments in cytoplasm were formed. 5.Determination
What organelles of a cell will take part in their
salvaging? [36]
*1.Lysosomes During the postsynthetic period of mitotic cycle
2.Golgi apparatus the synthesis of proteins – tubulines, which take
3.Endoplasmic reticulum part in the mitosis formation, was destroyed. It
4.Cell center can cause the impairment of:
5.Ribosomes *1.Duration of mitosis
2.Chromosome spiralization
[32] 3.Chromosome despiralization
4.Chromosome condensation
5.Cytokinesis 4.Smooth endoplasmic reticulum
5.Ribosomes
[37]
The study of mitotic cycle phases of onion root [42]
revealed the cell, in which the chromosomes are A small tumor was excised from the adrenal
situated in the equatorial plate, forming a star. gland of an adult male. After examination, the
What stage of the cell mitosis is it? cells of the tumor were shown to express
1. *Metaphase excessively high levels of lipid and steroid
2. Interphase hormones. Which of the following organelles
3. Prophase were abundant within these tumor cells?
4. Anaphase *1.Smooth endoplasmic reticulum
5. Telophase 2.Rough endoplasmic reticulum
3.Ribosomes
[38] 4.Lysosomes
Which of the following components of the 5.Peroxisomes
mitochondria would be most abundant in those
cells that have high ATP production? [43]
1. *Cristae A tumor was removed from the liver of a patient,
2. Inner membrane space and cell cultures of tumor cells were established.
3. Matrix Before chromosomal analysis of these cells,
4. Outer mitochondrial membrane colchicines was added to a culture. This alkaloid
5. Dense granules prevents the polymerization of microtubules.
These cancerous cells would be arrested at which
[39] of the following stages of mitosis?
Which of the following events occurs during 1. *Metaphase
anaphase of mitosis? 2. Anaphase
*1.Chromatids separate 3. Interphase
2.Chromosomes coil 4. Telophase
3.Nuclear envelope reforms 5. Prophase
4.Chromosomes uncoil
5.Nuclear envelope begins to break down [44]
A pathologist is studying a tissue section of the
[40] cerebrum by light microscopy. He notes a
After examining a tissue from a biopsy specimen, structure within a neuron that has both a granular
a pathologist notes that a large percentage of the and a fibrous component. Which of the following
cells have nuclei with abundant nucleoli. Which of is this pathologist examining?
the following cellular events would be elevated in *1.Nucleolus
these cells? 2.Ribosome
*1.rRNA production 3.Golgi apparatus
2.Mitosis 4.Rough endoplasmic reticulum
3.mRNA production 5.Mitochondria
4.Production of secreted proteins
5.Exocytosis [45]
A pathologist is studying a tissue section of the
[41] cerebrum by light microscopy. He notes a
Cells that are actively involved in the structure within a neuron that has both a granular
phagocytosis of extracellular material would and a fibrous component. What is the main
contain high levels of which of the following function of the cellular component?
cellular organelles? *1.rRNA production
*1.Lysosomes 2.ATP production
2.Rough endoplasmic reticulum 3.Protein trafficking
3.Golgi apparatus 4.Protein packaging
5.Site of protein synthesis Lack of the actin microfilaments

[46] [51]
A fresh biopsy specimen appears to be undergoing Which organelle in the liver is involved in the
atrophy. The nuclei of cells of this tissue are isolated detoxification of drugs?
by fractionation and homogenized. The resultant 1. *Smooth endoplasmic reticulum
nuclear material is separated in an agarose gel and 2. Peroxisomes
stained with ethidium bromide, which binds tymidin 3. Rough endoplasmic reticulum
residues of DNA. Examining the gel under ultraviolet 4. Ribosomes
light, the researcher notes that the DNA is separated in 5. Lysosomes
increments of 200 base pairs by comparing it with the
DNA standards. Which of the following accounts for [52]
this observation? Which of the following proteins binds to membrane
A. *Nucleosomes proteins and serves as a scaffold to support the
B. Chromosomes nuclear envelope?
C. Heterochromatin 1. *Lamins
D. Nucleoli 2. Actin
E. Euchromatin 3. Microtubules
4. Spectrin
[47] 5. Integrin
The primary function of intermediate filaments is
to [53]
1. *provide mechanical stability Which of the following statements BEST describe
2. generate movement basal folds?
3. produce the structural core of microvilli 1. *They are involved in active transport
4. produce the axoneme of flagella 2. They process cilia
5. be involved in the movement of chromosomes 3. They function in exocytosis
[48] 4. They are involved in lipid transport
Which of the following structures are involved in the 5. They make up part of Golgi complex
catabolism of hydrogen peroxide (H202)
*Peroxisomes [54]
Smooth endoplasmic reticulum Which one of the following cellular junctions
Rough endoplasmic reticulum prevents the free diffusion of materials across the cell
Ribosomes layer?
Lysosomes 1. *Hemidesmosome
2. Gap junction
[49] 3. Adherence junction
Which of the following statements BEST describes 4. Tight junction
immotile cilia (Kartagener`s) syndrome? 5. Nexus
1. *Lack of dynein arms
2. Degeneration of actin microfilaments [55]
3. Lack of centrioles At which of the following site is the characteristic
4. Absence of microtubules triple helical structure of the collagen initially
5. Lack of nexin proteins formed?
1. *Golgi apparatus
[50] 2. Exracellular space
Which of the following structures is responsible for 3. Nucleus
lack of movement of cilia? 4. Rough endoplasmic reticulum
*Dynein arms lacking the enzyme ATPase 5. Smooth endoplasmic reticulum
Lack of the nexin proteins
The microtubule of cilia lack the triplet formation [56]
Lack of the myosin microfilaments
Which of the following statement concerning the 2. It is continuous with the membranes of the
smooth endoplasmic reticulum is CORRECT? rough endoplasmic reticulum
1. *Is involved in the synthesis of 3. It serves as the boundary between the
phospholipids for membrane formation cytoplasmic and nuclear compartments
2. Stains with basic dyes 4. It is stabilized on its inner aspect by filaments
3. Is not involved in glycogen metabolism of the nuclear lamina
4. It is abundant in cells that are producing 5. It contains pores that regulate passage of
proteins for export materials in both directions between nucleus
5. Initial glycosylation of proteins occur here and cytoplasm
[57] [61]
Which of the following components of the cell Microvilli are structures that
enhances acidophilia? 1. *are made of actin microfilaments
1. *Smooth endoplasmic reticulum 2. are composed of intermediate filaments
2. Plasma membrane 3. allow flagella of spermatozoa to move
3. Rough endoplasmic reticulum 4. have basal
4. Golgi complex 5. bodies at their bases
5. Ribosomes
[62]
[58] All of the following features are characteristic of
Which of the following concerning the structure of apoptosis except one.
plasma membrane is CORRECT? 1. *Swelling of the cell.
1. *Carbohydrates attached to some proteins and 2. Formation of the "apoptosome", consisting of
lipids form the glycocalyx on the external side cytochromes
of the plasmalemma 3. Increase in cytoplasm calcium concentration.
2. The glycocalyx is present on the cytosolic side 4. Activation of endonucleases.
of the plasma membrane 5. Movement of mitochondria to the outside of the
3. Carbohydrates are located within the lipid plasma membrane.
bilayer
4. All integral proteins lack carbohydrate on [63]
external side of plasma membrane A metaphase-blocking dose of colchicines works by
5. Intermediate filaments attach to the which of the following mechanisms
carbohydrate portion of the glycoproteins 1. *Inhibition of tubulin polymerization
2. Depolymerization of myosin
[59] 3. Enhancement of tubulin polymerization
Which one of the following statements concerning the 4. Depolymerization of actin
nuclear structure is INCORRECT? 5. Binding to and stabilizing microtubules
1. *The nucleolus is composed of three parts: a
granular component, a fibrous component, and [64]
the nuclear envelope. Which organelle in the muscles is involved in the
2. The fibrous lamina is composed of Ca+2 accumulation?
intermediate filament proteins 1. *Smooth endoplasmic reticulum
3. Each nucleus of every human cell contains the 2. Peroxisomes
same DNA 3. Rough endoplasmic reticulum
4. The double membranes of the nuclear envelope 4. Ribosomes
are continuous with the rough endoplasmic 5. Lysosomes
reticulum
[60]
Which one of the following statements concerning the
nuclear envelope is INCORRECT?
1. *It is the site of synthesis of proteins that form
elements of the nuclear matrix
1
Basic tissues Моdul 2 A. Mucuous tissue
B. Allantois residue
1.After two months of appendectomy the patient again C. Tunic of umbilical arteries
has arrived in surgical department with the diagnosis: D. Tunic of umbilical vein
«The intestinal obstruction, an adhesive disease». E. Yolk sac residue
During operation damage of what epithelium covering a
peritoneum has reduced in development of adhesive 7.There is connecting with the gut vesicle is visible on
disease? the histologic micropreparation. It is an extraembryonic
A. Mesothelium organ. Primordial germ cells and erythroblasts
B. Endothelium (megaloblasts) are located in its wall. What organ is
C. Transitional epithelium this?
D. Simple columnar epithelium A. Yolk sac
E. Simple cuboidal epithelium B. Allantois
C. Placenta
2.Differentiation of cells within some type of D. Umbilical cord
hematopoiesis is accompanied by a nucleus extrusion. E. Amnion
What cell type is characterized by such morphologic
changes? 8.The primary function of brown adipose tissue is
A. Orthochromatophilic erythroblast A. To produce heat
(normoblast) B. To store unilocular energy
B. Polychromatophilic erythroblast C. To produce hormons
C. Basophilic erythroblast D. To mobilize lipid for export as fatty acide
D. Monoblast E. To initiate the shivering-induced mobilization of
E. Megakaryoblast lipids

3.Blood analysis of a healthy adult man revealed 3% of 9. After birth, growth in the length of long bones occurs
some blood formed elements. What formed elements primarily through
have this characteristic? A. Interstitial growth of cartilage cells in the
A. Eosinophils, band neutrophils epiphysial plate
B. Band neutrophils, lymphocytes B. Increased bone deposition under the periosteum
C. Eosinophils, lymphocytes C. The action of osteoblasts in the primary ossification
D. Eosinophils, mature neutrophils center
E. Lymphocytes, monocytes D. The action of osteoblasts in the secondary
ossification center
4. A patient with leukosis has disturbance of some type E. Appositional grows from the periphery
of hematopoiesis. This type has next characteristic:
antigen independent and antigen dependent phases, 10. The microscopic examination of wound lavage of
blastic transformation, clonal expansion, recirculation. a patient with acute woundy process of his shin
What type of hematopoiesis has enumerated features? revealed big contents of irregular extended-formed
A. Lymphocytopoiesis cells, with tough nucleus, the basophilic cytoplasm of
B. Granulopoiesis which includes many Lysosomes, phagosomes and
C. Erythropoiesis pinocytotic bubbles. What cells are found out in the
D. Monopoiesis wound?
E.Thrombopoiesis A. Connective tissue macrophages
B. Fibroblasts
5. A patient suffers from epithelial tumor of the right C. Plasmocytes
extrapulmonary bronchus. What morphologic type of D. Tissue basophils
epithelium gave rise to tumor? E. Fibrocytes
A. Simple columnar pseudostratified
B. Simple squamous
C. Simple cuboidal 11. After the radioactive exposure a patient has
D. Stratified squamous nonkeratinized haematopoietic stem cells disorder. The regeneration of
E. Transitional what cells of loose connective tissue will be damaged?
A. Macrophages
6. Compression of umbilical cord took place. But blood B. Fibroblasts
circulation between fetus and maternal organism C. Adipocytes
happened normally. What morphologic components of D. Pericytes
the umbilical cord promoted to this? E. Pigment cells
2
C. Т-killers cells
12. What parameter of a blood analysis most probably D. T-helper cells
testifies the presence in an organism of an acute E. T-suppressor cells
inflammation?
A. Ascending an amount of neutrophilic leucocytes 18. It is known, that plasma cells produce specific
B. Ascending an amount of reticulocytes antibodies on the given antigen. At injection of an
C. Ascending an amount of thrombocytes antigen the amount of plasmocytes increases. Due to
D. Decrease of an amount of basophilous leucocytes what blood cells the augmentation of number of
E. Decrease of an amount of monocytes plasmocytes is carried out.
A.B-cells
13. At study of a blood smear of the person with B. Eosinophils
presence of inflammatory process it is possible to see a C. Neutrophils
great number of spherical cells with a segmented D. Basophils
nucleus, acidophilic cytoplasm and shallow pink-violet E. T lymphocytes
granules in the cytoplasm. What blood cells are these?
A. Neutrophilic granulocytes 19. At the patient after transplantation of heterologous
B. Erythrocytes renal transplant the response of a casting-off has
C. Eosinophilic granulocytes developed. What basic effector cells participate in the
D. Basophilous granulocytes given immunological response?
E. Lymphocytes A. Cytotoxic T lymphocytes
B. T-helper cells
14. In a blood smear of the person who suffers an C. T-suppressor cells
allergy, it is possible to see a great number of spherical D. Plasmocytes
cells with a segmented nucleus, weakly basophilic E. В-plasmocytes
cytoplasm and major brightly pink granules in
cytoplasm. What blood cells type is in the smear? 20. At survey frequently sick child the poor amount of
A. Eosinophilic granulocytes immunoglobulins has been detected. Which of the
B. Erythrocytes enumerated cells of an immune system produce
C. Neutrophilic granulocytes immunoglobulins?
D. Basophilous granulocytes A. Plasma cells
E. Lymphocytes B. T-helper cells
C. Macrophages
15. After a bite of a bee in 12 hours allergic D. Plasmablasts
appearances on a skin (the edema, an itch) have E. T-suppressor cells
disappeared. It is known, that one of mediators of an
allergy is histamine. What blood cells have taken part in 21. In a patient with clinical signs of immunodeficiency
elimination of excess of histamine in a place of a bite? the number and functional activity of T and B-
A. Eosinophils lymphocytes are not changed. Defect with dysfunction
B. Neutrophils of antigen-presentation to the immunocompetent cells
C. Basophils was found during investigation on the molecule level.
D. Lymphocytes Defect of what cells is the most probable.
E. Monocytes A. Macrophages, B-cells
B. NK-cells
16. Punctuate hemorrhage was found our in the patient C. T-lymphocytes
after application of a tourniquet. With dysfunction of D. Fibroblasts, T-lymphocytes, B-lymphocytes
what blood cells is it connected? E .0-lymphocytes
A. Platelets
B. Monocytes 22. Live vaccine is injected into the human body.
C. Eosinophils Increasing activity of what cells of connective tissue
D. Neutrophils can be expected?
E. Lymphocytes A. Plasma cells and lymphocytes cells
B. Fibroblasts and mast cells
17. At survey of the patient the poor amount of C. Adipose cells and adventitial
immunoglobulins has been detected. What from the D. Macrophages and fibroblasts
enumerated cells of an immune system produce E. Pigment cells and pericytes
immunoglobulins?
A. Plasma cells (plasmocytes) 23. The patient with complaints of рain in eyes, which
B. Plasmablasts have arisen after duration stay of the patient in a field
3
during dust burs, converted to the ophthalmologist. The E. To macrophages
doctor has established the surface damages of a cornea
epithelium. What cells will provide regeneration of the 29. On a histology microslide in a loose connective
defective epithelium? tissue big cells filled by basophilic metachromatic
A. Cells stratum basale granules are found. Histochemically it fixed, that
B. Cells of a stratum corneum granules contain heparin and histamin. What cells are
C. Cells of a stratum granulosum most probably found in a microslide?
D. Cells of a stratum lucidum A. Mast cells
E. Superficial cells B. Fibroblasts
C. Plasma cells
24. The inner envelope of the blood and lymph vessels D. Macrophages
lines an epithelium. Name it. E. Аdipose cells
A. An endothelium
B. Stratified squamous epithelium 30. After the transferred chemical injury of an
C. Simple squamous epithelium esophagus there has occurred its local narrowing owing
D. Epidermis to formation of collagen cicatrix. What cells of a loose
E. Transitional epithelium connective tissue share in formation of cicatrices?
A. The mature active fibroblasts
25. The patient with complaints to a pain in a thoracic B. Inactive fibroblasts
cavity at respiration converted to the doctor. After C. Fibrocytes
survey there were detected modifications of an D. Мyofibroblasts
epithelium of a pleura. What epithelium has tested E. Adipose cells
modifications?
A. Simple squamous 31. At injection to the patient of a proteinaceous drug
B. Simple columnar ciliated the amount of plasma cells, which produce specific
C. Simple cuboidal antibodies on the given antigen, increases. Due to what
D. Simple columnar blood cells there is an augmentation of an amount of
E. Stratified columnar plasma cells?
A. B cells
26. Under action of radiation cells of a basal layer of an B. T-suppressor cells
epidermis have suffered. What function of the last will C. Т-killers
weaken or will be breaked first? D. T-helper cells
A. Regenerative E. В-memory cells
B. Absorptive
C. Protective 32. The leading part in shaping a vascular phase of an
D. Barrier inflammation is played with histamine. What cell of
E. Secretory loose connective tissue produce histamine?
A. Mast cell
27. In experiment the significant amount of stem cells B. A fibroblast
of bone marrow definitely eroded. Regeneration of C. A plasma cells
what population of cells in composition of loose D. A fibrocyte
connective tissue will be inhibited? E. A macrophage
A. Macrophages
B. Fibroblasts 33. At a wound repair on the defective field of
C. Pigment cells penetrating layers of a skin occurs granular tissue. What
D. Adipocytes cells are discovered after wound and produce granular
E. Pericytes tissue?
A. Myofibroblasts
28. The initial state of an inflammation - alterative is B. Fibrocytes
characterized by dilating of blood capillaries on a field C. Plasma cells
of damage, decrease of a circulation, a raise of a D. Macrophages
permeability of a wall of vessels. To what from the cells E. Adipose cells
given below, the leading role in inflammatory response
is necessary? 34. After wound of an extremity the woman had
A. To mast cells bleeding which is accompanied by time-lagged
B. To fibroblasts coagulation of a blood (at a normal amount of platelets
C. To plasma cels in analysis of a blood). What substance of loose
D. To eosinophils
4
connective tissue hinders with process of blood 40. At people of advanced age it is observed exuberant
coagulation? What cell is it produced by? losses of mass of an osteal tissue that reflects
A. Heparin, mast cells development of an osteoporosis. Activation of what
B. Histamin, mast cells cells of bone tissue causes development of the given
C. Heparin, macrophage disease ?
D. Heparin, plasma cells A. Osteoclasts
E. Heparin, fibroblast B. Osteoblasts
C. Macrophages
35. After a wound repair on its place the cicatrix has D. Mast cells
formed. What substance is the basic ingredient of this E. Osteocytes
type of a connective tissue?
A. Collagen 41. At the laboratory experiment the leukocyte culture
B. Keratan sulfate was mixed with staphylococci. Neutrophil leukocytes
C. Elastin engulfed and digested bacterial cells. This process are
D. Hyaluronic acid termed:
E. Chondroitin sulfate A. Phagocytosis
B. Facilitated diffusion
36. At an endochondral bone formation of tubular C. Pinocytosis
bones between epiphysis and diaphysis centers of an D. Osmosis
ossification form a plate, which further ensures body E. Diffusion
height of bones in length. How this plate is named?
A. Epiphyseal plate 42. In a patient after transplantation of the foreign
B. Bone lamellae kidney rejection of the transplated organ developed.
C. Subperiosteal bone collar What main effecter cells participate in this reaction at
D. An osteon first?
E. Inner circumferential lamellar A.T-killers (T-cytotoxic)
B. plasma cells
37. Owing to chondrodystrophia (anomaly of C. T-lymphocytes –supressors
development of a cartilage) has been damaged a D. B-cells
fibrocartilage. In what organ is possible to observe E Fibroblasts
pathological modifications?
A. In intervertebral disks 43.Examination of a patient who was subjected to X-
B. In a trachea ray irradiation revealed injury of the splenic white pulp.
C. In a larynx What cells of the white pulp undergo to this
D. On articulate surfaces pathological change at first?
E. In an auricle A. Lymphocytes
B. Basophils
38. At the patient with a permanent injury of the upper C. Neutrophils
extremity breaking processes of regeneration of a D. Mast cells
cartilage owing to damage of chondrogenic cells is E. Red blood cells
observed. What cells have tested damage?
A. Cells of inner cellular layer of perichondrium 44. The child’s scratch, which had been caused by light
B. Cells of an upper layer of perichondrium skin’s trauma, disappeared after 10 days. What skin
C. Cells in isogenous droups layer provided for reparative regeneration?
D. Cells of region of a young cartilage A. Stratum basale;
E. Cells which enter from blood vessels B. Stratum spinosum;
C. Stratum granulosum;
39. It is offered to the student two microslides. On first D. Stratum corneum;
- an elastic cartilage (stained by orsein), on second - E. Stratum brilliantum.
hyaline (stained by hematoxylin - eosine). To what
features of them it is possible to distinguish. 45. After two months of appendectomy the patient
A. On presence of elastic fibers again has arrived in surgical department with the
B. On presence of isogenic groups of cells diagnosis: «The intestinal obstruction, an adhesive
C. On presence of region a young cartilage disease». During operation, damage of what epithelium
D. On presence of perichondrium covering a peritoneum has reduced in development of
E. On presence of an extracellular matrix adhesive disease.
A. Mesothelium
B. Endothelium
5
C. Transitional epithelium blastic transformation, clonal expansion, recirculation.
D. Simple columnar epithelium What type of hematopoiesis has enumerated features?
E. Simple cuboidal epithelium A. Lymphocytopoiesis
B. Granulopoiesis
46. An experimental animal was undergone by C. Erythropoiesis
overdoses of X-rays. Due to this total destruction of D. Monopoiesis
CFU-GM cells in bone marrow took place. Further E. Thrombopoiesis
development of what blood formed elements will be
happen in this case normally? 52.A patient suffers from epithelial tumor of the right
A. Erythrocytes extrapulmonary bronchus. What morphologic type of
B. Monocytes epithelium gave rise to tumor?
C. Eosinophils A. Simple columnar pseudostratified
D. Basophils B. Simple squamous
E. Neutrophils C. Simple cuboidal
D. Stratified squamous nonkeratinized
47. Differentiation of cells within some type of E. Transitional
hematopoiesis is accompanied by a nucleus extrusion.
What cell type is characterized by such morphologic 53.Compression of umbilical cord took place. However,
changes? blood circulation between fetus and maternal organism
A. Orthochromatophilic erythroblast (normoblast) happened normally. What morphologic components of
B. Polychromatophilic erythroblast the umbilical cord promoted to this?
C. Basophilic erythroblast A. Mucous tissue
D. Monoblast B. Allantois residue
E. Megakaryoblast C. Tunic of umbilical arteries
D. Tunic of umbilical vein
48. Blood analysis of a healthy adult man revealed 3% Е Yolk sac residue
of some blood formed elements. What formed elements
have this characteristic? 54. The primary function of brown adipose tissue is
A. Eosinophils, band neutrophils A. To produce heat
B. Band neutrophils, lymphocytes B. To store unilocular energy
C. Eosinophils, lymphocytes C. To produce hormones
D. Eosinophils, mature neutrophils D. To mobilize lipid for export as fatty acids
E. Lymphocytes, monocytes E. To initiate the shivering-induced mobilization of
lipids
49.After invasion of bacteria into wounded surface of
skin they were destroyed by receptor mediated 55. The extra cellular matrix and the cytoskeleton
phagocytosis. What formed blood elements participate communicate across the cell membrane through
in this protective reaction? A. Integrins
A. Neutrophils B. Proteoglycans
B. Basophils C. Microtubules
C. Eosinophils D. Hyaluronic acid
D. Lymphocytes E. Microfilaments
E. Platelets
56. After birth, growth in the length of long bones
50. After overdoses of -rays destruction of B- occurs primarily through
lymphocytes in human hematopoietic organs happens. A. Interstitial growth of cartilage cells in the
Reduction of what blood plasma substances will be epiphysial plate
observes in this case. B. Increased bone deposition under the periosteum
A. Immunoglobulins (antibodies) C. The action of osteoblasts in the primary ossification
B. Albumins center
C. Cholesterol D. The action of osteoblasts in the secondary
D. Glucose ossification center
E. Fibrinogen E. Appositional grows from the periphery

51. A patient with leucosis has disturbance of some 57. The microscopic examination of wound lavage of a
type of hematopoiesis. This type has next characteristic: patient with acute wound process of his shin revealed
antigen independent and antigen dependent phases, big contents of irregular extended-formed cells, with
tough nucleus, the basophilic cytoplasm of which
6
includes many Lysosomes, phagosomes and pinocytotic 63.Blood analysis of a healthy adult man revealed 3%
bubbles. What cells are found out in the wound? of some blood formed elements. What formed elements
F. Connective tissue macrophages have this characteristic?
G. Fibroblasts 1. Eosinophils, band neutrophils
H. Plasma cells 2. Band neutrophils, lymphocytes
I. Mast cells 3. Eosinophils, lymphocytes
J. Fibrocytes 4. Eosinophils, mature neutrophils
5. Lymphocytes, monocytes
58.Blood sampling for bulk analysis is recommended to
be performed on an empty stomach and in the morning. 64. After invasion of bacteria into wounded surface of
What changes in blood count can occur if to perform skin they were destroyed by receptor mediated
blood sampling after food intake? phagocytosis. What formed blood elements participate
A. Increased contents of leukocytes in this protective reaction?
B. Reduced contents of erythrocytes 1. Neutrophils
C. Increased contents of erythrocytes 2. Basophils
D. Reduced contents of thrombocytes 3. Eosinophils
E. Increased plasma proteins 4. Lymphocytes
5. Platelets
59.After the radioactive exposure, a patient has
haematopoietic stem cells disorder. The regeneration of
what cells of loose connective tissue will be damaged? 65. A patient complains of pain in the abdomen. It is
F. Macrophages determined that the pain was a result of large
G. Fibroblasts substances passing between cells that line the intestinal
H. Adipose cells lumen directly into the underlying connective tissue.
I. Pericytes Which of the following types of junctions were absent
J. Pigment cells or not completely functional?
A. Zonulae occludens
B. Maculae adherens
60. The neonatal child has a disorder of a thymus gland. C. Zonulae adherens
What type of a hemopoiesis will be disturbed? D. Gap junctions
1. Lymphopoiesis E. Hemidesmosome
2. Monocytopoiesis
3. An erythrogenesis 66. Junctions are essential in maintaining the close
4. Granulocytopoiesis association of the cells within the epidermis. Which of
5. Platelet formation the following junctions in maintaining cell adherence in
all layers of the epidermis?
61. An experimental animal was undergone by a. Maculae adherens
overdoses of X-rays. Due to this total destruction of b. Gap junctions
CFU-GM cells in bone marrow took place. Further c. Hemidesmosome
development of what blood formed elements will be d. Zonulae adherens
happening in this case normally. e. Zonulae occludens
1. Erythrocytes
2. Monocytes 67. Which of the following proteins aids in
3. Eosinophils maintaining the close association of the cells of the
4. Basophils epidermis of the skin
5. Neutrophils a. Intermediate filaments
b. Occludins
62. Differentiation of cells within some type of c. Cadherins
hematopoiesis is accompanied by a nucleus extrusion. d. Vinculin
What cell type is characterized by such morphologic e. Collagen
changes?
1 .Orthochromatophilic erythroblast (normoblast) 68. Simple squamous epithelium is prevalent
2. Polychromatophilic erythroblast throughout the body and organ systems, which of the
3 .Basophilic erythroblast following is a primary function of this type of
4. Monoblast epithelium.
5. Megakaryoblast a. Lubrication
b. Secretion
c. Adsorption
7
d. Protection d. Simple squamous epithelium
e. Excretion e. Simple columnar epithelium

69. A biopsy of the urinary bladder is done in a patient 75. A pathologist is examining a tissue section with an
who has complained of lower abdominal pain. In the electron microscope. He notices immune cells in
pathologist’s report, the luminal epithelium is described connective tissue that have a prominent Golgi apparatus
as normal. What type of epithelium did the pathologist and a nucleus with heterochromatin arranged in a spoke
observe? wheel fashion. Which of the following cell types is
a. Transitional epithelium Stratified squamous being observed?
epithelium a.Plasma cell
b. Simple squamous nonkeratinized b.Macrophage
epithelium c.Fibroblast
c. Pseudostratified ciliated columnar epithelium d.Mast cell
d. Simple columnar epithelium e.Brown adipose cell

70. Microvilli are essential components of epithelial 76. Which of the following provides a primary
cells of the small intestine. Which of the following structural network in hematopoietic organs?
functions would be defective resulting from a lack of a. Reticular connective tissue
microvilli on epithelia? b. White adipose tissue
a. Absorption c. Brown adipose tissue
b. Stretching d. Pigment connective tissue
c. Movement e. Elastic connective tissue
d. Protection
e. Secretion 77. Your patient suffered a complete tear of biceps
brachial muscle at its insertion site at the radial
71. The layer of cells that lines the outer surface of the tuberosity. For proper treatment, you must identify the
lungs can be irritated by inhaled asbestos particles. damaged tissue. Which of the following tissues was
These cells can become cancerous and lethal. Which of damaged at the tear site?
the following is the cell layer being described? A Dense regular connective tissue
a. Simple squamous mesothelium b.Reticular tissue
b. Simple squamous endothelium c.Elastic tissue
c. Transitional epithelium d.Loose areolar connective tissue
d. Simple cuboidal epithelium e.Dense irregular connective tissue
e. Simple columnar epithelium
78. Your patient suffers from an immediate
72. Which of the following is a characteristic hypersensitivity (allergy) reaction. Which of the
typical of simple layers of epithelia? following cell types is responsible for this condition?
a. All the cells rest on a basal lamina a. Mast cell
b. All the cells cover surface of the skin b. Plasma cell
c. All the cells function to prevent c. Fibroblast
abrasion d. Macrophage
d. All the cells are joined by synapses e. Adipose cell
e. All the cells have flagella
79. Which of the following types of collagen is the
73. Macrophages would be the most abundant in most widely distributed and abundant within the body?
which of the following tissues? a. Type I collagen
a. Loose connective tissue b. Type II collagen
b. Dense regular connective tissue c. Type III collagen
c. Brown adipose tissue d. Type IV collagen
d. Dense irregular connective tissue e. Type V collagen
e. Embryonic tissue
80. Immunocytochemistry is performed on a tissue
74. What type of epithelium normally lines the section using an antibody specific for type IV collagen.
lumen of the trachea? The investigator notes staining within an organ that
a. Pseudostratified ciliated columnar contains abundant and varied connective tissue. Which
epithelium of the following would be identified by the antibody to
b. Stratified columnar epithelium type IV collagen?
c. Transitional epithelium a. Basal lamina
8
b. Reticular fibers d. Cartilage at pubic symphysis
c. Elastic fibers e. Costal cartilage
d. Loose connective tissue
e. Adipose cells 88. Which of the following organelles plays an
essential role in the sulfation of glycosaminoglycans
81. Which of the following microscopically best within chondrocytes?
characterizes brown fat cells? a. Golgi apparatus
a. Multiple droplets of lipid b. Polyribosomes
b. Lack of mitochondria c. Smooth endoplasmic reticulum
c. Peripheral flattened nucleus d. Rough endoplasmic reticulum
d. Abundant RER e. Ribosomes associated with the nuclear
e. Poor blood supply. envelope

82. Which of the following provide the nutrients to 89. Within compact bone, which of the following
mature chondrocytes? form connections between Haversian systems?
a. Diffusion through the matrix a. Volkmann’s canals
b. Vasculature of the cartilage b. Howship’s lacuna
c. Canals within the cartilage c. Canaliculi
d. Capillary network within the matrix d. Lacunae
e. Cellular interconnections between adjacent e. Cement lines
chondrocytes
90. Cessation of growth at which of the following
83. Which of the following would best characterize sites would result in no further longitudinal growth of
fibrocartillage? long bones.
a. Rows on chondrocytes a. Epiphyseal plate
b. Presence of a perichodrium b. Endosteum
c. Found at the epiphyseal plate c. Primary center of ossification
d. Large, densely stained territorial matrix d. Secondary center of ossification
e. Lines bones at articular sites e. Diaphysis
84. Which of the following is the major component
of the matrix of the cartilage? 91. A blood analysis of your patient shows an
a. Collagen increased level of parathyroid hormone. Which of the
b. Isogenous groups following would result from increased levels of this
c. Lacunas hormone?
d. Chondrocytic processes a. Increased activity of osteoclasts and
e. Perichondrium bone resorbtion
b. Increased mitotic activity of
85. Chondroblasts are derived from which of the osteoprogenitor cells
following cell types? c. Decreased serum calcium levels
a. Mesenchymal cells d. Increased bone formation
b. ibroblasts e. Increased hydroxyapatite crystal formation
c. Proliferating chondrocytes
d. Cells within isogenous groups 92. Which of the following could be administered
e. Cells of the perichondrium to override the effects of increased parathyroid
hormone?
86. Which of the following best characterizes a. Calcitonin
hyaline cartilage? b. Vitamin D
a. Abundant matrix c. Bone morphogenic protein
b. No perichondrium d. Osterprotegerin
c. Single chondrocytes e. Somatrophin
d. Found between vertebral bodies
e. Least resilient of all forms of cartilage 93. Osteoclasts would be found at which of the
following sites in bone?
87. Elastic cartilage is found in which of the a. Howship’s lacuna
following body structures? b. Haversian canal
a. Epiglottis c. Canaliculi
b. Intervertebral discs d. Endosteum
c. Tracheal rings e. Periosteum
9
100.. At formation of a muscle tissue in an
94. Bone of a 75-year-old man shows a decreased embryogenesis process of fusion of myoblasts in
number of osteoblasts. However, the number of myosyncithium is quenched. What tissue development
osteoclasts is greatly increased. This individual’s bones will be disturbed?
are easily fractured. Which of the following disease A. A skeletal muscle tissue
conditions would you expect? B. A muscle tissue derivate of neural tube
a. Osteoporosis C. A muscle tissue derivate of ectoderm
b. Osteoartrosis D. A smooth muscle tissue derivate of mesenchyme
c. Osteopetrosis E. A cardiac muscle tissue
d. Osteochondrosis
e. Rickets 101. At examination of a striated muscle fiber after a
mechanical trauma destruction of thick myofilaments is
95. The hormone calcitonin acts directly inhibited observed. Where pathological modifications if
on which of the following cell types? filaments will be studied in polarized light will be
a. Osteoclasts localized?
b. Osteocytes A. In A band
c. Osteoblasts B. In I band
d. Chondroblasts C. In half of A-band
e. Chondrocytes D. In A band and in I band
E. In half of I band
96. At which of the following sites in bone would
you find osteoblasts during active deposition of new 102. At examination of a striated muscle fiber after
bone matrix? treatment of hydrolyses enzymes destruction of thin
a. Surface of bone myofilaments is observed. What supermolecular
b. Haversian canal structures have tested damage?
c. Lacuna A. Actin myofilaments
d. Periosteum B. Myosin myofilaments
e. Canaliculi C. Tonofibrils
D. Tropocollagin complexes
97. In which of the following bone sites are E. Nucleoprotein complexes
canaliculi found?
a. Compact bone 103. During an operative measure at the patient the
b. Perichondrium part of a small intestine is removed. Due to what
c. Bone marrow elements regeneration of a muscular layer is possible?
d. Newly mineralized bone matrix A. Smooth muscle cells
e. Howship’s lacuna B. Мyosyncithium
C. Adipose cells
D. Satellite cells
98.. Breaking architectonics in a somite region near to E. Fibrocytes
an endoderm and notochord is found during biopsy
examination of embryonic material. Disorders of what 104. Patient with injured muscles of the lower
tissues or organs will be? extremities was admitted to the traumatological
A. Skeletal tissues department. Due to what cells is reparative regeneration
B. Urogenital system of the muscle fibers and restoration of the muscle
C. Striated skeletal muscle function possible?
D. Striated cardiac muscle A. Satellite-cells
E. Fibrillar connective tissue of a skin B. Myoepithelial cells
C. Myoblasts
99..In traumatological hospital, the patient with damage D. Myofibroblasts
of muscles of the inferior extremity is supplied. Due to E. Fibroblasts
what cells reparation of muscle filaments and
regeneration of function of muscles is possible? 105. At the patient the tremor and convulsions that is
A. Satellite cells caused by breaking of nerve fibers myelination of a
B. Myoepithelial cells central nervous system is observed. Function of what
C. Myofibroblasts cells suffers at this disease?
D. Fibroblasts A. Olygodendrocytes
E. Myoblasts B. Fibrillar astrocytes
C. Ependymal cells
10
D. Protoplasmatic astrocytes arteriorraphy was performed. Regeneration of what
E. Microglial cells muscle tissue is necessary for regeneration of a
wholeness of the vessel`s tunica media?
106. In what cells during life the mitosis is not A. Smooth muscle tissue
observed, and the quantitative content DNA remains to B. Striated skeletal muscle tissue
stationary values? C. Striated cardiac muscle tissue
A. In neurones D. Myoepithelial cells
B. In hemopoietic E. Satellite cells
C. In striated muscle fiber
D. In smooth muscle 112. The patient with complaints of pain in eyes, which
E. In an epidermis have arisen after duration stay of the patient in a field
during dust burs, converted to the ophthalmologist. The
107 At patient with a chronic alcoholism disturbance of doctor has established the surface damages of a cornea
coordination of movements and equilibrium owing to epithelium. What cells will provide regeneration of the
morphologic disturbances in cerebellum. Damage of defective epithelium?
what cerebellar cells we can suppose at a first head? A. Cells stratum basale
A. Purkinje cells B. Cells of a stratum corneum
B. Basket cells C. Cells of a stratum granulosum
C. Golgi cells D. Cells of a stratum lucidum
D. Pyramidal cells E. Superficial cells
E. Stellate cells
113.In traumatological hospital, the patient with
108. Some nervous system organ is presented in damage of muscles of the inferior extremity is supplied.
histological section. It contains capsule, connective Due to what cells reparation of muscle filaments and
tissue layers that extend from the capsule. Bipolar regeneration of function of muscles is possible?
(pseudounipolar) neurons are located bellow capsule. A. Satellite cells
Peryikaryons of this neurons are surrounded by glial B. Myoepithelial cells
cells. Center of organ contains neurons processes. What C. Myofibroblasts
organ is it? D. Fibroblasts
A. Dorsal root ganglion (spinal ganglion) E. Myoblasts
B. Vegetative ganglion
C. Spinal cord 114. At formation of a muscle tissue in an
D. Cerebral cortex embryogenesis process of fusion of myoblasts in
E. Cerebellar cortex myosyncithium is quenched. What tissue development
will be disturbed?
109. Cerebral cortex contains large and giant pyramidal A. A skeletal muscle tissue
shaped neurons. It is a feature of the cerebral cortex. B. A muscle tissue derivate of neural tube
Discovery of this cells was connected with the name of C. A muscle tissue derivate of ectoderm
following scientist: D. A smooth muscle tissue derivate of mesenchyme
A. Bets E. A cardiac muscle tissue
B. Purkinje
C. Golgi 115. At examination of a striated muscle fiber after a
D. Nissle`s mechanical trauma destruction of thick myofilaments is
E. Paneth`s observed. Where pathological modifications if
filaments will be studied in polarized light will be
110. In human embryo neural tube neuroepithelial localized?
ectodermic cells differentiate into neuroblasts and A. In A band
gliablasts. Owing to migration of these cells some B. In I band
zones (layers) are formed in neural tube. Which of the C. In half of A-band
following zone is where bodies of neuroblasts locate in? D. In A band and in I band
A. Mantle (intermediate) zone E. In half of I band
B. Ventricular zone
C. Marginal zone 116. At examination of a striated muscle fiber after
D. Outer limiting membrane treatment of hydrolyses enzymes destruction of thin
E. Inner limiting membrane myofilaments is observed. What super molecular
structures have tested damage?
111. During road accident at the man of 30 years the A. Actin myofilaments
femoral artery has been injured. In hospital the B. Myosin myofilaments
11
C. Tonofibrils cells. Center of organ contains neurons processes. What
D. Tropocollagin complexes organ is it?
E. Nucleoprotein complexes A. Dorsal root ganglion (spinal ganglion)
B. Vegetative ganglion
117. During an operative measure at the patient the C. Spinal cord
part of a small intestine is removed. Due to what D. Cerebral cortex
elements regeneration of a muscular layer is possible? E. Cerebellar cortex
A. Smooth muscle cells
B. Мyosyncithium 123. Cerebral cortex contains large and giant pyramidal
C. Adipose cells shaped neurons. It is a feature of the cerebral cortex.
D. Satellite cells Discovery of this cells was connected with the name of
E. Fibrocytes following scientist:
A. Bets
118. Patient with injured muscles of the lower B. Purkinje
extremities was admitted to the traumatological C. Golgi
department. Due to what cells is reparative regeneration D. Nissle`s
of the muscle fibers and restoration of the muscle E. Paneth`s
function possible?
A. Satellite-cells 124. In human embryo neural tube neuroepithelial
B. Myoepithelial cells ectodermal cells differentiate into neuroblasts and
C. Myoblasts gliablasts. Owing to migration of these cells some
D. Myofibroblasts zones (layers) are formed in neural tube. Which of the
E. Fibroblasts following zone is where bodies of neuroblasts locate in?
A. Mantle (intermediate) zone
119. At the patient the tremor and convulsions that is B. Ventricular zone
caused by breaking of nerve fibers myelination of a C. Marginal zone
central nervous system is observed. Function of what D. Outer limiting membrane
cells suffers at this disease? E. Inner limiting membrane
A. Olygodendrocytes
B. Fibrillar astrocytes 125. Myocardiodistrophy promotes a breach of cardiac
C. Ependymal cells muscle cells metabolism. Which of the following
D. Protoplasmatic astrocytes embryonic origin breach causes this pathology?
E. Microglial cells A. Myoepicardial plate
B. Myotome
120. In what cells during life the mitosis is not C. Endoderm
observed, and the quantitative content DNA remains to D. Ectoderm
stationary values? E. Mesenchyme
A. In neurones
B. In hemopoietic 126. Under the influence of long vibration and a motion
C. In striated muscle fiber in motor regions of cortex brain occurs a gliolis –
D. In smooth muscle phagocytosis of damaged neurons by neuroglial cells.
E. In an epidermis What neuroglia participates in this process?
A. Microglia cells
121. At patient with a chronic alcoholism disturbance B. Protoplasmatic astrocytes
of coordination of movements and equilibrium owing to C. Oligodendrocytes
morphologic disturbances in cerebellum. Damage of D. Ependymal cells
what Cerebellar cells we can suppose at a first head. E. Fibrous astrocytes
A. Purkinje cells
B. Basket cells 127. At a combustion of a skin the patient feels a strong
C. Golgi cells pain. Processes of what neurons first conduct pain
D. Pyramidal cells nervous impulse in central nervous system?
E. Stellate cells A. Pseudounipolar neurons
B. Bipolar neurons
122. Some nervous system organ is presented in C. Unipolar neurons
histological section. It contains capsule, connective D. Multipolar neurons
tissue layers that extend from the capsule. Bipolar E. Purkinje neurons
(pseudounipolar) neurons are located bellow capsule.
Peryikaryons of these neurons are surrounded by glial
12
128. At a brain injury are defected glial cells, which 134. Which of the following regions contains
meet in gray matter to central nervous system more the Z line in skeletal muscle?
often. What is the name of these cells? a. I band
A. Protoplasmic astrocytes b. H band
B. Oligodendrocytes
c. A band
C. Microglia
D. Fibrous astrocytes
d. M band
E. Ependymal cells e. At the junction of the A band and I band

129. In skeletal muscle fibers, which of the 135. Which of the following molecules is found
following would exhibit ATPase activity? within the groove created by actin filaments?
a. Myosin filaments a. Tropomyosin
b. Actin filaments b. Troponin I
c. Troponin c. Myosin
d. T-tubule system d. Dystrophin
e. Tropomyosin e. Alpha Actinin

130. Dense bodies within smooth muscle fibers 136. What is the function of the T tubule system
most closely associate with which of the following in skeletal muscle?
components of skeletal muscle? a. Regulates Ca2+ release from sarcoplasmic
a. Z line reticulum
b. A band b. Stores Ca2+
c. I band c. Provides channel for Ca2+ movement with
d. H band muscle
e. M line d. Wraps individual muscle fibers to form a
fascicle
131.Which of the following best characterizes a e. Site of attachment of actin filaments
cardiac muscle fiber?
a. Intercalated disks
b. Multiple nuclei
c. Lack of striations
d .Spindle shaped fiber
e. Myoneural junctions

132. Which of the following events occur


during the process of contraction of skeletal
muscle?
a. Release of calcium ions by
sarcoplasmic reticulum
b. Calcium ions bind to tropomyosin
c. Actin filaments shorten
d. Z line disappears
e. Sarcomeres lengthen

133. Which of the following best characterizes


smooth muscle fibers?
a. Caveolae as the calcium ion binding vesicals
b. Regularly arranged actin and myosin
myofilaments
c. Multiple, peripheral nuclei
d. Extensive myoneural junctions
e. Extensive sarcoplasmic reticulum
S/M 5-6
1 At survey of the patient the poor amount of immunoglobulins has been detected. What from the enumerated cells of an
immune system produce immunoglobulins?
A. Plasma cells (plasmocytes)
B. Plasmablasts
C. Т-killers cells
D. T-helper cells
E. T-suppressor cells

2. It is known, that plasma cells produce specific antibodies on the given antigen. At injection of an antigen the amount of
plasmocytes increases. Due to what blood cells the augmentation of number of plasmocytes is carried out?
A.B-cells
B. Eosinophils
C. Neutrophils
D. Basophils
E. T lymphocytes

3. At the patient after transplantation of heterologous renal transplant the response of a casting-off has developed. What
basic effecter cells participate in the given immunological response?
A. Cytotoxic T lymphocytes
B. T-helper cells
C. T-suppressor cells
D. Plasmocytes
E. В-plasmocytes

4. At survey frequently sick child the poor amount of immunoglobulins has been detected. Which of the enumerated cells of
an immune system produce immunoglobulins?
A. Plasma cells
B. T-helper cells
C. Macrophages
D. Plasmablasts
E. T-suppressor cells

5. In a patient with clinical signs of immunodeficiency the number and functional activity of T and B-lymphocytes are not
changed. Defect with dysfunction of antigen-presentation to the immunocompetent cells was found during investigation on
the molecule level. Defect of what cells is the most probable?
A. Macrophages, B-cells
B. NK-cells
C. T-lymphocytes
D. Fibroblasts, T-lymphocytes, B-lymphocytes
E .0-lymphocytes

6. The specimens present sections of hemopoietic and immunogenetic organs. Organ consists of lymphoid tissue forming
different structures (lobules, cortex, medulla ). In what organ does antigen-independent, proliferation and differentiation
take place.
A. Thymus
B. Lymph nodes
C. Spleen
D. Tonsil
E. Hemolymph nodes

7. Live vaccine is injected into the human body. Increasing activity of what cells of connective tissue can be expected?
A. Plasma cells and lymphocytes cells
B. Fibroblasts and mast cells
C. Adipose cells and adventitial
D. Macrophages and fibroblasts
E. Pigment cells and pericytes

1
8. The patient with complaints of pain in eyes, which have arisen after duration stay of the patient in a field during dust burs,
converted to the ophthalmologist. The doctor has established the surface damages of a cornea epithelium. What cells will
provide regeneration of the defective epithelium?
A. Cells stratum basale
B. Cells of a stratum corneum
C. Cells of a stratum granulosum
D. Cells of a stratum lucidum
E. Superficial cells

32. The inner envelope of the blood and lymph vessels lines an epithelium. Name it.
A. An endothelium
B. Stratified squamous epithelium
C. Simple squamous epithelium
D. Epidermis
E. Transitional epithelium

33. The patient with complains to a pain in a thoracic cavity at respiration converted to the doctor. After survey, there were
detected modifications of an epithelium of pleura. What epithelium has tested modifications?
A. Simple squamous
B. Simple columnar ciliated
C. Simple cuboidal
D. Simple columnar
E. Stratified columnar

34. Under action of radiation cells of a basal layer of an epidermis have suffered. What function of the last will weaken or will
be break first?
A. Regenerative
B. Absorptive
C. Protective
D. Barrier
E. Secretory

35. In experiment the significant amount of stem cells of bone marrow definitely eroded. Regeneration of what population of
cells in composition of loose connective tissue will be inhibited.
A. Macrophages
B. Fibroblasts
C. Pigment cells
D. Adipose cells
E. Pericytes

36. The initial state of an inflammation - alterative is characterized by dilating of blood capillaries on a field of damage,
decrease of a circulation, a raise of a permeability of a wall of vessels. What cells which given below, the leading role in
inflammatory response is necessary?
A. Mast cells
B. Fibroblasts
C. Plasma cells
D. Eosinophils
E. Macrophages

39. After the transferred chemical injury of an esophagus there has occurred its local narrowing owing to formation of
collagenic cicatrix. What cells of a loose connective tissue share in formation of cicatrices?
A. The mature active fibroblasts
B. Inactive fibroblasts
C. Fibrocytes
D. Myofibroblasts
E. Adipose cells

40. At injection to the patient of a proteinaceous drug the amount of plasma cells which produce specific antibodies on the
given antigen increases. Due to what blood cells there is an augmentation of an amount of plasma cells?
2
A. B cells
B. T-suppressor cells
C. Т-killers
D. T-helper cells
E. В-memory cells

56. At the patient the tremor and convulsions that is caused by breaking of nerve fibers myelination of a central nervous
system is observed. Function of what cells suffers at this disease?
A. Olygodendrocytes
B. Fibrillar astrocytes
C. Ependymal cells
D. Protoplasmatic astrocytes
E. Microglial cells

58. At patient with a chronic alcoholism disturbance of coordination of movements and equilibrium owing to morphologic
disturbances in cerebellum. Damage of what Cerebella cells we can suppose at a first head.
A. Purkinje cells
B. Basket cells
C. Golgi cells
D. Pyramidal cells
E. Stellate cells

59 Some nervous system organ is presented in histological section. It contains capsule, connective tissue layers that extend
from the capsule. Bipolar (pseudounipolar) neurons are located bellow capsule. Peryikaryons of these neurons are surrounded
by glial cells. Center of organ contains neurons processes. What organ is it?
A. Dorsal root ganglion (spinal ganglion)
B. Vegetative ganglion
C. Spinal cord
D. Cerebral cortex
E. Cerebellar cortex

60. Cerebral cortex contains large and giant pyramidal shaped neurons. It is a feature of the cerebral cortex. Discovery of
these cells was connected with the name of following scientist:
A. Bets
B. Purkinje
C. Golgi
D. Nissle`s
E. Paneth`s

61. In human embryo neural tube neuroepithelial ectodermal cells differentiate into neuroblasts and gliablasts. Owing to
migration of these cells some zones (layers) are formed in neural tube. Which of the following zone is where bodies of
neuroblasts locate in?
A. Mantle (intermediate) zone
B. Ventricular zone
C. Marginal zone
D. Outer limiting membrane
E. Inner limiting membrane

62 The increased intraocular tension is observed in the patient with glaucoma. Secretion of aqueous humor by the ciliary
body is normal. Injury of what structure of the eyeball wall caused the disorder of flow-out from the anterior chamber?
A. Venous sinus
B. Ciliary body
C. Choroid
D. Ciliary muscle
E. Posterior epithelium of cornea

63. Stromal compartment of some organ is composed of branch-shaped epithelial cells that form tri-dimensional meshwork.
What is this organ?
A. Thymus gland
B. Red bone marrow
3
C. Spleen
D. Lymph node
E. Tonsil

64. In a patient after transplantation of the foreign kidney rejection of the transplanted organ developed. What main effecter
cells participate in this reaction at first?
A.T-killers (T-cytotoxic)
B. plasma cells
C. T-lymphocytes –suppressors
D. B-cells
E Fibroblasts

65. Examination of a patient who was subjected to X-ray irradiation revealed injury of the splenic white pulp. What cells of
the white pulp undergo to these pathological changes at first?
A. Lymphocytes
B. Basophils
C. Neutrophils
D. Mast cells
E. Red blood cells

66. Morphologic investigation of a spleen revealed activation of immune reactions in organism. Within what splenic
structure antigen dependent proliferation of T-cells, begins
A. Periarterial lymphatic sheath
B. Corona of a lymphoid nodule
C. Marginal zone
D. Germinal center of a lymphoid nodule
E. Red pulp

67. Under influence of unfavorable factors on the organism fast reconstruction of a thymus gland took place. It accompanied
by the massive death of T-cells, its migration into peripheral lymphoid organs and by the proliferation of epithelial reticular
stromal cells. What is the name of that reaction?
A Accidental involution
B Age involution
C. Thymic atrophy
D.Thymic hypotrophy

68 Some organ is presented in histological section. It has cortex and medulla. Cortex is composed of outer cortex with
lymphoid nodules and paracortex. Medulla contains medullary cords, sinuses and trabecules. What organ is it?
A.Lymph node
B.Thymus
D. Kidney
C. Adrenal gland
E. Spleen

69 .At newborn boy congenital involution of a thymus is revealed. Disturbance of what cells differentiation we may wait for?
A. T-cells
B. B-cells
C. Macrophages
D. Plasma cells
E. B-memory cells

70. At a patient with AIDS supression of B-cells differentiation is revealed. In what organ antigen independent differentiation
and proliferation of B-cells takes place?
A. Red bone marrow
B.Lymph node
C.Tonsil
D.Thymus
E. Spleen

4
71. There is congenital atrophy of a thymus at a newborn. Absence of what zone in peripheral lymphatic organs takes place at
this child?
A. Paracortex in lymph node
B. Outer cortex in lymph node
C. Medulla in lymph node
D. Red pulp in spleen
E. Lymphatic nodules in spleen

72. There is displayed an organ of layer’s type structure at the histological section, which is covered by stratified squamous
keratinized epithelium. Under basement membrane of epithelium loose connective tissue, which protrudes in the form of
papillae, is found. The dense irregular connective tissue is located below and forms reticular layer. What organ does have this
morphological feature?
A. Skin
B. Tongue
C. Lingual tonsil
D. Esophagus
E. Uterine cervix

73 The child’s scratch, which had been caused by light skin’s trauma, disappeared after 10 days. What skin layer provided
for reparative regeneration?
A. Stratum basale;
B. Stratum spinosum;
C. Stratum granulosum;
D. Stratum corneum;
E. Stratum brilliantum.

74. It is determined the blood vessel on the histological micropreparation of the spleen. Its wall consists of tunica intima
(endothelium, basal membrane), lack of the distinct tunica media, tunica adventitia (contains collagen and elastic fibers, that
anchor the vessel in the surrounding connective tissue trabeculae. What type of vessel is it?
A. Fibrous vein
B. Muscular vein
C. Arteriole
D. Muscular artery
E. Elastic artery

75. Which of the following is an embryonic origin of vessel wall?


A. Mesenchyme
B. Endoderm
C. Mesoderm
D. Ectoderm
E. Neural tube

76. It is determined the large, oval and round cells with light cytoplasm and a small number of organelles and myofibrils on
histological micropreparation of the heart wall. What are these cells?
A. Purkinje fibers
B. Contractile cardiac muscle
C. Secretory cardiac muscle
D. Smooth muscle
E. P- cells

77. It is the wall of heart on the histological micropreparation. There are contractile, secretory and conductive cardiac
muscle cells and endomysium with blood vessels. For which of the following layers are, these structures belong to?
A. Myocardium of atria
B. Endocardium of ventricles
C. Pericardium
D. Tunica adventitia
E. Epicardium

78. They are determined cardiac muscle cells with glycoprotein rich granules. What are these cells?
A. Secretory
5
B. Contractile
C. P-cells
D. Purkinje cells
E. Transitional cells

79. It was determined the anastomosing in cords one of the layers of cardiovascular system organ. These cords consist of cells
with junctional specializations making up the intercalated disk. What tissue forms this layer?
A. Striated cardiac
B. Striated skeletal
C. Smooth muscle
D. Loose connective
E. Dense connective

80. Myocardiodistrophy promotes a breach of cardiac muscle cells metabolism. Which of the following embryonic origin
breach causes this pathology?
A. Myoepicardial plate
B. Myotome
C. Endoderm
D. Ectoderm
E. Mesenchyme

81. In the histological specimen slender conical projections of the stratified keratinized epithelium and underlining connective
tissue are found, 2-3 mm in length, with their tips pointing toward the one side. Which of the following are in the slide?
A. Filiform papillae of the tongue
B. Circumvallate papillae of the tongue
C. Papillary layer of the dermis
D. Dental papillae
E. Fungiform papillae of the tongue

82. The teen-ager was admitted to an infectious disease hospital with epidemic parotids. The virus of epidemic parotitis
destroys glandular cells of parotid glands. Which of the following cell types undergo destruction?
A. Serous cells
B. Mucous cells
C. Plasma cells
D. Myoepithelial cell
E. Pigment cells

83. The serous cells form the secretory potions of a parotid gland. Which of the following organelles provide synthesis and
secretion of saliva components?
A. Roughed endoplasmic reticulum, Golgi apparatus
B Mitochondria, Golgi apparatus
C. Smooth endoplasmic reticulum
D. Lysosomes
E. Endosomes

84. The patient is 46-years-old man. He complains of increased appetite, heartburn, acid belching, epigastric pain and unstable
stool (constipation mainly). Laboratory examination of gastric juice pH discovers total hyperacidity. Which of the following
cells’ activity is increased?
A. Parietal (oxyntic) cells
B. Chief cells
C. Mucous neck cells
D. APUD cells
E. Paneth`s cells

85. The 5-year-old boy was admitted to the gastroenterologic department of children’s hospital. He has felt acute pain in the
epigastria area just after he drank unknown fluid. Patient complains of nausea, vomiting, cutting stomach pain. Endoscopic
examination of the stomach reveals destruction of the gastric mucous layer. Regeneration of epithelial cell types in the
stomach does come from differentiation of:
A. Mucous neck cells
B. Parietal (oxyntic) cells
6
C. APUD cells
D. Chief cells
E. Surface-lining cells

86. The patient 32-years-old complains of lost appetite, belching, nausea and dull pain in the epigastria region. Laboratory
examination of the gastric juice reveals absence of pepsinogen. Pepsinogen production is associated with which of the
following cell types?
A. Chief cells
B. M-cells
C. Parietal (oxyntic) cells
D. APUD cells
E. Mucous neck cells

87. In the surgical specimen (biopsy) colored by the hematoxylin-eosin you can see large cells with oxyphilic cytoplasm.
Electron microscopy shows system of intracellular canaliculi. What do they produce?
A. Hydrochloric acid (chlorides)
B. Gastrin
C. Pepsinogen
D. Secretin
E. Mucinogen

88. Patient 42-years-old is 2 years after total resection of the stomach. He complains of general weakness, lost appetite,
paleness. Doctor put him diagnosis “Pernicious anemia”. Which of the following cell type’s absence does initiate this
pathology?
A. Parietal (oxyntic) cell
B. Chief cell
C. Mucous neck cell
D. Surface – lining cell
E. APUD cell

89. In a histological specimen of a hollow organ the lining stratified squamous nonkeratinized epithelium is seen in the
mucous. What organ is it?
A. Esophagus
B. Uterus
C. Duodenum
D. Colon
E. Appendix

90. One of the heart layers similar to blood vessel wall by histogenesis and cellular structure. What is the embryonic origin
of this layer?
A. Mesenchyme
B. Splanchnic mesoderm
C. Endoderm
D. Ectoderm
E. Somites

91. Submucous layer of intestine contains serous gland. What part of intestine is in histological slide?
A. Duodenum
B. Coecum
C. Large intestine
D. Ileum
E. Appendix

92. Section of basal portion of intestinal crypts of Lieberkuhn showing the basophilic cells with large acidophylic granules.
What type of cell is it?
A. Paneth`s cells
B. Surface absorptive cells
C. Goblet cells
D. Enteroendocrine cells
E. M-cells
7
93. After examination of a patient with small intestine disease disturbance of terminal digestion was found. Terminal digestion
is associated with which of the following cell type?
A. Surface absorptive cells
B. Goblet cells
C. Enteroendocrine cells
D. M-cells
E. Paneth`s cells

94. In light microscope section you may see epithelial cells are grouped in interconnected plates that form polygonal lobule.
The space between these plates contains sinusoidal capillaries. What organ is presented?
A. Liver
B. Spleen
C. Pancreas
D. Thymus
E. Lymph node

95. Secretion of which gastrointestinal hormones is primarily decreased in a patient with removed duodenum?
A. Cholecystokinin and secretin
B. Gastrin
C. Neurotensin
D. Gastrin and histamine
E. Histamine

96. An attack of bronchial asthma began at a patient at night. Cyanosis and dyspnea are observed at a patient. What parts of
the bronchial tree participate in this pathologic process?
A. Small (intralobular) bronchi
B. Primary ( large, extrapulmonary) bronchi
C. Medium (lobar) bronchi
D. Terminal bronchiole
E. Alveoli.

97. Autopsy of a man who suffered from pulmonary disease revealed pathologic process in bronchi. Histological
investigation reveled presence of glands, cartilage plates and simple columnar pseudostratified epithelium. Into what bronchi
type changes were observed?
A. Medium (lobar) bronchi
B. Principal (large, extrapulmonary) bronchi
C. Small (intralobular) bronchi
D. Terminal bronchioles
E. Alveoli

98. After histological investigation of lungs at a patient who suffered from pneumonia. Injury of alveolar cells that are
responsible for respiratory function was observed. What cells are these?
A. Pneumocyte type 1
B. Pneumocyte type 2
C. Alveolar macrophages
D. Clara cells
E. Fibroblasts

99. After histological examination of the respiratory system some tubular organ is revealed. In the wall of this organ
epithelium is not tall, smooth muscle layer is very thick, absence of glands and cartilage. Name this organ.
A. Small (intralobular) bronchi
B. Larynx
C. Principal (large, extrapulmonary) bronchi
D. Trachea
E. Alveoli

100. Lung of premature infant is presented on electron micrograph of biopsy material. Collapse of the alveolar wall caused
by the deficiency of surfactant was revealed. Which cell type dysfunction of the alveolar wall has been caused it?
A. Pneumocyte type 2
8
B. Fibroblasts
C. Alveolar macrophages
D. Secretory cells
E. Pneumocyte type 1

101. Characterizing a stress, the student has supposed an inaccuracy when has told, that synthesis of glucocorticoids of a
suprarenal cortex is stimulated by hormones of a pituitary gland. What improvement is necessary?
A. Аdrenocorticotropic hormon (ACTH)
B. Somatotropin
C. Prolactin
D. Gonadotropic hormones
E. Тhyrotropic hormone

102. At the woman 30 years old have revealed a shortage of sexual hormones, have found an increase level of follicle
stimulating hormone. What cells of an Adenohypophysis will be modified thus?
A. Gonadotropes
B. Thyrotropes
C. Mammotropes
D. Corticotropes
E. Somatotropes

103. At the patient who during 7 years suffers a hypothyroidism, failure of thyroid hormones is detected. What cells of an
Adenohypophysis thus will be modified?
A.Thyrotropes
B. Gonadotropes
C. Mammotropes
D. Corticotropes
E. Somatotropes

104. The removing of an endocrine gland of experimental animals occurs a premature puberty. Which of the following gland
is removing can occur a premature puberty?
A. A pineal gland
B. A pituitary gland
C. Adrenal gland
D. A thyroid gland
E. Parathyroid gland

105. During a puberty man's sexual glands produce to the blood man's sexual hormone testosterone which is responsible for
development of the secondary sexual features.. What cells of testes synthesize this hormone?
A. Leydig cells
B. Sertoli cells
C. Spermatozoons
D. Fibroblasts
E. Spermatids

106. It was determined, that the thyroid follicles are enlarged, owing to increases of follicular epithelial height and increased
thyroglobulin deposits on the micropreparation of the thyroid gland. Which functional state is the following histological
picture characteristic of?
A. Hyper function
B. Hypo function
C. Moderate activity
D. Age changes
E. Rise of iodine level the blood plasma

107. A 2-year- old child experienced convulsions because of lowering calcium ions concentration in the blood plasma.
Function of what structure is decreased?
A. Parathyroid glands
B. Hypophysis
C. Pineal gland
D. Thymus
9
E. Adrenal cortex

108. Usage of oral contraceptives with sex hormones inhibits secretion of the hypophyseal hormones. Secretion of which of
the indicated hormones is inhibited while using oral contraceptives with sex hormones.
A. Follicle-stimulating
B. Oxytocin
C. Vasopressin
D. Somatotropic
E. Thyrotropic

109. Upward invagination of ectoderm lining primitive oral cavity forms the Rathke’s pouch. Which of the following
structures does develop from this embryonic origin?
A. Adenohypophysis
B. Median eminence
C. Neurohypophysis
D. Neural stark
E. Pars distalis

110. Patient with Cushing syndrome (high level of glucocorticoids in the blood) has the high blood sugar. Which of the
following organ pathology is the cause of this disease?
A. Adrenal cortex
B. Adrenal medulla
C. Pancreas
D. Pineal gland
E. Thyroid gland

111. It is known, that aldosterone controls water and electrolyte balance in the organism. Which of the following cell types
of adrenal gland produce this hormone?
A. Cells of Zona glomerulosa
B. Cells of Zona reticularis
C. Epinephrine- secreting cells
D Cells of Zona fasciculata
E. Norepinephrine-secreting cells

113. The operation of thyroid gland sectoral ectomy caused spastic muscular contraction of a 30- year old woman. It was
determined the low blood calcium. Lack of what hormone secretion disrupts neuromuscular function?
A. Parathyroid hormone
B. Thyroxin
C. Melatonine
D. Somatotropin
E. Antidiuretic hormone

114. Patient had been receiving the great doses of hydrocortisone for a long time. Which of the following cells of adrenal
cortex must be atrophied after the influence of this exogenous hormone?
A. Cells of zona fasciculata
B. Cells of zona reticularis
C. Cells of adrenal medulla
D. Cells of zona glomerulosa
E. Cells of capsule

115. Arterial hypertension is caused by the stenosis of the renal arteries in the patient. Activation of what system is the main
link in the pathogenesis of this form of hypertension.
A. Renin-angiotensin
B. Hypothalamic-pituitary
C. Parasympathetic
D. Sympathoadreal
E. Kallikrein-kinin

116. In a patient with chronic glomerulonephritis acidity of urine does not occur and bacteriostatic effect is absent. Which
structures of collecting tubules are associated with hydrochloric acid production?
10
A. Dark cells
B. Glomerulus
C. Podocytes of capsule
D. Proximal tubule
E. Distal tubule

117. The electron microscopic photograph of a kidney’s fragment shows afferent arteriole. Under the endothelium of arteriole,
large cells are seen with secretory granules in the cytoplasm. Name this type of cells:
A. Juxtaglomerullar cells
B. Intraglomerullar mesangial cells
C. Extraglomrullar cells
D. Smooth muscle cells
E. Interstitial cells

118. The electron microscopic photograph of renal corpuscle demonstrates branched cells between capillaries of glomeruli.
Large number of filaments is seen in the cytoplasm of these cells. Name this type of cells.
A. Mesangial cells
B. Jucstaglomerullar cells
C. Podocytes
D. Smooth muscular cells
E. Interstitial cells

119. The patient with glomerulonephritis complains of high blood pressure caused pathological process of nephrons. Which
of the following vessels of the kidney was distressed?
A. Fenestrated capillaries
B. Continuous capillaries
C. Discontinuous capillaries
D. Fibrous veins
E. Elastic artery

120. A 10-14 % of newborn suffer from developmental anomalies. Which embryonic origin urinary system develops from?
A. Intermediate mesoderm
B. Dorsal mesoderm
C. Visceral layer of ventral mesoderm
D. Parietal layer of ventral mesoderm
E. Mesenchyme

121. Slow healing of an umbilicus is caused by patent primitive urinary duct - urachus. Which embryonic origin is urinary
bladder and urachus develop from?
A. Allantois
B. Cloacae
C. Urogenital sinus
D. Paramesonephric duct
E. Mesonephric duct

122. Clinical examination of the 35-year-old women with kidney’s disease reveals presence of blood cells and fibrinogen in
urine that says about distressed filtration barrier. Renal filter consists of:
A. Endothelium, podocyte, basal membrane
B. Podocyte, basal membrane
C. Only basal membrane
D. Endothelium, basal membrane
E. Endothelium, podocyte

123. Electron microscopic photograph of a renal corpuscle shows the large cell with large and numerous small processes.
Small processes attach the basal membrane of fenestrated capillaries. Name this cell.
A. Podocyte
B. Juxtavascular cell
C. Smooth muscular cell
D. Endothelial cell
E. Mesangial cell
11
124. In microslide the muscle - gland organ, which has a lobules, is introduced. In lobules glands are posed, ducts are
unclosed in the large canal located at the center of a organ, the wall of this canal is covered by a pseudostratified colomnar
epithelium. Stroma of the organ compound of loose connective and muscle tissues. What organ is it?
A. A prostate
B. A uterus
C. A seminal vesicles
D. An epididymis
E. A mammary gland

125.During a puberty man's sexual glands produce in a blood man's sexual hormone testosterone which is responsible for
development of the secondary sexual features.. What cells of spermaries synthesize this hormone?
A. Leydig cells
B. Sertoli cells
C. Spermatozoons
D. Fibroblasts
E. Spermatids

126. In histological micropreparation there is an organ, which is covered by tunica vaginalis and tunica albuginea. Stroma of
the organ consists of connective tissue with the Leydig’s cells. Parenchyma contains tubules, which are lined by the
spermatogeneous epithelium. What organ is there?
A. Testis
B. Prostate gland
C. Epididymis
D. Mammary gland
E. Ovary

127. Patient complains of frequent and difficult urination. Imperfection of what formation can cause it.
A. Prostate
B. Epedidimis
C. Bulb urethral glands
D. Sperm bubbles
E. Testicles

128. In the ovary specimen colored with hematoxylin-eosin, follicle is determined where cubic-shaped follicle epithelium
cells are placed in 1-2 layers, and scarlet covering is seen around ovocyte. Name this follicle:
A. Primary
B. Secondary
C. Mature
D. Atretic
E. Primordial

129. At the histological section of ovary stained by hematoxylin and eosin one can determine follicle consists of 1-2 layers
cuboidal follicular cells. A blazing–red coat covers oocyte. Name this follicle.
A. Primary follicle;
B. Primordial follicle;
C. Secondary follicle;
D. Graafian (mature) follicle;
E. Atretic follicle.

130. There are lot of large branched-shaped cells found in the myometrium, which contain smooth myofillaments in the
cytoplasm, and well-developed rER. What are these cells?
A. Smooth muscle cells;
B. Myoepithelial cells;
C. Skeletal muscle fibers;
D. Fibrocytes;
E. Myofibroblasts.

131. In the microslide of ovary a spherical structure 5 sm in diameter is revealed. Its large cells contain yellow pigment and
lipid droplets. From what type of cells this ovarian structure is composed of?
12
A. Lutein cells;
B. Myoid cells;
C. Follicular cells;
D. Interstitial cells;
E. Fibroblasts.

132. The pregnant woman was a subject of caesarean section operation with the aim of fetus extraction. The uterus wall was
sected at the long distance. By what means will a healing in the myometrium stitch take place?
A. Formation of connective tissue cicatrix
B. Hypertrophy of smooth muscle cells
C. New formation of smooth muscle tissue
D. Formation of skeletal muscle fibers
E. Proliferation of satellite cells

133. A 34-years-old patient was admitted to a hospital after significant hemorrhage. Decreased blood volume promotes
decreased blood pressure and activates the renin-angiotensin-II- aldosterone mechanism that contributes to the maintenance of
blood pressure in a patient’s organism. Which of the following cells are producing renin?
A. Juxtamedullary cells
B. Juxtaglomerular cells
C. Light cells of collecting tubule
D. Interstitial cells
E. Dark cells of collecting tubule

134. A girl of 5-years-old has admitted to nephrologic department of a municipal hospital. She complains of pain in low
back, urine contains constantly raised amount of red blood cells (erythrocytes), proteins (albumins and globulins). Which
from the following structures is destroyed?
A. Glomerulus of nephrons
B. Proximal convoluted tubule of nephron
C. Distal convoluted tubule of nephron
D. Mesangium of glomerulus
E. Collecting tubule of nephron

135. A woman complains of excessive menstruation, menstrual colic usually. Ultrasound investigation of the uterus shows
16 mm thickness of endometrium (N 8-9mm) 1day before menstruation. Which from the following cells are producing an
increased level of estrogen and are stimulating overgrowth of endometrium?
A. Cells of granulosa in secondary and Graafian follicles
B. Endocrine cells of Adenohypophysis
C. Endocrine cells of pars nervosa
D. Cells of adrenal cortex
E. Cells of adrenal medulla

136. A patient has observed by a doctor at a polyclinic. Patient complains of acute pain in epigastria region after he’s taken
a tablet of aspirin. Aspirin can disrupt the epithelial layer of stomach and initiate the ulceration. Which type of cells can be
destroyed at first?
A. Surface- lining cells
B. Mucous neck cells
C. DNES cells
D. Parietal cells
E. Chief cells

137. A 2-years-old boy was admitted to the hospital with malabsorption syndrome. Deficiencies of disaccharides and
dipeptidases have been found in this patient. What from the following types of epithelium is providing deficiency of
enzymes?
A. Simple columnar epithelium
B. Simple cuboidal epithelium
C. Simple squamous epithelium
D. Pseudostratified epithelium
E. Simple columnar ciliated epithelium

13
138. In the histological specimen an organ is seen. Wall of this organ consists of 4 layers: first is mucous layer forming pits
and glands; second is submucous layer of connective tissue; third is muscularis externa containing tree layers of smooth
muscles; fourth is serous layer. Which organ can be recognized in the specimen?
A. A stomach
B. A jejunum
C. A duodenum
D. An esophagus
E. A colon

139. At a vulgar pemphigus in a skin epidermis forms vesicles, which breaks out a wholeness of stratum spinosum and
stratum granulosum. What function of an epithelium is broken first?
A. Barrier
B. Regenerative
C. Absorptive
D. Secretory
E. Endocrine

140. Under the influence of long vibration and a motion in motor regions of cortex brain occurs a gliolis – phagocytosis of
damaged neurons by a neuroglial cells. What neuroglia participates in this process?
A. Microglia cells
B. Protoplasmatic astrocytes
C. Oligodendrocytes
D. Ependymal cells
E. Fibrous astrocytes

141. At a person genetic disorder of metabolism of amino acid tyrosine broke synthesis of a black pigment melanin. In what
cells of loose connective tissue and a skin epidermis lack of this pigment will be observed?
A. Pigment cells
B. Dendritic cells
C. Adipose cells
D. Mast cells
E. Pericytes

142. At a combustion of a skin the patient feels a strong pain. Processes of what neurons first conduct pain nervous impulse in
central nervous system.
A. Pseudounipolar neurons
B. Bipolar neurons
C. Unipolar neurons
D. Multipolar neurons
E. Purkinje neurons

143. After two months of appendectomy the patient again has arrived in surgical department with the diagnosis: «The
intestinal obstruction, an adhesive disease». During operation damage of what epithelium covering a peritoneum has reduced
in development of adhesive disease?
A. Mesothelium
B. Endothelium
C. Transitional epithelium
D. Simple columnar epithelium
E. Simple cuboidal epithelium

144. During road accident at the man of 30 years the femoral artery has been injured. In hospital the arteriorraphy was
performed. Regeneration of what muscle tissue is necessary for regeneration of a wholeness of the vessel’s tunica media.
A. Smooth muscle tissue
B. Striated skeletal muscle tissue
C. Striated cardiac muscle tissue
D. Myoepithelial cells
E. Satellite cells
14
147. On microslide of the bulb of the fetus eye a damage of a cornea is observed. Part of what germ layer has been struck
during embryonic development
A. Ectoderm
B. Endoderm
C. Mesoderm
D. Dermatome
E. Nephrotome

148. At examination, replying about development of hard and soft tissues of dens, the student has made a mistake, when has
told, that enamel is formed of a mesenchyme cells. What should be a right answer?
A. From inner enamel epithelium
B. From stellate reticulum
C. From outer enamel epithelium
D. From dental sac
E. From dental papilla

149. At a brain injury are defected glial cells, which meet in gray matter to central nervous system more often. What is the
name of these cells?
A. Protoplasmic astrocytes
B. Oligodendrocytes
C. Microglia
D. Fibrous astrocytes
E. Ependymal cells

150. As a result of the transferred orchitis (an inflammation of a testis) at the man of 43 years old the spermatozoons
producion was broken. In what testis structures pathological changes were formed?
A. Seminiferous tubules
B. Rete testis
C. Tubuli seminiferi recti
D. Ductuli efferentes testis
E. Ductus deferens

151. During an embryogenesis there was a breaking shaping of a pharyngeal gut. Specify possible localization of anomalies of
development?
A. Organs of an oral cavity.
B. A stomach
C. An ileum
D. A pancreas
E. A jejunum

152. The neonatal child has a disorder of a thymus gland. What type of a hemopoiesis will be disturbed?
A. A lymphopoiesis
B. A monocytopoiesis
C. An erythropoiesis
D. Granulocytopoiesis
E. Platelet formation

157. In chronically diseased liver some cells proliferate and acquire the feature of myofibroblasts with or without the lipid
droplets. These cells are found close to the damaged hepatocytes and play a major role in development of fibrosis. What are
these cells?
A. Ito’s cells
B. Hepatocyte
C. Kupffer cells
D. Pit cells
E. Adipose cells

15
158. One of the principal types of diabetes (type 1) is an autoimmune disease in which antibodies against endocrine cells of
pancreas (islets of Langhergans) and depresses activity of cells. Which cells activity is depressed?
A. B-cells
B. A-cells
C. D-cells
D. PP-cells
E. D1-cells

159. Examination of the patient with pernicious anemia revealed antibodies against cell’s proteins of gastric glands. What
kind of cells in gastric glands was destroyed?
A. Parietal cells
B. Mucous cells
C. Chief cells
D. DNES-cells
E. Surface- lining cells

160. The epithelial enamel organ is observed in maxilla’s histological section of 3.5-month human fetus. Mesenchyme cells
surround this enamel organ. What is name of these cells?
A. Dental sac
B. Odontoblasts
C. External enamel cells
D. Internal enamel cells
E. Stellate reticulum

161. The histological slide shows an organ of oral cavity consisting of tree principal parts: cutaneus, intermediate, mucous.
What organ is described here?
A. Lip
B. Gum
C. Hard palate
D. Soft palate
E. Cheek

163. In the histological specimen of an organ numerous lymphocytes in periphery of each lobule and stromal reticular
epithelial cells can be observed. Which of the following hematopoietic tissue or organ develops from endoderm?
A. Thymus
B. Tonsils
C. Bone marrow
D. Spleen
E. Blood islands

165. The cystoscopy of a patient with cystic tumor revealed the area of mucous layer that is without folds.
Which of the following parts of urinary bladder was observed?
A. Fundus
B. Body
C. Cervix
D. Apex
E. All mucous layer

166. Hematuria (damaged erythrocytes) is detected in the urine of a patient with glomerulonephritis. Which part of nephron is
destroyed in this patient?
A. Bowmen’s capsule
B. Proximal convoluted tubule
C. Distal convoluted tubule
D. Collecting tubule
E. Henle’s loop

171. Scientists marked developing B lymphocytes of the red bone marrow by radioactive label. In what regions of
peripheral hematopoietic organs this label may be determined at first?
A. Outer cortex and medulla of lymph node
16
B. Paracortex of lymph node
C. PALS of spleen
D. Splenic red pulp cords
E. Splenic sinusoids

172.Scientiests marked developing T lymphocytes of thymus gland by the radioactive label. In what regions of peripheral
hematopoietic organs this label may be determined at first?
A. Paracortex of lymph node, PALS of spleen
B. Medulla of lymph node
C. Outer cortex of lymph node
D. Splenic red pulp cords
E. Splenic sinusoids

173. After overdoses of -rays destruction of B-lymphocytes in human hematopoietic organs happens. Reduction of what
blood plasma substances will be observes in this case.
A. Immunoglobulins (antibodies)
B. Albumins
C. Cholesterol
D. Glucose
E. Fibrinogen

174. A patient with leucosis has disturbance of some type of hematopoiesis. This type has next characteristic: antigen
independent and antigen dependent phases, blastic transformation, clonal expansion, recirculation. What type of
hematopoiesis has enumerated features?
A. Lymphocytopoiesis
B. Granulopoiesis
C. Erythropoiesis
D. Monopoiesis
E.Thrombopoiesis

175.A patient suffers from epithelial tumor of the right extrapulmonary bronchus. What morphologic type of epithelium gave
rise to tumor?
A. Simple columnar pseudostratified
B. Simple squamous
C. Simple cuboidal
D. Stratified squamous nonkeratinized
E. Transitional

176. Wife and husband complained of children birth impossibility. Injury of spermatogenic testicular epithelium was revealed
after examination. Due to this absence of spermatozoa in seminal fluid and male infertility took place. Injury of what testicular
regions took place in this case?
A. Seminiferous tubules
B. Straight tubules
C. Rete testis
D. Ductus epididymis
E. Efferent ductules

177. After gynecologic operation woman complained of pain in lower abdominal region and vaginal discharge. Gynecologist
made a diagnosis - inflammation of inner uterine layer. Name this layer.
A. Endometrium
B. Myometrium
C. Perimetrium
D. Parametrium
E. Peritoneum

179. A dentist revealed abnormal development of the enamel at a patient. What components of the enamel organ were injured
at an embryo?
A. Outer epithelium of enamel organ
B. Inner epithelium of enamel organ
C. Stellate reticulum of enamel organ
17
D. Dental papilla
E. Dental sac

180. Injury of superficial mesenchyme cells of enamel organ’s dental papilla took place. Disturbance in formation of what
dental tissue can wait for?
A. Dental dentin
B. Dental enamel
C. Dental pulp
D. Dental cementum
E. Cuticle of enamel

181. Toxin of virus caused injury of cells of enamel organs dental sac. Disturbance in formation of what dental tissue can
wait for?
A. Dental cementum
B. Dentin
C. Dental pulp
D. Enamel
E. Cuticle of enamel

182. A patient is ill with cerebellum tumor. Due to this shakiness of the step is developed. What Cerebella cortex cells
provide equilibrium in normal states at first?
A. Purkinje cells
B. Small pyramidal cells
C. Basket cells
D. Large pyramidal cells
E. Golgi cells

183. Some sensory organ is presented on the histological section. Main morphologic component of it is a chain that
composed of 3 neurons. Name this structure.
A. Retina of the eye
B. Cornea of the eye
C. Organ of Corti
D. Organ of equilibrium
E. Organ of taste

184. Toxin of virus caused death of pseudounipolar neurons. Due to this loss of sensitivity took place at a patient.
Disturbance of what organ function is observed at this patient.
A. Dorsal root ganglion
B. Spinal cord
C. Cerebral cortex
D. Cerebellar cortex
E. Sympathetic ganglion

185. Death of ciliated epithelial cells in bronchi took place at a worker of chemical concern after inspiration of hydrochloric
steams. After 30 days, normal structure of conducting portion epithelium was restored. What epithelial cells in respiratory
epithelium took part in regeneration?
A. Basal cells
B. Goblet cells
C. Clara cells
D. Endocrine cells
E. Brush cells

186. Enteroendocrine sells differ from goblet cells in:


A. The direction of release of secretion
B. The use of exocytosis for release of secretary product from the cell
C. The presence in the small and large intestine
D. Secretion by the regulated pathway
E. They origin from a crypt stem cell

18
187. Parents brought a one-month-old male infant to the emergency room by his. The examining emergency room physician
notes that his skin and sclera are icteric. A blood test indicates elevated unconjugated bilirubin in the serum. The elevated
bilirubin levels in this patient are most likely the result of
A. Deficiency of hepatocyte enzymes regulating bilirubin solubility
B. Decreased destruction of red blood cells
C. Increased function of Ito-cells
D. Decreased function of Pit-cells
E. Deficiency of Kupffer- cells

188. The retinal pigment epithelium is characterized by


A. The presence of the photoreceptor (rod and cone) perikaria
B. Phagocytosis of worn-out components of photoreceptor cells
C. Origin from the inner layer of optic cup during embryonic development
D. Presence of amacrine cells
E. Synthesis of vitreous humor

192. Deficiency of vitamin A causes a breach of vision in twilight. Which of the following cells carry out this function?
A. Rods
B. Cones
C. Bipolar cells
D. Amacrine cells
E. Ganglion cells

193. There is connecting with the gut vesicle is visible on the histological Micropreparations. It is an extraembryonic organ.
Primordial germ cells and erythroblasts (megaloblasts) are located in its wall. What organ is this?
A. Yolk sac
B. Allantois
C. Placenta
D. Umbilical cord
E. Amnion

194. It was determined the fall of activity of amylase in saliva owing to investigation of 47- years- old man with extracted
salivary gland. What gland was extracted?
A. Parotid
B. Buccal
C. Submandibular
D. Palatinar
E. Submandibular

197. Which of the following is found exclusively in the renal medulla?


A. Thin loops of Henle
B. Distal convoluted tubules
C. Collecting ducts
D. Afferent arterioles
E. Proximal convoluted tubules

198. A 33-year-old patient with an average menstrual cycle of 28 days comes in for a routine Pap smear. It has been 35 days
since the start of her last menstrual period, and a vaginal smear reveals clumps of basophilic cells. As her physician, you
suspect
A. There are detectable levels of hCG in her serum and urine
B. She will ovulate within a few days
C. Her serum progesterone levels are very low
D. She will begin menstruating in a few days
E. She is undergoing menopause

199. Synthesis of milk by the mammary gland specifically requires


A. Prolactin
B. Production of progesteron by the corpus luteum
C. The influence of vasopressin
D. Placental lactogen
19
E. Neurohumoral reflexes

201.A histological spacemen presents parenchymal organ, which has cortex and medulla. Cortex consists of epitheliocytes bars,
between them there are blood capillaries; the bars form three zones. Medulla consists of chromafinocytes and venous sinusoids.
What organ has these morphological features?
A. Adrenal gland
B. Kidney
C. Lymph node
D. Thyroid
E. Thymus

202.A 39-year-old patient after radiotherapy because of hepatoma developed ulcer of small intestine. It was caused by the
inhibition of mitotic activity of the cells, which are responsible for regeneration of small intestine surface epithelium. Inhibition
of what cells mitotic activity does this patient have?
A. Regenerative cells
B. Paneth`s cells
C. Surface absorptive cells
D. DNES-cells
E. Goblet cells

204. The electronic microphotograph of kidney fragment has demonstrated the afferent glomerular arteriole, which under its
endothelium has giant cells, containing secretory granules. Name the type of these cells:
A. Juxtaglomerular
B. Interstitial
C. Mesangial
D. Juxtavascular
E. Smooth muscular

207.A patient complaining of weight loss (10 kg during 2 months), palpitation and exophthalmia came to the endocrinologist.
For the hyper function of what endocrine gland (glands) are these complaints the most typical?
A. Thyroid
B. Pancreas
C. Adrenal glands
D. Parathyroid glands
E. Ovaries

210.In the microspecimen of red bone marrow there were revealed multiple capillaries through the walls of which mature blood
cells penetrated. What type of capillaries is it?
A. Sinusoidal
B. Somatic
C. Fenestrated
D. Lymphatic
E. Visceral

212. Decreased blood supply to the organs causes hypoxia that activates fibroblast function. Volume of what elements is
increased in this case?
A. Exracellular matrix
B. Nerve elements
C. Parenchymatous elements of the organ
D. Lymphatic vessels
E. Vessels of microcirculatory bed

217. Careless student occasionally met his dean, The concentration of what hormone will most likely increase in the blood of
the student?
A. Epinephrine
B. Corticotropic
C. Thyrotropin-releasing hormone
D. Cortisol
20
E. Somatotropin

211.The increased intraocular tension is observed in the patient with glaucoma. Secretion of aqueous humor by the ciliar body is
normal. Injury of what structure of the eyeball wall caused the disorder of fluid flow-out from the anterior chamber.
A. Venous sinus
B. Back epithelium of cornea
C. Choroid
D. Ciliar body
E. Ciliary muscle

21
1. Vasopressin and oxytocin are - small peptides , secreted by neuron
2. Name the substances that do not pass through the filtration barrier of the kidney -
fibrinogen , immunoglobulin
3. In what structures of the kidney does water reabsorption occur - proximal convulated
tubule , descending limb of henles loop , collecting ducts
4. List hormones of kidney - erythropoietin , renin
5. A renal lobule consists of - loop of henle , renal pyramid
6. The colloid in thyroid follicles - contains thyroglobulin , contains glycoproteins
7. Red bone marrow in adults is present in the - sternum , vertebrae , ribs , flat bones , long
bones
8. Name the sources of development of the epithelium of the digestive tract - ectoderm ,
endoderm
9. What cell are in acini of parotid gland - serous , myoepithelial
10. What are the enzymes of pancreatic juice involved in the breakdown of proteins - trypsin
, chymotrypsin
11. Name the structures that are related to the indifferent gonad - ductules of mesonephrons
, mesenchyme
12. Name the structures from which the uterus develops - paramesonephric ducts ,
mesenchyme
13. Name the types of follicles containing oocytes with a diploid set of chromosomes -
primary follicle , primordial follicle
14. What are the structures of the ovary that produce estrogen - cells of the theca interna of
follicle , theca luteocytes of the corpus luteum
15. The thyroid glands - stores thyroglobulin in extracellular loci , secrete hormones that
help control basal metabolic rate , functions under the influence of an hypophyseal
hormone
16. Oxyphil cells of parathyroid glands - are strongly acidophilic , have large accumulations
of mitochondria
17. The smooth muscles of arterial walls - is responsible for vasocontriction or vasodilation
in response to drugs , is in a state of tonus to help maintain the diameter of the vessel ,
is innervated by the sympathetic nervous system , helps maintain blood pressure
18. The cuticle of hair is compose of cells that are - colorless , strongly keratinized ,
flattened and overlapping
19. The parathyroid glands are - essential to life , encapsulated , pharyngeal derivatives ,
usually found embedded within thyroid glands , typically four in total
20. Upward invagination of ectoderm lining primitive oral cavity forms the rathkes pouch.
Which of the following structures does develop from this embryonic origin -
adenohypophysis , pars distalis
21. Thick skin is found on the - palms of the hands , soles of the feet
22. Reticulocytes are - found in peripheral blood , stained by cresyl blue because of
cytoplasmic ribosomal RNA
23. The thyroid gland secrets into the blood - T4 , T3 , calcitonin
24. Type T lymphocytes - arise from stem cells in the thymus , can be converted to
lymphoblasts that can divide , are cytotoxic to foreign cells
25. The white pulp of the spleen contains - lymphatic nodules , central arteries
26. The cells of the adrenal cortex are - of mesodermal origin , producers of steroid
hormones , rich in mitochondria with tubular cristae
27. Name the embryonic sources from which the uterus develops - mesenchyme ,
paramesonephral ducts
28. Name the cells of the endometrial epithelial lining - ciliated , secretory
29. Name the cells of the epithelium of epididymis - basal , principal
30. Name epithelial cells of the nasal cavity olfactory portion - supporting, receptor
31. Name the epithelial cells of the terminal bronchiole - ciliated cell , clara cell
32. Name the tissue that forms the muscular layer of the esophagus in the middle third -
smooth , striated visceral
33. What are the function of the large intestine - formation of vitamin B , water absorption
34. What organs are lined with simple columnar epithelium - intestine , stomach
35. Name the gastric wall tunic developing from the visceral layer of the splanchnotome -
serosa
36. What are the enzymes of intestinal juice involved in the breakdown of proteins -
peptidase , pepsin , elastase
37. On what basis can you distinguish the adventitia from the serous membrane - lack of
mesothelium
38. During gastroscopy a patient has an insufficient amount of mucus that covers the
mucous membrane.which cells of the stomach wall is dysfunctional - neck cells
39. The space of disse contains - fat storing cells , blood , microvilli
40. What cell are in acini of parotid gland - myoepithelial , serous
41. The bile canaliculi are - enclosed by tight junctions , formed from modifications of the
apical surfaces , rich in ATP-ase activity
42. Bile canaliculi - are formed from modified plasma membranes o adjacent hepatocytes ,
have microvilli projecting into their lumen , are lined by cuboidal epithelial cells
43. What are the functions of the epididymis duct - promotes the motor activity of sperm ,
maturation of sperm
44. The functions of the colon include - formation of feces , water absorption , production of
mucus
45. Devices to increase surface area for absorption in the small intestine include - microvilli ,
villi
46. The tongue has - intrinsic mucous and serous salivary gland , a covering of stratified
squamous epithelium , an abundance of nerves and blood vessels , interlaced striated
muscle bundles oriented in several directions , collection of lymph nodules
47. What type of connective tissue in the periodontal space - dense regular , loose irregular
48. Absorptive cells of the small intestine when examined by transmission electron
microscopy are seen to possess - secretory granules , stereocilia ,chylomicra , microvilli
with glycocalyx

NEWLY ADDED MCQ

1. In the histopreparation of the pyloric part of the stomach,consisting of fusiform cells


containing a rod shaped nucleus in the center of the cytoplasm . what kind of tissue do
the cells form - smooth muscle
2. A 55 year old patient is observed by an endocrinologist for a violation of the endocrine
function of the pancreas which is manifested by a decrease in the amount of the
hormone glucagon in the blood the function of which cell is impaired in this case - A-cells
islets of langerhans
3. Choose a feature of the structure of the dorsal surface of the tongue - lack of mucosa
4. What is the time of the beginning of prenatal tooth histogenesis - 6 week
5. In the biopsy material of the patients stomach, histological examination revealed a
significant decrease or complete absence of parietal cells in the glands . what part of the
stomach was the mucous membrane studied - pyloric part
6. Submucosa of intestine contains serous gland . what part of intestine is in histological
slide - duodenum
7. What epithelium is a part of tonsils - stratified squamous non keratinizing
8. Name the area of hypomineralized dentin - circumpulpal dentin
9. With the proliferation of connective tissue in the liver parenchyma due to chronic disease
there is a violation of blood circulation in the classic lobules what is the direction of
movement of blood in such lobules - from the periphery of the center
10. The parotid gland has end section formed by serocytes. What organelles of these cells
provide for the synthesis and secretion of saliva components - RER
11. In a histological specimen the parenchyma of the organ is represented by structures that
have the shape of hexagonal prism and consist of anastomosing plates, between which
lie sinusoidal capillaries radially converging to thee central vein what anatomical organs
does this morphological structure have - liver
12. Select the function for the hepatocyte granular endoplasmic reticulum - synthesis of
blood plasma proteins
13. Name the pancreatuc islet endocrine cells which inhibits activity of acinar cells of
pancreas - PPcells
14. What is the structure of the parotid gland - complex alveolar tubular branched
15. With acute inflamation of the parotid salivary gland the processes of secretion and
excretion are disrupted what cells are affected - serous ,myoepithelial cells
16. What hormone increases blood glucose levels has a hyperglycemic effect - glucagon
17. What does the apical part of the acinocyte contain and what is it called - zymogenic ,
zymogenic granules
18. For microscopic examination striated muscle tissue of an organ of the digestive system
is presented what organ was the biopsy taken from - stomach
19. What substance is the main constituent of the parotid saliva -amylase
20. For the pathological examination a malignant tumor is presented that has developed
from the striated muscle tissue of the digestive system what organ was the biopsy taken
from - upper third of the esophagus
21. Name the source of the development of cementoblasts - mesenchyme of dental sac
inner layer
22. The interlobular excretory duct of the pancreas is lined with epithelium - simple columnar
23. The macrophage in the liver includes - hepatocytes
24. Name the most numerous cells of the pancreatic islet - Bcells
25. The histopreparation presents an organ of the digestive tract, the wall of which consists
of 4 membranes mucous,submucosa,muscular and serous . the mucous membrane has
folds and pits. Determine which organ has a given relief - stomach
26. A dentist revealed abnormal development of the enamel at a patient . what components
of the enamel organ were injured at embryo - outer epithelium of enamel organ
27. Morphological analysis of the biopsy material of the esophageal mucus taken from the
patient revealed the process of epithelial keratinization. Which of the following types of
epithelium covers the lining of the esophagus - striated squamous keratinized
28. What is the epithelium lining the nasopharynx - psudostratified columnar ciliated
29. One of the organs of the oral cavity is a few folds of the mucous membrane , in the
lamina propria of which there are numerous lymphoid follicles. What organ is it - palatine
tonsil
30. What are the disse spaces located - between the walls of hemocapillaries and
hepatocytes
31. Which hormone of pancreatic islet endocrine cells participate in the breakdown of
glycogen - glucagon
32. Due to the developmental abnormality in the newborn , a violation of the formation of the
large salivary glands was revealed . violation of what embryonic sources is this anomaly
caused - ectoderm
33. In hepatocytes two poles are distinguished biliary and vascular. The biliary pole of
hepatocytes is the pole with which the hepatocytes face each other and they form a kind
of liver structure what is this structure - bile canaliculi
34. Name the cells of the developing tooth,the source of which is the ectoderm - ameloblasts
35. Which of the following salivary glands contains the most mucous acini - sublingual
36. The interlobular excretory duct of the pancreas is lined with epithelium - simple
columnar.
37. Name the embryonic source of the pancreatic capsule development - mesenchyme
38. The histoprepration presents an organ of the digestive system , which has a strongly
thickened lamina of the mucous membrane of its own containing together with the
submucosa numerous lymphoid nodules in which organ do lymphoid nodules occupy the
largest volume in relation to the thickness of the entire wall - appendix
39. The teen ager was admitted to an infectious disease hospital with epidemic parotitis. The
virus of epidemic parotitis destroys cells of parotid gland which of the following cell type
undergo destruction - serous cells
40. Parents brought a one month old male infant to the emergency room by his the
examining emergency room physician notes that his skin and sclera are icteric. A blood
test indicate elevated unjunctioned bilirubin in the serum . the elevated bilirubin levels in
this patient are most likely the result of - deficiency of hepatocytes enzyme regulating
bilirubin solubility
41. Injury of superficial mesenchyme cells of enamel organ dental papilla took place.
Distribution in formation of what dental tissue can wait for - dental dentin
42. In the process of embryogenesis there was a violation of the formation of the anterior
section of the primary intestine. Indicate the possible localization of developmental
anomalies - jejunum
43. Name the type of secretion of the pancreas acinar cells - merocrine
44. The wall of the perisinusoidal space of disse includes 4 types of cells. Which cell
corresponds to this description have a stellate shape , fat droplets , a moderately
developed endoplasmic reticulum are determined in their cytoplasm . Fat soluble
vitamins are deposited under physiological conditions - ito cells
45. The child has damaged the lateral surface of the tongue. Which papillae of the tongue
are most likely damaged - circumvallate
46. What kind of dentin is formed during the formation and eruption of a tooth - transparent
47. Name the most numerous cell in pancreatic islet - B cells
48. In the histological specimen an organ is seen. Wall of this organ consists of 4 layers first
is mucous layer forming pits and glands , second is submucous layer of connective
tissue , third is muscularis externa containing three layers of stomach muscles , fourth is
serous layer. Which organ can be recognized in the specimen - a stomach
49. On a histological specimen of the wall of the small intestine at the bottom of the crypts ,
the cells located in the groups are determined in the apical part of which there are large
acidophilic secretory granules , the cytoplasm is basophilic. What kind of cells are they -
paneth cells
50. After examination of a patient with small intestine disease disturbance of terminal
digestion was found. Terminal digestion id associated with which of the following cell
type - surface absorptive cells
51. Name the area of hypomineralized dentin - mantle dentin
52. Patient 42 years old, is 2 years after total resection of the stomach. He complains of
general weakness, lost appetite , paleness . Doctor put him diagnosis pernicious
anemia.Which of the following cell types absence does initiate this pathology -
parietal(oxyntic) cells
53. Name the epithelial cell of the small intestine involved in the transport of antigent into the
lamina propria mucosa followed by stimulation of the immune response - M-cells
54. In a histological specimen of a hollow organ the lining stratified squamous
nonkeratinized epithelium is seen in the mucosa . What organ is it - esophagus
55. In light microscope section you may see epithelial cells are grouped in interconnected
plates that form polygonal lobule. The space between these plates contains sinusoidal
capillaries . What organ is presented - liver
56. After two months of appendectomy the patient again has arrived in surgical department
with the diagnosis the intestinal obstruction an adhesive disease during operation
damage of what epithelium covering a peritoneum has reduced in development of
adhesive disease - mesothelium
57. The epithelial enamel organ is observed in maxillas histological section of 3.5 month
human fetus. Mesenchyme cells surround this enamel organ . What is name of these
cells - dental sac
58. A sick child has a white coating on the tongue. Due to which papillae does this
phenomenon occur - filiformis papillae
59. What is hard tissue of the tooth, which contains 70% of inorganic substances - dentin
60. A low level of albumin and fibrinogen was found in the patients blood . A decrease in the
activity of which organelles of hepatocytes of the liver is most likely due to this
phenomenon - RER
61. Secretion of which gastrointestinal hormone is primarily decreased in a patient with
removed duodenum - cholecystokinin and secretion
62. Section of basal portion of intestinal crypts of lieberkuhn showing the basophilic cells
with large acidophilic granules . what type of cell is it - paneths cells
63. The histological slide shows an organ of oral cavity consisting of three principal parts :
cutaneous , intermediate , mucous. What organ is described here - lip
64. Name the localization of the odontoblast bodies - dentin
65. During an embryogenesis there was a breaking shaping of a pharyngeal gut . specify
possible localization of anomalies of development - organs of an oral cavity
66. The lining of the large intestine differs from the lining of the small intestine - lack of villi
67. Where does acinocyte enzyme activation occur - inside the cells
68. It was determined the fall of activity of amylase in saliva owing to investigation of 47
years old man with extracted salivary gland. What gland was extracted - parotid
69. Examiantion of a patient with a disease of the small intestine revealed a violation of
parietal and membrane digestion. With the dysfunction of which cells is it connected -
absorbtive cells
70. Name the type of tongue papillae present only in childhood - foliate
71. Which part of the tongue has a submucous membrane - ventral
72. Which gland produces 70% of saliva - submandibular
73. Name the pancreatic islet endicrine cells, which inhibits activity of acinar cells of
pancreas. - D cells
74. What are the structures of the ovary that produce progesterone - granulosa lutein cells of
the corpus luteum
75. Name the follicle in the wall of which the theca appears - secondary
76. WhatisthebeginningofdevelopmentofRS - 3-4months
The patient with complaints to a pain in a thoracic cavity at respiration converted to the doctor.
After survey there were detected modifications of an epithelium of a pleura.What epithelium has
tested modifications - simple squamous

You might also like